<<

Geography Previous Year Questions

www.laex.in Page No. 1 https://elearn.laex.in Geography Previous Year Questions

www.laex.in Page No. 1 https://elearn.laex.in Geography UPSC Previous Year Questions

Geography UPSC Previous Year Questions

INDEX No. of Page No. SL.No. Topic Qns From To I. Geography 01. Astronomy 21 1 9 02. Geomorphology 22 9 18 03. Oceanography 12 19 24 04. Climatology 50 25 48 05. World Physical Features 66 48 71 II. Indian Geography 01. Indian Physiography 50 73 93 02. Rivers 32 93 107 03. Indian 10 107 112 04. Soil 8 112 115 05. Minerals 20 115 124 06. Energy 13 124 129 07. Forest 27 130 142 08. Agriculture 36 142 159 09. Industry 11 161 166 10. Transport 18 166 172 III.Demography 01. World Demography 9 173 176 02. Language 6 176 178 03. 31 178 190 04. 12 190 194

www.laexias.com https://elearn.laex.in Geography UPSC Previous Year Questions

I. World Geography 4. In order of their distances from the Sun, which of the following planets lie 1. Astronomy between Mars & Uranus? (a) and Jupiter 1. On 21st June, the Sun (b) Jupiter and Saturn (a) Does not set below the horizon at the (c) Saturn and Earth Circle (d) Saturn and Neptune (b) Does not set below the horizon at Circle 5. What is the average distance (c) Shines vertically overhead at noon on the (approximate) between the Sun & the Equator Earth? 5 (d) Shines vertically overhead at the Tropic of (a) 70 x 10 km 5 Capricorn (b) 100 x 10 (c) 110 x 106 km 2. What is the difference between asteroids (d) 150 x 106 km and comets? 1) Asteroids are small rocky planetoids, while 6. (Assertion): To orbit around the Sun, the comets are formed of frozen gases held planet Mars takes lesser time than the time together by rocky and metallic material taken by the Earth 2) Asteroids are found mostly between the (Reason): The diameter of the planet Mars orbits of Jupiter and Mars, while comets are is less than that of the Earth found mostly between Venus & (a) Both A are R are true R is the correct 3) Comets show a perceptible glowing tail, explanation of A while asteroids do not (b) Both A and R are true but R is not a correct Which of the statements given above is/are explanation of A correct? (c) A is true but R is false (a) 1 and 2 only (d) A is false but R is true

(b) 1 and 3 only 7. (Assertion): The same face of the Moon is (c) 3 only always presented to the Earth (d) 1,2 and 3 only (Reason): The Moon rotates about its own 3. Which one of the following planets has axis in 231/2 days which is about the same largest number of natural satellites or time that it takes to orbit the Earth moons? (a) Both A are R are true R is the correct (a) Jupiter explanation of A (b) Mars (b) Both A and R are true but R is not a correct (c) Saturn explanation of A (d) Venus (c) A is true but R is false (d) A is false but R is true

www.laexias.com Page No. 1 https://elearn.laex.in Geography UPSC Previous Year Questions

8. (Assertion): Existence of human life on 12. A “Black Hole “is a body in space which Venus is highly improbable does not allow any radiation to come out. (Reason): Venus has extremely high level of This property is due to its carbon di oxide in its atmosphere (a) Very small size (a) Both A are R are true R is the correct (b) Very large size explanation of A (c) Very high density (b) Both A and R are true but R is not a correct (d) Very low density explanation of A 13. At which one of the following positions (c) A is true but R is false shown in the diagram will the height of (d) A is false but R is true the tide be maximum? 9. Among the following which planet takes maximum time for one revolution around the Sun? (a) Earth (b) Jupiter (c) Mars (d) Venus

10. Which one of the following statements is (a) M1 correct with reference to our solar (b) M2 system? (c) M3 (a) The Earth is the densest of all the planets in (d) M4 our solar system (b) The predominant element in the 14. One Astronomical Unit is the average composition of Earth is silicon distance between (c) The Sun contains 75 percent of the mass of (a) Earth and the Sun the solar system (b) Earth and the Moon (d) The diameter of the Sun is 190 times that of (c) Jupiter and the Sun the Earth (d) Pluto and the Sun

11. If the stars are seen to rise perpendicular 15. Consider the following statements to the horizon by an observer, he is regarding asteroids: located on the 1) Asteroids are rocky debris of varying sizes (a) Equator orbiting the Sun (b) Tropic of Cancer 2) Most of the Asteroids are small but some (c) South Pole have diameter as large as 1000 km (d) North Pole

www.laexias.com Page No. 2 https://elearn.laex.in Geography UPSC Previous Year Questions

3) The orbit of asteroids lies between the orbits 18. A person stood alone in a on a of Jupiter & Saturn dark night and wanted to reach his Which of the statements given above is/are village which was situated 5 km east of correct? the point where he was standing. He had (a) 1,2 and 3 are correct no instruments to find the direction but (b) 2 and 3 are correct he located the polestar. The most (c) 1 and 2 are correct convenient way now to reach his village (d) 1 and 3 are correct is to walk in the (a) direction facing the polestar 16. Match List-I with List II and select the (b) direction opposite to the polestar correct answer using the codes given (c) direction keeping the polestar to his left below the lists: (d) direction keeping the polestar to his right List-I (Special List-II (Name Characteristics) of the Planet) 19. What is the approximate mean velocity A. Smallest planet of the 1. Mercury with which the Earth moves round the Solar System Sun in its orbit? B. Largest planet of the 2. Venus (a) 20km/s Solar System (b) 30km/s C. Planet second from the 3. Jupiter (c) 40km/s Sun in the solar system (d) 50km/s

D. Planet nearest to the Sun 4. Pluto 20. Which one of the following scholars 5. Saturn suggests the earth's origin is from gases Codes: and dust particles? (a) A-2; B-3; C-5; D-1 (a) James Jeans (b) A-3; B-5; C-1; D-2 (b) H Alfven (c) A-4; B-1; C-2; D-3 (c) F Hoyle (d) A-4; B-3; C-2; D-1 (d) O Schmidit

17. The terms 'Event Horizon', 'Singularity', 21. Sun's halo is produced by the refraction 'String Theory and Standard Model' are of light in sometimes seen in the news in the (a) Water vapour in Stratus clouds context of (b) Ice crystals in Cirro-Cumulus clouds (a) Observation and understanding of the (c) Ice crystals in Cirrus clouds Universe (d) Dust particles in Stratus clouds (b) Study of the solar and the 1lunar eclipses (c) Placing satellites in the orbit of the Earth (d) Origin and evolution of living organisms on the Earth

www.laexias.com Page No. 3 https://elearn.laex.in Geography UPSC Previous Year Questions

The new asteroids are being discovered 1. Key and Explanation continuously. 1. Key: A Comets: Comets are the most exciting heavenly bodies and have ever been the objects of man’s Explanation: The day 21 June is known as curiosity as well as fear. The word is summer solstice. The rays of the sun fall Comet vertically at the tropic of cancer (23½º N). The derived from the Greek word Aster Kometes North Pole is inclined or tilted towards the sun. meaning ‘Long Haired Star’. They are made up It, therefore, experiences complete light for 24 of small ice particles and meteoric fragments. hours. The South Pole is tilted away from the They revolve around the Sun. But their orbits sun so it is in complete darkness for 24 hours. It are irregular. Sometimes they get very close (Perihelion) to the sun and in other times they go is summer in the and winter in the . In the far away (Aphelion) from the sun. Northern hemisphere, the days are longer than Educational Objective: Understanding the nights. Asteroids and Comets. Position of the earth on 22 December- the South 3. Key: A Pole is inclined towards the sun and the North Explanation: Jupiter has large number of Pole is away from it. The rays of the sun fall natural satellites, which has 67 natural vertically at the tropic of Capricorn (23½º S). The satellites. Mars has 2 natural satellites. Saturn greater part of the southern hemisphere gets the has 53 natural satellites. Venus and Mercury direct rays of the sun so the days are long and don’t have any natural satellite. the nights are short here. In the northern Educational Objective: Understanding our hemisphere the nights are longer than the days solar system. at this time. The southern hemisphere has 4. Key: B summer. The northern hemisphere has winter. Explanation: Saturn and Jupiter planets are This day (22 December), when the sun’s rays fall placed between Mars and Uranus. Jupiter and vertically on the Tropic of Capricorn, is known Saturn are 778.6 million kilometer (mkm), as winter solstice. 1433.5mkm from Sun respectively. Educational Objective: Understanding solstices and equinoxes Mer- Ven- Ear Jup Sat Ura Nep Planet Mars Cury us th iter urn nus tune 2. Key: B Average Explanation: Asteroids: Asteroids are a small distanc rocky celestial body that revolves around the e 57.9 108.2 149.6 227.9 778.6 1433.5 2872.5 4495.1 from Sun, like other planets. They are also called the sun ‘Minor Planets’ or planetoids. There are lots of Mer- Ven- Ear Jup Sat Ura Nep asteroids in the solar system. These are found in Planet Mars between the planets Mars and Jupiter. This belt Cury us th iter urn nus tune Natural is known as ‘Asteroid belt’. The diameter of the Satellit 0 0 1 2 67 53 27 13 asteroids varies from100 km to a size of a pebble. e The asteroids may be the fragments of a planet exploded in the past or some parts of comets.

www.laexias.com Page No. 4 https://elearn.laex.in Geography UPSC Previous Year Questions

The Jupiter: Jupiter is the largest planet in the one rotation on its own axis. The Earth takes solar System. It is made primarily of gases and 365.25 days to complete one revolution around is therefore known as ‘Giant Gas planet’. It the Sun. Earth’s surface temperature varies takes 9 hours 55 minutes to complete one from – 88q to 58qC and it is the densest planet rotation on its axis and it takes 11.86 years to in the solar system. The Earth is a unique planet complete one revolution. Jupiter has the because of its distance from the sun, its motions, shortest day in the solar system. Jupiter has a atmosphere with oxygen, presence of water and faint ring system around it. They are mostly moderate temperature. The earth is neither too comprised of dust particles. Jupiter has 67 close nor too far from the sun. It is the only confirmed satellites orbiting the planet. known planet to support life. It is also known as Ganymede, the satellite of Jupiter, is the largest the ‘Blue Planet’ because of the presence of natural satellite in the solar system (even bigger water. Earth has only one natural satellite called than the planet Mercury). the Moon. The sun light takes about 8.3 minutes to reach the earth. The Saturn: Saturn is the sixth planet from the Educational Objective: Understanding our sun and the second largest planet in the solar Solar system system. Saturn is called as the Ringed Planet. It is because of large, beautiful and extensive ring 6. Key: D systems that encircles the planet. These rings Explanation: To orbit around the Sun, the are mostly made from the chunks of ice and Planet Mars takes 687 days but earth takes carbonaceous dust. Saturn is the only planet in 365.25 days. The diameter of planet earth is our solar system whose average density is less 6794km. Earth diameter is 12756km.

than water. The Saturn has 30 rings and Mer Planets Venus Earth Mars Jupiter Saturn Uranus Neptune 53confirmed natural satellites. The Saturn takes cury 10 hours 34 minutes to complete one rotation Dia 4879 12,104 12,756 6,794 1,42,984 1,20,536 51,118 49,528 on its axis and it takes 29.4 years to complete meter(km) one revolution around the sun Educational Objective: Understanding Solar Educational Objective: Understanding our System Solar system 7. Key: C 5. Key: D Explanation: The moon is located at a distance Explanation: The average distance between of 8,84,401 km from the earth the moon revolves Earth and Sun is 150 million kilometre around the earth. The moon takes 27 days and 7 The Earth: Earth is the third nearest planet to hours and 43 minutes for both its rotation and the sun. It is the fifth largest planet in the solar revolution around the earth. Hence, the system. The Earth’s orbit lies between the orbits observers on the earth can see only one side of of Venus and Mars. It takes 23 hours 56 the moon. The moon is the fifth largest natural minutes and 4 seconds for the earth to complete satellite in the solar system. The moon was likely

www.laexias.com Page No. 5 https://elearn.laex.in Geography UPSC Previous Year Questions

to be formed after a Mars sized body collided Mars, Venus takes 365.25 days, 4331days, with Earth. There are many craters, high and 224.7days respectively. steep mountains of different sizes which cast Educational Objective: Understanding solar shadows on the Moon’s surface. The light which system is reflected by the Moon will reach the Earth in 10. Key: A just one and a quarter seconds. Since the moon Explanation: is smaller than the earth, it has 1/6 of the Merc Ve Ear Mar Jupite Satur Uranu Neptu gravitational pull of the earth. So, man weighs 6 Planets ury nus th s r n s ne times less on the moon than the earth. Educational Objective: Understanding Orbital period in 88 224 365 687 4,331 10747 30,589 59,800 natural satellite-moon. days 8. Key: A The Sun: The Sun is at the centre of our solar Explanation: The Venus: ‘Venus’ is the second system. It is a yellow dwarf star, with a hot ball nearest planet to the sun. It is also called as of glowing gases. Its gravity holds the solar ‘Earth’s Sister’ planet due to its similar size system together and it keeps everything from the and mass as that of our Earth. biggest planets to the smallest particles of debris Its atmosphere mainly consists of carbon dioxide in its orbit. Electric currents in the Sun generate with clouds of sulphuric acid, and only traces a magnetic field that is carried out through the amounts of water. The atmosphere is heavier solar system by the solar wind. than that of any other planet, leading to a surface pressure that's over 90 times that of Structure of the Sun: By mass, the Sun is Earth — similar to the pressure that exists made up of about 70.6% hydrogen and 27.4% helium. The Sun's enormous mass is held 3,300 feet (1,000 meters) deep in the ocean. It is together the lower of the solaratmosphere also the hottest planet in the solar system and is called the chromospheres. Its name is derived experiences a mean surface temperature of from the Greek word chroma (meaning colour), 462’C due to green house effect. for it appears bright red when viewed during a solar eclipse. A thintransition region, It is popularly known as “Morning star and wheretemperature risessharply, separates the Evening star” It is seen in the east sky before chromospheres fromthe vast corona above. The sunrise (dawn) in the morning and in the west upper mostportion of the Sun's atmosphere is calledthe corona, and is surprisingly much sky after the sunset (twilight).It rotates clockwise hotterthan the Sun's surface (photosphere) i.e. east to west direction on its own axis. theupper corona gradually turns into the Educational objective: Understanding solar solarwind. Solar wind is a flow of plasma thatmoves outward through our solar system systeminto interstellar space. 9. Key: B The Earth: Earth is the third nearest planet to Explanation: Jupiter takes 10,747 days to the sun. It is the fifth largest planet in the solar complete one revolution around sun. Earth, system. The Earth’s orbit lies between the orbits

www.laexias.com Page No. 6 https://elearn.laex.in Geography UPSC Previous Year Questions

of Venus and Mars. It takes 23 hours 56 north celestial pole moves closer to the zenith as minutes and 4 seconds for the earth to complete the observer moves towards the North and the one rotation on its own axis. The Earth takes two coincide at the North Pole. Similarly as the 365.25 daysto complete one revolution around observer moves south, the south celestial pole the Sun. Earth’s surface temperature varies moves closer to the zenith and the two coincide from – 880 to 580C and it is the densest planet at the South Pole. in the solar system. The Earth is a unique Educational Objective: Understanding planet because of its distance from the sun, its positions of stars with respect to earth. motions, atmosphere with oxygen, presence of 12. Key: C water and moderate temperature. The earth is Explanation: A black hole is an object in space neither too close nor too far from the sun. It is that is so dense and has such strong gravity the only known planet to support life. It is also that no matter or light can escape its pull. known as the ‘BluePlanet’ because of the Because no light can escape, it is black and presence of water. Earth has only one natural invisible. There’s a boundary at the edge of a satellite called the Moon. The sun light takes black hole called the event horizon, which is the about 8.3minutes to reach the earth. point of no return — any light or matter that Educational objective: understanding Earth crosses that boundary is sucked into the black and Sun hole. It would need to travel faster than the 11. Key: A speed of light to escape, which is impossible. Explanation: The celestial equator is an Anything that crosses the event horizon is imaginary circle that goes around the sky destined to fall to the very centre of the black directly above the equator. All the stars rotate in hole and be squished into a single point with a path parallel to the equator that is always 90 infinite density, called the singularity. degrees from the North and South Poles. In Educational Objective: to understand Black astronomy the celestial sphere refers to an holes

imaginary sphere with the Earth at the center of 13. Key: D the sphere. Though the sky extends infinitely in Explanation: Tide is caused by the rotation of all directions stars are assumed to lie on the the earth apart from the gravitational pull of the surface of the celestial sphere from different sun and the moon. The rhythmic rise and fall of locations on the surface of the Earth the the water due to gravitational pull of the visibility of stars changes. The zenith refers to a moon and the sun is called a Tide. Isaac Newton point directly above the observer, which is (1642–1727) was the first person to explain tides defined by the line from the center of the Earth, scientifically. The rise of seawater towards the and perpendicular to the horizon. land is known as High tide or flow tide. The fall If the observer is at the Earth's equator, the of seawater more towards sea is known as ‘Low celestial equator passes though the zenith. The tide water’ or ebb tide. On any day there will be

www.laexias.com Page No. 7 https://elearn.laex.in Geography UPSC Previous Year Questions

two high tides and two low tides. The highest Educational Objective: Solar system. high tide occurs on full moon day and new moon 17. Key: A day. It is known as spring tide Spring tide Explanation: In astrophysics, an event happens when the sun, earth and moon aligned horizon is a boundary beyond which events in straight line. The lowest low tide is known as cannot affect an observer. The initial neap tide. It happens when the sun, earth and singularity is a gravitational moon are positioned at right angles. singularity predicted by general relativity to have Tides and types of tides Educational Objective: existed before the Big Bang and thought to have 14. Key: A contained all the mass and space-time of Explanation: One astronomical Unit (AU) is the the Universe. string theory is a theoretical average distance from the Sun to Earth, or framework in which the point-like about 150 million kilometres. particles of particle physics are replaced by one- Educational Objective: Understanding dimensional objects called strings. It describes Astronomy how these strings propagate through space and interact with each other. On distance scales 15. Key: C larger than the string scale, a string looks just Explanation: Asteroids: Asteroids are small like an ordinary particle, with its mass, charge, rocky celestial bodies.That revolves around the and other properties determined by Sun, like other planets. They are also called the vibrational state of the string The Standard ‘Minor Planets’. There are lots of asteroids in the Model of particle physics is the theory solar system. Larger asteroids are called describing three of the four known fundamental Planetoids. These are found in between the forces (the electromagnetic, weak, planets Mars and Jupiter. This belt is known as and strong interactions, and not including ‘Asteroid belt’. The diameter of the asteroids the gravitational force) in the universe, as well varies from100 km to a size of a pebble. The as classifying all known elementary particles asteroids may be the fragments of a planet Educational Objective: Current Astronomy exploded in the past or some parts of comets. The new asteroids are being discovered 18. Key: C continuously. Explanation: The pole star or Polaris is the Educational Objective: Asteroids brightest star in the constellation of Ursa Minor which is located in the North direction always. 16. Key: D Explanation: Smallest planet of the solar system is Pluto. Largest planet of the solar system is Saturn. Planet second from the Sun in the solar system is Venus. Planet nearest to the Sun is Mercury.

www.laexias.com Page No. 8 https://elearn.laex.in Geography UPSC Previous Year Questions

 As pole star is to the north and he has to 2. Geomorphology move to east to reach his village pole star will lay on his left as he moves providing 1. Consider the following statements: him correct direction to reach there. 1) The Earth's magnetic field has reversed Educational Objective: To know about location every few hundred thousand years. of important objects in space and their 2) When the Earth was created more than importance for navigation and various other 4000 million years ago, there was 54% purposes. oxygen and no carbon dioxide. 3) When living organisms originated, they 19. Key: B modified the early atmosphere of the Earth. Explanation: The mean velocity of earth which Which of the statements given above is/are moves around the sun is calculated by correct? Speed of Revolution = Distance/Time = (a) 1 only 942,000,000 km / 8766 hr = 107,000 km/hr = (b) 2 and 3 only 30 km/sec. (c) 1 and 3 only Educational Objective: Planetary physics (d) 1,2 and 3 20. Key: D 2. Which of the following phenomena might Explanation: Schmidt suggests that the earth's have influenced the evolution of origin is from gases and dust particles. O. organisms? Schmidt in 1943 gave Inter-Stellar Dust 1) Continental drift Hypothesis for the origin of the earth and solar 2) Glacial cycles system. Select the correct answer using the code Educational Objective: formation of earth given below. 21. Key: C (a) 1 only Explanation: Sun’s halo is also known is also (b) 2 and 3 only known as 22 degree halo. It is produced by the (c) Both 1 and 2 refraction of light in ice crystals in Circus clouds (d) 1, 2 and 3 within the Earth’s Atmosphere. A halo ring is a 3. On the planet earth, most of the ring or light that forms around the Sun or Moon freshwater exists as ice caps and glaciers. as the Sun’s or Moon’s light refracts off the ice Out of the remaining freshwater, the crystals present in a thin veil of cirrus clouds. largest proportion Educational Objective: Understanding Sun and (a) Is found in atmosphere as moisture and the concept of halo formation. clouds (b) Is found in freshwater lakes and rivers (c) Exists as groundwater (d) Exists as soil moisture

www.laexias.com Page No. 9 https://elearn.laex.in Geography UPSC Previous Year Questions

4. Consider the following: Which of the statements given above is/ are 1. Electromagnetic radiation correct? 2. Geothermal energy (a) 1 only 3. Gravitational force (b) 2 only 4. Plate movements (c) Both 1 and 2 5. Rotation of the earth (d) Neither 1 nor 2 6. Revolution of the earth 7. In the structure of planet Earth, below Which of the above are responsible for the mantle, the core is mainly made up bringing dynamic changes on the surface of of which one of the following? the earth? (a) Aluminium (a) 1, 2, 3 and 4 only (b) Chromium (b) 1, 3, 5 and 6 only (c) Iron (c) 2, 4, 5 and 6 only (d) Silicon (d) 1, 2, 3, 4, 5 and 6 only 8. Match List I with List II and select the 5. The Brahmaputra, Irrawady and Mekong correct answer using the code given rivers originate in and flow through below the lists: narrow and parallel mountain ranges in List-I (Geographic List-II (Country) their upper reaches. Of these rivers, feature) Brahmaputra makes a "U" turn in its A. Great Desert 1. course to flow into . This "U" turn is B. Grand Canyon 2. due to C. Lake Winnipeg 3. (a) Uplift of folded Himalayan series D. Southern Alps 4. USA (b) Syntaxial bending of geologically young Codes: (a) A-1; B-2; C-4; D-3 (c) Geo-tectonic disturbance in the tertiary (b) A-1; B-4; C-2; D-3 folded mountain chains (c) A-3; B-2; C-4; D-1 (d) Both (a) and (b) above (d) A-3; B-4; C-2; D-1 6. Consider the following statements: 9. Consider the following statements: 1) On the planet Earth, the fresh water 1) The Richter scale is a logarithmic scale and available for use amounts to about less than so an increase of 1 magnitude unit 1 % of the total water found. represents a factor of 10 times in amplitude. 2) Of the total fresh water found on the planet 2) Each integer reading of the Richter scale Earth 95% is bound up in polar ice caps has energy 100 times that of the previous and glaciers integer reading.

www.laexias.com Page No. 10 https://elearn.laex.in Geography UPSC Previous Year Questions

Which of the statements given above is/are 2) The formation of sedimentary rocks involves correct? the weathering of pre-existing rocks (a) 1 only 3) Sedimentary rocks contain fossils (b) 2 only 4) Sedimentary rocks typically occur in layers (c) Both 1 and 2 Which of these statements are correct? (d) Neither 1 nor 2 (a) 1 and 2 (b) 1 and 4 10. Consider the following geological (c) 2,3 and 4 phenomena: 1) Development of a fault (d) 1, 2, 3 and 4 2) Movement along a fault 13. Quartzite is metamorphosed from 3) Impact produced by a volcanic eruption (a) Limestone 4) Folding of rocks (b) Obsidian Which of the above cause ? (c) Sandstone (a) 1, 2 and 3 (d) Shale (b) 2 and 4 14. Which one of the following pairs is (c) 1, 3 and 4 correctly matched? (d) 1, 2, 3 and 4 Geographical Feature Region 11. Consider the following statements: (a) Abyssinian Plateau Arabia 1) Most magmas are a combination of liquid, (b) North-Western solid and gas 2) Water vapour and carbon dioxide are the (c) Guiana Highlands South-Western principal gases dissolved in a magma Africa 3) Basaltic magma is hotter than the silicic magma (d) Okavango Basin 4) The magma solidified between sedimentary 15. Where is the volcanic mountain, Mount rocks in a horizontal position is known as dyke St Helens located? Which of these statements are correct? (a) Chile (a) 1, 2 and 3 (b) Japan (b) 1 and 4 (c) (c) 2, 3 and 4 (d) of America (d) 1, 2 and 4 16. Volcanic eruptions do not occur in the 12. Consider the following statements made (a) about the sedimentary rocks: (b) 1) Sedimentary rocks are formed at earth's (c) Sea surface by the hydrological system (d)

www.laexias.com Page No. 11 https://elearn.laex.in Geography UPSC Previous Year Questions

17. Match List-I with List-II and select the 3. Maximum percentage of Earth's water is in correct answer using the codes given the . below the lists: Which of the statements given above is/are List-I List II (Country) correct? (Volcanic Mountain) (a) 1 and 2 A. Mt. Rainier 1. Italy (b) 2 and 3 B. Etna 2. (c) 1 only C. Paricutin 3. Philippines (d) 3 only

D. Taal 4. USA 20. Consider the following statements: Codes: 1. The axis of the earth's magnetic field is a. A-4; B-2; C-1; D-3 inclined at 23 to the geographic axis of the b. A-4; B-1; C-2; D-3 earth. c. A-2; B-1; C-4; D-3 2 The earth's magnetic pole in the northern hemisphere is located on a peninsula in d. A-4; B-3; C-2; D-1 18.Consider the following statements 3. Earth's magnetic equator passes through regarding the earthquakes: Thumba in . 1) The intensity of is measured on Which of the statements given above is/are Mercalli Scale correct? 2) The magnitude of an earthquake is a (a) 1,2 and 3 measure of energy release (b) 2 and 3 3) Earthquake magnitudes are based on direct (c) 2 only measurements of the amplitude of seismic (d) 3 only waves 21. Assertion: The weight of a body decreases 4) In the Richter Scale, each whole number with the increase latitude on earth. demonstrates a hundred fold increase in the Reason: The earth is not a perfect sphere. amount of energy released (a) Both A are R are true R is the correct Which of these statements are correct? explanation of A (a) 1, 2 and 3 (b) Both A and R are true but R is not a correct (b) 2, 3 and 4 explanation of A (c) l and 4 (c) A is true but R is false (d) 1 and 3 (d) A is false but R is true

19. Consider the following statements: 22. Assertion: The amount of moisture in the 1. Total land area of Earth is approximately atmosphere is related to latitude. 1475 lakh square kilometres. Reason: The capacity to hold moisture in 2. Ratio of land area to water area of Earth is the form of water vapour is related to approximately 1:4 temperature.

www.laexias.com Page No. 12 https://elearn.laex.in Geography UPSC Previous Year Questions

(a) Both A are R are true R is the correct may easily bring about new ones or makes some explanation of A species go into extinction. (b) Both A and R are true but R is not a correct Since Environmental changes act as the motor explanation of A of evolution Glacial cycles also have influenced (c) A is true but R is false the evolution of organisms. (d) A is false but R is true Educational Objective: To study about the different stages of evolution of earth and the 2. Key and Explanation factors that were responsible for the evolution

1. Key: C 3. Key: C Explanation: According to Scientist the Earth's Explanation: Distribution of the water on Earth magnetic field has reversed every few hundred is as follows: thousand years, but there is no apparent Ocean water : 97.2 percent periodicity to the occurrence. The reversal of Glaciers and other ice : 2.15 percent polarity is a random event. Groundwater : 0.61 percent At the time when earth was created there was no Fresh water lakes : 0.009 percent atmosphere. Thus statement 2 is wrong. Inland : 0.008 percent Initially life might have started anaerobic Soil Moisture : 0.005 percent respiration. later on the process of Atmosphere : 0.001 percent photosynthesis and other process slowly Rivers : 0.0001 percent modified the early atmosphere of the Earth. From this we can tell that among the given Thus statement 3 is correct. options, groundwater has most of the earth’s Educational Objective: To learn about the fresh water interior of the earth and the processes that are Educational Objective: To learn about the undergoing. Also have knowledge about the water table and it’s distribution on the Earth evolution of the earth 4. Key: D 2. Key: C Explanation: Among the given forces, we know Explanation: According to the theory of plate that plate movements obviously cause dynamic tectonics, the tectonic plates drift around the changes as explained in Plate Tectonics globe, either toward or away from each other. In Theory. this process, the seem to "drift" Rotation and Revolution of the earth causes around Earth from one climatic region to variation in day and night and seasons another, over a span of millions of years. respectively. The Continental drift has a major impact on Hence, they too change the surface of earth as evolution of life, because of climate change that variation in seasons can cause or occurs during continental drift. This process droughts.

www.laexias.com Page No. 13 https://elearn.laex.in Geography UPSC Previous Year Questions

Gravitational force causes tides which affects glaciers, and just over 30 percent is found in coastal areas. Geothermal energy to hot ground water. springs that would also change the surface of Due to climate change and increase in global earth. Hence, option D is correct warming, the glaciers and the ice caps are Educational Objective: To have knowledge melting at very high rate. about the different forces existing in the interior Educational Objective: To know about the of the earth and also exterior of the Earth. Also percentage of water on the earth. Also about the have to learn their effects on the surface of the fresh water percentage and ocean water.And earth also about the forms that the water takes like river water, ocean water, glaciers, water vapor 5. Key: D etc. Explanation: The Indian lithospheric plate started drifting toward the present location 200 7. Key: C million years ago , The collision of Indian plate Explanation: Earth is divided in to multiple with Eurasian plate resulted in series of folding, layers such as crust, Mantle and Core. faults and upliftments, which resulted in The Earth's crust is made up of several elements upliftment of Himalayan ranges. like oxygen, silicon, aluminium, iron, sodium, The Syntaxial bent was created along the potassium, and magnesium. Himalaya due to the impact of collision. as result Whereas the mantle under the crust is of this bent the takes a U composed mostly of silicate rocks rich in turn on reaching the Namcha Barwa peak and magnesium and iron. enters India in through a The centre of the Earth is the core , both inner gorge. and outer core combined accounts for one third Educational Objective: To know about the of earth mass, It is mainly composed of a nickel- important rivers in India and their pattern of iron (85% of mixture is iron) alloy . Thus flow. And also the physical features that they statement 3 is correct. form while flowing. Educational Objective: To have knowledge 6. Key: A about the structure of the interior of the earth. Explanation: Even though 71 percent of the And also about various layers and their Earth's surface is covered by water. More than composition. 99 percent of it (Earth's water) is unusable by 8. Key: B humans and many other living things . Only Explanation: The is found about 0.3 percent of our fresh water is found in in the south western region of the Australia. the surface water of lakes, rivers and swamps The Grand Canyon is a steep-sided canyon are useable. carved by the Colorado River in the state of Over 68 percent of the fresh water available on Arizona, United States of America. the Earth surface is found in icecaps and

www.laexias.com Page No. 14 https://elearn.laex.in Geography UPSC Previous Year Questions

Lake Winnipeg is a large and shallow fresh water on the surface of the Earth directly above a lake located in the province of Manitoba, Canada. hypocentre or focus. The Southern Alps are the mountain range Earthquakes are mainly caused by rupture of found in New Zealand. The Southern Alps run geological faults but also by other events such approximately 500 km in northeast to southwest as Impact produced by a volcanic activity, direction. The tallest peak in Southern Alps is landslides, mine blasts, and nuclear tests. Aoraki / Mount Cook, which is also the highest When the epicentre of an earthquake is located point in New Zealand. offshore, the displacement of seabed may result Educational Objective: To have knowledge in a tsunami. about the salient geographical features across Educational Objective: To know about the the world and also in India causes and effects of the earth quake

9. Key: C 11. Key: A Explanation: Richter scale is used in Explanation: Magma is extremely hot liquid and quantitative measure of an earthquake’s semi-liquid rock located under Earth’s surface. magnitude. The earthquake’s magnitude is The magma that erupts onto the Earth's surface determined using the logarithm of the amplitude is known as lava. Like solid rock magma is also of the largest seismic wave calibrated to a scale a mixture of many minerals. by a seismograph. The magma also contains small amounts of The each increase of 1 unit on the scale dissolved gases such as water vapor, carbon represents a 10-fold increase in the magnitude dioxide, and sulphur. of an earthquake. That is each integer reading of Basaltic magma is hotter than the silicic magma. the Richter scale has an energy 100 times that Basaltic magma is rich in iron, magnesium, and of the previous integer reading. calcium but has very low percentage of Each increase of one unit also represents the potassium and sodium. The temperature of release of about 31 times more energy Basaltic magma ranges from about 1000oC to Educational Objective: To know about the 1200oC. earthquakes and the scales used to measure the Dikes are the volcanic landforms, When the earth quake magnitude. And also the difference magma is injected vertically as narrow walls of between Richter scale and mercalli scale igneous rocks within the sedimentary layer are termed as dykes. 10. Key: D Educational Objective: To know about the Explanation: An earthquake is the formation of Magma and their chemical vibration/shaking experienced on surface of the composition. Earth resulting from a sudden release of energy in the Earth's lithosphere that creates seismic 12. Key: D waves. In seismology the epicentre is the point Explanation: Sedimentary rocks are created from the sediments accumulated over a long

www.laexias.com Page No. 15 https://elearn.laex.in Geography UPSC Previous Year Questions

period of time at earth's surface by the Atlas Mountains are the series of mountain hydrological system. ranges in northwestern Africa, running generally The sedimentary rocks are usually formed in southwest to northeast these mountain ranges layer and are often termed as stratified rocks separates the Mediterranean and Atlantic and these strata may vary from few centimetres coastlines from the Desert. to many feets. Guiana Highlands or plateau is a region in The formation of sedimentary rocks involves the located north of the Amazon river weathering of pre-existing rocks. and south of the Orinoco River. The Sedimentary rocks can in any form, fine The Okavango Basin is found in southwestern grained, coarse, hard or soft. Sedimentary rocks Africa, River Okavango is the chief stream in the often contain fossils of plants, Animals and basin, The basin extends across portions of micro organisms. Angola, Botswana, Namibia and Zimbabwe. Educational Objective: To know about the Educational Objective: To have knowledge various types of rocks and their formation. Also about the salient geographical features across about the chemical composition of sedimentary the world and also in India rocks, igneous rocks and metamorphic rocks

13. Key: C 15. Key: D Explanation: Under the great temperature and Explanation: On Earth surface, A volcano is a pressure any rock can undergo change, such vent or fissure in the earth's crust through rock are known as metamorphic rock. The which hot molten lava, ash and gases erupt. A character and the appearance of the volcanic mountain is formed by the metamorphosed rock is greatly different from the accumulation of this eruptive pyroclastic parent rock. material over the years. Ex: Clay can be metamorphosed in to Slate Mount St. Helens is an active stratovolcano, it is Sandstone can be metamorphosed in to in located in Skamania County, Washington, in the Quartzite. region of the United States of Lime stone can be metamorphosed in to Marble. America. Shale can be metamorphosed in to Schist. A Stratovolcanoes is also called as composite Educational Objective: To know about the volcanoes, because they are built of layers of various types of rocks and their formation. Also alternating lava flow, ash and sometimes blocks about the chemical composition of sedimentary of unmelted stone. rocks, igneous rocks and metamorphic rocks Educational Objective: To know about the various hotspots of volcano on the earth crust. 14. Key: B Also the locations of various volcanic mountains Explanation: Abyssinian Plateau is a plateau in and the type of volcano Ethiopia It is located at an elevation of 1,388 meters above sea level.

www.laexias.com Page No. 16 https://elearn.laex.in Geography UPSC Previous Year Questions

16. Key: A Educational Objective: To know about the Explanation: According to the theory of plate various hotspots of volcano on the earth crust. tectonics, the earth's lithosphere is broken in to Also the locations of various volcanic mountains major and minor plates. These tectonic plates and the type of volcano

drift around the globe, either toward or away 18. Key: A from each other. Explanation: An earthquake is the Volcanos usually occur at the edge of these plate vibration/shaking experienced on surface of the boundaries. But in case of Baltic sea, the Baltic Earth resulting from a sudden release of energy sea does not lie at the edge of these tectonic in the Earth's lithosphere that creates seismic plates but instead the basaltic rocks which waves. found in Baltic sea is transported by glacier. The intensity of earthquake is measured on Thus the possibility of volcanos in the Baltic Sea Mercalli Scale where as the magnitude of an is very less. earthquake is a measured on Richter Scale. Educational Objective: To know about the The earthquake’s magnitude is determined using locations where volcanic eruptions occur and the the logarithm of the amplitude of the largest or seas they are bordering with. seismic wave calibrated to a scale by a 17. Key: B seismograph. Explanation: On Earth surface, A volcano is a The each increase of 1 unit on the scale vent or fissure in the earth's crust through represents a 10-fold increase in the magnitude which hot molten lava, ash and gases erupt. A of an earthquake. That is each integer reading of volcanic mountain is formed by the the Richter scale has an energy 100 times that accumulation of this eruptive pyroclastic of the previous integer reading. material over the years. Educational Objective: To know about the Mount Rainier is a large active stratovolcano and earthquakes and the scales used to measure the It is the tallest mountain in Washington and the earth quake magnitude. And also the difference Cascade, in Mount Rainier National Park, USA. between Richter scale and mercalli scale

(A Stratovolcanoes is also called as composite 19. Key: D volcanoes, because they are built of layers of Explanation: Total land area of earth is around alternating lava flow, ash and sometimes blocks 500 million square kilometres. Statement 1 is of unmelted stone) incorrect. Ema Volcano in Italy is a active volcano. Ratio of land to water of Earth is 1 : 2 and not Whereas the Paricutin is a cinder cone volcano 1: 4. Statement 2 is also incorrect. located in the Mexico. We know that world's largest ocean is Pacific Taal Volcano is a large caldera filled by water and hence maximum percentage of water will be forming Taal Lake on the Island in the in Pacific Ocean. Statement 3 is correct. Philippines

www.laexias.com Page No. 17 https://elearn.laex.in Geography UPSC Previous Year Questions

Educational Objective: To have knowledge Explanation: Weight of a body is the force about the area of the land and water on earth. experienced by the body due to gravity. W =mg 20. Key: D The weight of a body decreases with increase Explanation: Magnetic dip, dip angle, or in distance from the earth's surface because magnetic inclination is the angle made with weight is inversely proportional to the distance the horizontal by the Earth’s magnetic field of body from the surface of earth. The decrease lines.Magnetic dip at the magnetic equator is 0, is not with respect to latitude. Hence, assertion and at the magnetic poles, it is 90. Hence, is false statement 1 is wrong The earth is not a perfect sphere. We know that, the shape of the earth is Geoid. It is bulge at equator and flat at poles. Hence, reason is true Educational Objective: To learn about the variation of weight with respect to latitude and longitude. Also have basic knowledge about the Earth's shape, structure etc. 22. Key: A Explanation: The amount of moisture (humidity) in the atmosphere decreases as we move from equator to poles. This is because of the temperature starts decreasing. Hence, Statement is correct and reason is the correct explanation of statement The North Magnetic Pole (86 N, 172 W) lie to the Educational Objective: To have knowledge north of Ellesmere about the interrelationship between the Island in northern Canada and is rapidly temperature, latitude, humidity etc. drifting towards . It is not in the peninsula of Canada. Hence, statement 2 is incorrect Earth’s magnetic equator passes through Thumba is correct. Educational Objective: To have knowledge about the Earth’s magnetic field and related information like the magnetic inclination, declination, current location etc., 21. Key: D

www.laexias.com Page No. 18 https://elearn.laex.in Geography UPSC Previous Year Questions

3) The survival of some animals that have 3. Oceanography phoplanktonic larvae will be adversely 1. Consider the following statements: affected. 1) Most of the world's coral reefs are in tropical 4) The cloud seeding and formation of clouds waters. will be adversely affected. 2) More than one-third of the world's coral Which of the statements given above is/are reefs are located in the territories of correct? Australia, and Philippines. (a) 1,2 and 3 only 3) Coral reefs host far more number of animal (b) 2 only phyla than those hosted by tropical (c) 1 and 3 only rainforests. (d) 1,2,3 and 4 Which of the statements given above is/are 4. The 2004 Tsunami made people realize correct? that mangroves can serve as a reliable (a) 1 and 2 only safety hedge against coastal calamities. (b) 3 only How do mangroves function as a safety (c) 1 and 3 only hedge? (d) 1,2 and 3 (a) The mangrove swamps separate the human 2. Tides occur in the oceans and seas due to settlements from the sea by a wide zone in which people neither live nor venture out which among the following? (b) The mangroves provide both food and 1) Gravitational force of the Sun medicines which people are in need of after 2) Gravitational force of the Moon any natural disaster 3) Centrifugal force of the Earth (c) The mangrove, trees are tall with dense Select the correct answer using the code canopies and serve as an excellent shelter given below. during a cyclone or tsunami (d) The mangrove trees do not get uprooted by (a) 1 only storms and tides because of their extensive (b) 2 and 3 only roots (c) 1 and 3 only 5. Consider the following statements: (d) 1, 2 and 3 1) Ocean currents are the slow-surface 3. The acidification of oceans is increasing. movement of water in the ocean Why is this phenomenon a cause of 2) Ocean currents assist in maintaining the concern? Earth's heat balance 1) The growth and survival of calcareous 3) Ocean current are set in motion primarily phytoplankton will be adversely affected. by prevailing winds 2) The growth and survival of coral reefs will 4) Ocean current are affected by the be adversely affected. configuration of the ocean

www.laexias.com Page No. 19 https://elearn.laex.in Geography UPSC Previous Year Questions

Which of these statements are correct? (d) Neither 1 nor 2 (a) 1 & 2 9. The largest coral reef in the world is (b) 2,3 &4 found near the coast of which one of the (c) 1,3 & 4 following countries? (d) 1,2,3 & 4 (a) Australia 6. Assertion: During the Neap Tides, the high (b) Cuba tide is lower and the low tide is higher than (c) Ghana usual. (d) Philippines Reason: The Neap Tide, unlike the Spring Tide, occurs on the New Moon instead of on 10. In which one of the following oceans is the Full Moon. Diamantina Trench situated? (a) Both A are R are true R is the correct (a) Pacific Ocean explanation of A (b) (b) Both A and R are true but R is not a correct (c) explanation of A (d) (c) A is true but R is false 11. Match the following and select the (d) A is false but R is true correct answer using the codes given 7. The most important fishing grounds of below: the world are found in the where Oceanic Trench Location (a) Warm and cold atmospheric currents meet A. Aleutian 1. Indian Ocean (b) Rivers drain out large amounts of B. Kermadec 2.North Pacific Ocean freshwater into the sea C. Sunda 3.South Pacific Ocean (c) Warm and cold oceanic currents meet D. S Sandwich 4.SouthAtlanticOcean (d) Continental shelf is undulating Codes: 8. Between India and East , the (a) A-2; B-4; C-1; D-3 navigation-time and distance can be (b) A-2; B-3; C-1; D-4 greatly reduced by which of the following? (c) A-1; B-3; C-2; D-4 1) Deepening the Malacca straits between (d) A-1; B-4; C-2; D-3 and Indonesia. 2) Opening a new canal across the Kra 12. In the given map, which one of the isthmus between the Gulf of Siam and following pairs of ocean currents is . shown? Which of the statements given above is/are correct? (a) 1 only (b) 2 only (c) Both 1 and 2

www.laexias.com Page No. 20 https://elearn.laex.in Geography UPSC Previous Year Questions

(a) Benguela and Falkland  The moon’s gravitational pull to a great (b) Canary and Humboldt extent and to a lesser extent the sun’s (c) Agulhas and gravitational pull, are the major causes for (d) Benguela and Guinea the occurrence of tides.  Another factor is centrifugal force which 3. Key and Explanation acts opposite to gravitational pull of earth. 1. Key: D  Together, the gravitational pull and the Explanation: Corals are living animals, and are centrifugal force are responsible for creating in a symbiotic relation with microscopic algae the two major tidal bulges on the earth. called Zooxanthellae. Hence all the three options are correct. There are two types of corals, hard coral and soft coral. Only hard coral build the reefs. Most of the world's coral reefs are found in tropical waters, Where the water is warm and clear. Coral reef are highly sensitive to temperature change and live within a relatively narrow temperature range (20 to 25 degree). Educational Objective: To have knowledge Coral reefs are one of the most productive about how tides are formed the factors that help coastal ecosystem in the world with very high in formation of tides and also about neap tide biological activity. Thy host reefs host far more and spring tide conditions. number of animal phyla than those hosted by 3. Key: D tropical rainforests. Explanation: Ocean acidification is the ongoing Some of the major reasons for coral bleaching decrease in the pH of the Earth’s oceans, caused are- Temperature change, Solar Irradiance, by the uptake of carbon dioxide (CO2) from the Sedimentation, Variation in salinity, Subareial atmosphere. exposure and Anthrogenic causes etc Effects Educational Objective: To have comprehensive  Increasing acidity depresses metabolic rates knowledge about the corals as they are in and immune responses in some organisms. critically endangered category in the IUCN list  Seawater absorbs CO2 to produce carbonic 2. Key: D acid, bicarbonate and carbonate ions. Explanation: The periodical rise and fall of the  The decrease in the amount of carbonate sea level, once or twice a day, mainly due to the ions available makes it more difficult for attraction of the sun and the moon, is called a marine calcifying organisms, such as coral tide. (calcareous corals) and some plankton Factors (calcareous plankton), to form biogenic

www.laexias.com Page No. 21 https://elearn.laex.in Geography UPSC Previous Year Questions

calcium carbonate. Hence option 1,2,3 are Explanation: Ocean current is a continuous, correct directed movement of sea water generated by a  Ocean acidification affects the formation of number of forces acting upon the water. Only dimethyl sulfide (DMS) produced by surface water is displaced and hence, statement phytoplankton. 1 is correct. This DMS enters the atmosphere to form Ocean currents deflect back the uneven solar aerosols which in turn help in the cloud radiation from the sun and hence, help in heat formation. Hence it is also adversely affected. balance. Hence, statement 2 is correct Hence option 4 is also correct The winds pull surface water with them, Educational Objective: To have knowledge creating currents. Hence, statement 3 is about various environmental issues like ocean correct. acidification, ozone depletion etc and also about The factors that can modify the currents are their effects on living beings. direction and shape of coastlines. Hence, statement 4 is also correct 4. Key: D Educational Objective: To have complete Explanation: Mangroves reduce the height knowledge about the ocean currents, factors and energy of wind and swell waves passing responsible for its origin and their influence on through them, reducing their ability to the Earth. sediments and to cause damage to structures such as dikes and sea walls. This is because of 6. Key: C their strong and intertwined roots. The roots of Explanation: There is a seven day mangroves are called as Stilt or prop roots. interval between the spring tides and neap Hence, option D is correct tides. Sun and moon are at right angles to Option A is incorrect for the reason that each other and the forces of the sun and mangroves don’t separate any zone. They are the moon tend to counteract one another. Hence, transition between aquatic and land ecosystem high tide is lower and low tide is higher than Option B says that mangroves provide food and usual. Hence, assertion is correct medicine after natural disaster which is Spring tides occur twice a month, one on full incorrect as mangroves always provide them. moon period and another during new moon Not only after disaster period. This makes the reason false. Mangroves are not tall trees. Hence, Option C is incorrect Educational Objective: To have thorough knowledge about the mangroves. Also know its utilities to mankind.

5. Key: D

www.laexias.com Page No. 22 https://elearn.laex.in Geography UPSC Previous Year Questions

Educational Objective: To learn about the tides, their formation, factors responsible for their formation and the effects.

7. Key: C Educational Objective: To have thorough Explanation: World’s best fishing grounds are knowledge about the various straits in news. found where the warm and cold oceanic currents meet. This is for the reason that the region Also have complete knowledge about the South provides optimal conditions for the growth of East Asian region planktons. Fishes that rely on these planktons 9. Key: A breed in the region. Hence, option C is correct Explanation: The builders of coral reefs are tiny Warm and cold atmospheric currents have marine animals called Polyps. Only hard coral influence on the landform and has nothing to do with oceans. Hence, option A is wrong. build the reefs. These corals by them self are not If rivers drain out large amount of freshwater, a colourful, instead they are white these corals the water body reduces it’s salinity and doesn’t are in a symbiotic relation with microscopic influence the breeding of fishes. Hence, option algae called Zooxanthellae, by which these B is incorrect corals gets coloration. The undulating continental shelf is not suitable The Great Barrier Reef of Australia is the world's for planktons as they rise and fall. Hence, not a good place for fish breeding. Hence, option D is largest coral reef system. And it is composed of incorrect. over 2,900 individual reefs. Educational Objective: To study about the The Great Barrier Reef of Australia is also the ocean currents, their meeting points and their world's biggest single structure made by living effects on the adjoining areas organisms and was selected as a World Heritage 8. Key: B Site in 1981. Explanation: Malacca Strait, between Increase in Golbal warming has resulted in Indonesia and Malaysia reduces the travel time increase in ocean water temperature which in to from India. See figure. turn increased the coral bleaching. The coral Kra canal is proposed to be built near isthmus bleaching in northern parts of the Great Barrier of kra. It is near Thailand. Once this canal becomes operative, it connects Reef of Australia is more widespread. with Andaman Sea Educational Objective: To know about the coral reefs, their locations, conditions required for the growth coral reefs

www.laexias.com Page No. 23 https://elearn.laex.in Geography UPSC Previous Year Questions

10. Key: C South Atlantic Ocean to the east of the South Explanation: Deep-sea trench, also called Sandwich Islands. oceanic trench are the deepest parts of the Educational Objective: To know about the ocean floor, relatively narrow in width, but very various oceanic features like trenches, ridges etc long. and their locations.

Oceanic trenches are a distinctive feature of 12. Key: D convergent plate boundaries, where two tectonic Explanation: Guinea Current: The Guinea plate move towards each other and one tectonic Current is a warm water slow moving current plate subducts under another (Except that flows to the east along the Guinea coast of continental-continental plate convergent). . The Guinea current similar to the Diamantina Trench is situated in south-eastern Equatorial Counter Current in the Indian and Indian Ocean seafloor and it’s a part of The Pacific Oceans. Diamantina Fracture Zone. Benguela Current: The Benguela Current is Educational Objective: To know about the cold ocean current flowing from roughly Cape trenches and the ocean that they are in. Point in the south to the Angola-Benguela. 11. Key: B Falkland Current: Falkland Current is a cold Explanation: Aleutian Trench: The Aleutian water current and it is a branch of the Antarctic Trench also known as Aleutian Trough is an Circumpolar Current in the Southern deep oceanic trench in the North Pacific Ocean, Hemisphere. along a convergent plate boundary which runs It flows northward in the South Atlantic Ocean along the southern coastline of and the along the east coast of Argentina where it meet . the southward-flowing Brazil Current and get Kermadec Trench: The Kermadec trench deflected eastward. located in South Pacific Ocean,, it runs parallel Canary Current: The Canary Current is a wind- with and to the east of the Kermadec Ridge and driven, south flowing cold water current along island arc. the North West Africa and it is part of the North It is one of the deepest oceanic trenche of the Atlantic Gyre. Earth, reaching a depth of 10,047 metres. It is Agulhas Current: The Agulhas Current is the formed by the subduction of the Pacific Plate warm water carrying western boundary current under the Indo-Australian Plate of the southwest Indian Ocean. Along the Sunda Trench: The Sunda Trench, also known southeast coast of Mozambique and the coast of as the Java Trench is an oceanic trench located South Africa before turning eastward to join the in the Indian Ocean near Sumatra flow towards Australia. South Sandwich Trench: The South Sandwich Educational Objective: To know about all the Trench is a deep oceanic trench located in the ocean currents and their locations/ place of origin on map.

www.laexias.com Page No. 24 https://elearn.laex.in Geography UPSC Previous Year Questions

(d) Occurrence of the belt of calm near the 4. Climatology equator 1. Why are dewdrops not formed on a 5. The seasonal reversal of winds is the cloudy night? typical characteristic of (a) the Earth's surface. (a) Equatorial climate (b) Clouds reflect back the Earth's radiation. (b) Mediterranean climate (c) The Earth's surface would have low (c) Monsoon climate temperature on cloudy nights. (d) All of the above (d) Clouds deflect the blowing wind to ground level. 6. Variations in the length of daytime and night-time from season to season are due 2. Each day is more or less the same, the to morning is clear and bright with a sea breeze; as the Sun climbs high in the sky, (a) The earth's rotation on its axis heat mounts up, dark clouds form, then (b) The earth's revolution round the sun in an rain comes with thunder and lightning. elliptical manner But rain is soon over." Which of the (c) Latitudinal position of the place following regions is described in the (d) Revolution of the earth on a tilted axis above passage? (a) Savannah 7. The annual range of temperature in the (b) Equatorial interior of the continents is high as (c) Monsoon compared to coastal areas. What is/are (d) Mediterranean the reason/reasons? 1) Thermal difference between land and water 3. In the South Atlantic and South-Eastern 2) Variation in altitude between continents and Pacific regions in tropical latitudes, oceans cyclone does not originate. What is the 3) Presence of strong winds in the interior reason? 4) Heavy rains in the interior as compared to (a) Sea surface temperatures are low coasts (b) Inter-Tropical Convergence Zone seldom Select the correct answer using the codes occurs given below. (c) Coriolis force is too weak (a) 1 only (d) Absence of land in those regions (b) 1 and 2 only 4. What explains the eastward flow of the (c) 2 and 3 only equatorial counter-current? (d) 1,2, 3 and 4 (a) The Earth's rotation on its axis 8. During the thunderstorm, the thunder in (b) Convergence of the two equatorial currents the skies is produced by the (c) Difference in salinity of water 1) Meeting of cumulonimbus clouds in the sky

www.laexias.com Page No. 25 https://elearn.laex.in Geography UPSC Previous Year Questions

2) Lightning that separates the nimbus clouds 11. Which one of the following is the 3) Violent upward movement of air and water characteristic climate of the Tropical particles Savannah Region? Select the correct answer using the codes (a) Rainfall throughout the year given below. (b) Rainfall in winter, only (a) 1 only (c) An extremely short dry season (d) A definite dry and wet season (b) 2 and 3 12. La Nina is suspected to have caused (c) 1 and 3 recent floods in Australia. How is La Nina (d) None of the above produces the thunder different from EI Nino? 9. Consider the following factors: 1) La Nina is characterised by unusually cold 1) Rotation of the Earth ocean temperature in equatorial Indian 2) Air pressure and wind Ocean whereas EI Nino is characterised by 3) Density of ocean water unusually warm ocean temperature in the equatorial Pacific Ocean. 4) Revolution of the Earth 2) EI Nino has adverse effect on south-west Which of the above factors influence the monsoon of India, but La Nina has no effect ocean current on monsoon climate. (a) 1 and 2 only Which of the statements given above is/are (b) 1,2 and 3 correct? (c) 1 and 4 (a) 1 only (d) 2,3 and 4 (b) 2 only (c) Both 1 and 2 10. Normally, the temperature decreases with (d) Neither 1 nor 2 the increase in height from the Earth's 13. Westerlies in southern hemisphere are surface, because stronger persistent than in northern 1) The atmosphere can be heated upwards hemisphere. Why? only from the Earth’s surface. 1) Southern hemisphere has less landmass as 2) There is more moisture in the upper compared to northern hemisphere. atmosphere 2) Coriolis force is higher in southern 3) The air is less dense in the upper hemisphere as compared to northern atmosphere hemisphere Select the correct answer using the codes Which of the statements given above is/are given below correct? (a) 1 only (b) 2 and 3 only (a) 1 only (c) 1 and 3 only (b) 2 only (d) 1,2 and 3 (c) Both 1 and 2 (d) Neither 1 nor 2

www.laexias.com Page No. 26 https://elearn.laex.in Geography UPSC Previous Year Questions

14. A layer in the Earth's atmosphere called (c) Both 1 and 2 ionosphere facilitates radio (d) Neither 1 nor 2 communication Why? 17. A new type of El Nino called El Nino 1) The presence of ozone causes the reflection Modoki appeared in the news. In this of radio waves to Earth, context, consider the following 2) Radio waves have a very long wavelength. statements: Which of the statements given above is/ are 1) Normal El Nino forms in the Central Pacific correct? ocean whereas El Nino Modoki forms in (a) 1 only Eastern Pacific ocean (b) 2 only (c) Both 1 and 2 2) Normal EI Nino results in diminished (d) Neither 1 nor 2 hurricanes in the Atlantic Ocean but El Nino Modoki results in a greater number of 15. What could be the main reason /reasons hurricanes with greater frequency for the formation of African and Eurasian Which of the statements given above is/are desert belt? correct? 1) It is located in the sub-tropical high (a) I only pressure cells. (b) 2 only 2) It is under the influence of warm ocean (c) Both 1 and 2 currents. (d) Neither 1 nor 2 Which of the statements given above is / are correct in this context? 18. A geographic area with an altitude of 400 (a) 1 only meters has following characteristics

(b) 2 only Month J F M A M J J A S O N D

(c) Both 1 and 2 Average Maximum 31 31 31 31 30 30 29 28 29 29 30 31 (d) Neither 1 nor 2 Temp.C 16. The jet aircrafts fly very easily and Average Minimum 21 21 21 21 21 21 20 20 20 20 20 20 smoothly in the lower stratosphere. What Temp.C Rainfall could be the appropriate explanation? 51 85 188 158 139 121 134 168 185 221 198 86 (mm) 1) There are no clouds or water vapour in the If this geographic area were to have a natural lower stratosphere. forest, which one of the following would it 2) There are no vertical winds in the lower most likely be? stratosphere. (a) Moist temperate Coniferous forest Which of the statements given above is/ are (b) Montane sub-tropical forest correct in this context? (c) Temperate forest (a) 1 only (d) Tropical rain forest (b) 2 only

www.laexias.com Page No. 27 https://elearn.laex.in Geography UPSC Previous Year Questions

19. A geographic region has the following Which of the statements given above is/are distinct characteristics: correct? 1) Warm and dry climate (a) 1 only 2) Mild and wet winter (b) 2 only 3) Evergreen oak trees (c) Both 1 and 2 The above features are the distinct (d) Neither 1 nor 2 characteristics of which one of the following 23. Consider the following statements: regions? 1) The albedo of an object determines its visual (a) Mediterranean brightness when viewed with reflected light. (b) Eastern 2) The albedo of Mercury is much greater than (c) the albedo of the Earth. (d) Atlantic coast of Which of the statements given above is/are 20. What causes wind to deflect toward left correct in the Southern hemisphere? (a) 1 only (a) Temperature (b) 2 only (b) Magnetic field (c) Both I and 2 (c) Rotation of the earth (d) Neither 1 nor 2 (d) Pressure 24. Consider the following statements: 21. Consider the following which can be 1) Either of the two belts over the oceans at found in the ambient atmosphere: about 30° N to 35° N and S Latitudes is 1) Soot known as Horse Latitude 2) Sulphur hexafluoride 2) Horse latitudes are low pressure belts. 3) Water vapour Which of the statements given above is/are Which of the above contribute to the correct? warming up of the atmosphere? (a) 1 only (a) 1 and 2 only (b) 2 only (b) 3 only (c) Both 1 and 2 (c) 2 and 3 only (d) Neither 1 nor 2

(d) 1 , 2 and 3 25. Consider the following statements: 22. Consider the following statements: 1) The annual range of temperature is greater 1) In the world, the tropical occur in the Pacific Ocean than that in the along the western margins of continents Atlantic Ocean within the trade wind belt. 2) The annual range of temperature is greater 2) In India, the East Himalayan region gets in the Northern Hemisphere than that in high rainfall from north-east winds. the Southern

www.laexias.com Page No. 28 https://elearn.laex.in Geography UPSC Previous Year Questions

Which of the statements given above is/are (c) A is true but R is false correct? (d) A is false but R is true (a) 1 only 29. Assertion: Areas lying within five to eight (b) 2 only degrees latitude on either side of the (c) Both 1 and 2 equator receive rainfall throughout the year. (d) Neither 1 nor 2 Reason: High temperature and high 26. Assertion: Wind patterns are clockwise in humidity cause convectional rain to fall the northern hemisphere and anti-clockwise mostly in the afternoons near the equator in the southern hemisphere. (a) Both A are R are true R is the correct Reasons: The directions of wind patterns in explanation of A the northern and the southern hemisphere (b) Both A and R are true but R is not a correct are governed by the Coriolis effect. explanation of A (a) Both A are R are true R is the correct (c) A is true but R is false explanation of A (d) A is false but R is true (b) Both A and R are true but R is not a correct 30. Consider the following statements: explanation of A 1) In equatorial regions, the year is divided (c) A is true but R is false into four main seasons (d) A is false but R is true 2) In Mediterranean region, summer receives 27. Which one of the following statements is more rain correct? 3) In China type climate, rainfall occurs (a) Cirrus clouds are composed of ice crystals throughout the year (b) Cirrus clouds exhibit a flat base and have 4) Tropical highlands exhibit vertical zonation the appearance of rising domes of different climates (c) Cumulus clouds are white and thin, and Which of these statements are correct? form delicate patches and give a fibrous and (a) 1, 2, 3 and 4 feathery appearance (b) 1, 2 and 3 (d) Cumulus clouds are classified as high (c) 1, 2 and 4 clouds (d) 3 and 4 28. Assertion: The thickness of the atmosphere 31. Assertion: 60°-65° latitudes in both the is maximum over the Equator. hemispheres have a low pressure belt Reason: High insolation and strong instead of high pressure. convection currents occur over the Equator. Reason: The low pressure areas are (a) Both A are R are true R is the correct permanent over oceans rather than on land. explanation of A (a) Both A are R are true R is the correct (b) Both A and R are true but R is not a correct explanation of A explanation of A

www.laexias.com Page No. 29 https://elearn.laex.in Geography UPSC Previous Year Questions

(b) Both A and R are true but R is not a correct (a) 1 and 2 explanation of A (b) 2, 3 and 4 (c) A is true but R is false (c) 1,2 and 4 (d) A is false but R is true (d) 1,2, 3 and 4

32. Assertion: The surface winds spiral inwards 35. For short-term climatic predictions, upon the centre of the cyclone which one of the following events, detected Reason: Air descends in the centre of the in the last decade, is associated with cyclone occasional weak monsoon rains in the Indian (a) Both A are R are true R is the correct sub-? explanation of A (a) La Nina (b) Both A and R are true but R is not a correct (b) Movement of Jet Streams explanation of A (c) El Nino and Southern Oscillations (c) A is true but R is false (d) Greenhouse effect on global level (d) A is false but R is true 36. Match List-I with List-II and select the 33. Which one of the areas marked as A, B, C correct answer using the codes given and D in the given figure of the cyclone, below the lists. witnesses heavy torrential short duration List-I (Local Wind) List-II (Region) rainfall accompanied by thunderstorms? A. Fohn 1. Argentina B. Samun 2. C. Santa Ana 3. California D. Zonda 4. Alps Codes: (a) A-2; B-4; C-1; D-3 (a) A (b) A-4; B-2; C-3; D-1 (b) B (c) A-2; B-4; C-3; D-1 (c) C (d) A-4; B-2; C-1; D-3 (d) D 37. Cloudy nights are warmer compared to 34. Consider the following climatic and clear cloudless nights, because clouds geographical phenomena (a) Prevent cold waves from the sky from 1) Condensation descending 2) High temperature and humidity (b) Reflect back the heat given off by earth 3) Orography (c) Produce heat and radiate it towards earth 4) Vertical wind (d) Absorb heat from the atmosphere and send Thunder Cloud development is due to which it towards earth of these phenomena?

www.laexias.com Page No. 30 https://elearn.laex.in Geography UPSC Previous Year Questions

38. Which one of the following weather (a) China type conditions is indicated by a sudden fall (b) Equatorial type in barometer reading? (c) Hot desert type (a) Stormy weather (d) Monsoon type (b) Calm weather 41. Consider the following temperature and (c) Cold and dry weather rainfall data: (d) Hot and sunny weather Rainfall Month Temp C 39. The temperature and rainfall of a cm meteorological station are given below: Jan 6.7 14.0 Average temperature: 12.8° C Feb 6.7 13.2 Average Rainfall: 54.9 cm per annum Mar 7.2 11.4 Identify the region having the above climatic Apr 8.9 9.4 pattern from amongst the following May 11.1 8.1 (a) Mediterranean region Jun 13.9 8.1 (b) Monsoon region Jul 15.0 9.6 (c) Steppe region Aug 15.0 12.2 (d) N.W. European region Sep 13.9 10.4 40. Consider the following climatic Oct 11.1 14.4 conditions (Northern Hemisphere): Nov 8.9 14.0 Temperature Rainfall Dec 7.8 16.8 Month C Cm The climate to which this data pertains is. Jan 3.9 4.7 (a) St Lawrence type Feb 4.4 5.7 (b) China type Mar 8.3 8.2 (c) West European type Apr 14.4 9.2 (d) Mediterranean type

May 20.0 9.2 42. "Climate is extreme, rainfall is scanty Jun 23.3 17.7 and the people used to be nomadic Jul 27.8 14.5 herders." Aug 27.8 14.0 The above statements best describes which Sep 22.3 12.7 of the following regions? Oct 18.3 7.0 (a) African Savannah Nov 12.2 5.0 (b) Central Asian Steppe Dec 6.7 3.5 (c) North American Prairie (d) Siberian Tundra These are most likely to be found in the natural regions of

www.laexias.com Page No. 31 https://elearn.laex.in Geography UPSC Previous Year Questions

43. Consider the following statements about 47. A ship sailing from the eastern extremity the 'Roaring Forties': of the Aleutian Islands to Dutch harbour 1) They blow uninterrupted in the Northern crosses 1800 meridian at 23:30 hrs on and Southern Hemispheres January 1, 1999, What time and date will 2) They blow with great strength and be recorded by the captain of the ship in constancy his diary after one-hour journey from the 3) Their direction is generally from North West point of crossing of the meridian? to East in the Southern Hemisphere (a) January 1,00:30 hrs 4) Overcast skies, rain and raw weather are (b) January 2,00:30 hrs generally associated with them (c) January 3,00:30 hrs Which of these statements are correct? (d) January 4,00:30 hrs (a) 1,2 and 3

(b) 2,3 and 4 48. Which one of the following straits is (c) 1,3 and 4 nearest to the International Date Line? (d) 1,2 and 4 a. Malacca Strait 44. Along which one of the following b. meridians did India experience the first c. Strait of Florida light of the sunrise of the new d. Strait of Gibraltar millennium? 49. When there is noon at I.S.T meridian (a) 200 300W people at another place of the earth are (b) 820 300E taking their 6 O' clock morning tea. The (c) 920 300W longitude of the place is: (d) 920 300E a. 17° 30'E 45. Most of the explosions in mines occur b. 7°30’ W due to the mixing of c. 172°39’E (a) Hydrogen with oxygen d. 90°W (b) Oxygen with acetylene 50. What are the possible limitations of (c) Methane with air India in mitigating the global warming at (d) Carbon dioxide with ethane present and in the immediate future? 46. Which one of the following reflects back 1) Appropriate alternate technologies are not more sunlight as compared to other three? sufficiently available (a) Sand desert 2) India cannot invest huge funds in research (b) Paddy crop land and development (c) Land covered with fresh snow 3) Many developed countries have already set (d) Prairie land up their polluting industries in India

www.laexias.com Page No. 32 https://elearn.laex.in Geography UPSC Previous Year Questions

Which of the statements given above is/ are 2. Key: B correct? Explanation: The given description is about the a. 1 and 2 only. equatorial region and Hence option B is b. 2 only correct. In this region as the coriolis force is c. 1 and 3 only nil or near zero, the atmospheric oscillations are d. 1, 2 and 3 low and hence great uniformity in Temperature is seen. The rainfall that occurs is because of 4. Key and Explanation convection and this related to position of ITCZ.

1. Key: B Option A is incorrect as Savannah region is Explanation: Dew is formed when objects region with alternate dry and wet seasons. Rain radiate heat thoroughly, so that the moist air occurs only in summer. It has considerably less coming into contact with them may be annual rainfall. Days are hot and nights are cold. sufficiently cooled and the water vapour is Option C is incorrect for the reason that the condensed into water. Clouds act as a blanket monsoon climate depends on the position of the for the surface of the earth. They check the ITCZ along with other factors like the radiation so that the it does not radiate the heat Temperature. Annual Temperature is very low thoroughly. Hence, dew is not formed on cloudy and annual rainfall is high. The rainfall is not nights. Hence, option B is correct. uniform throughout the year. It occurs only in a particular season. Option D is incorrect as Mediterranean climate receives rainfall only in winter which is unique to this. The climate is favourable for viticulture. Educational objective: To know about all the climatic conditions along with their features, Option A is not correct since it is given that flora and fauna. the cloud absorbs radiation from Earth’s surface. When clouds absorb radiation, the condition 3. Key: B favours the dew formation. Explanation: The south Atlantic and South- Option C and D is incorrect due to the fact Eastern Pacific region in Tropical latitudesgets that on cloudy night, the temperature is a little few tropical cyclones are that the tropospheric higher than the clear nights, as the radiations vertical wind shear is strong and there is from the earth is reflected back. Hence, the typically no ITCZ over the ocean Without ITCZ chances of wind formation is low. to provide synoptic vorticity and convergence Educational Objective: To learn about the having strong shear, it is difficult to have genesis clouds, their formation and the weather of tropical cyclones. Hence option B is correct. conditions favourable for the formation of dew, frost etc.,

www.laexias.com Page No. 33 https://elearn.laex.in Geography UPSC Previous Year Questions

Though these regions have low surface know about their location, type(warm or cold) temperature, the main reason is because of the and the reason for their formation.

high wind shear. Hence option A is incorrect. 5. Key: C Explanation: Monsoon climate is the only climate that has seasonal reversal of winds. Hence option C is correct.

Option C is in correct as Coriolis force is not weak in the region Equatorial climate has no seasonal reversal of Option D is not correct for the reason that the wind. Hence option A and D are incorrect region has sufficient landmass Mediterranean climate doesn’t experience Educational objective: To study about the seasonal reversal of winds and this makes cyclones, their formation, favourable factors for option B incorrect. the formation etc., Educational Objective: To learn about the 4. Key: B world climates and their features. Explanation: There are two equatorial currents flowing from east towards west. In-between 6. Key: D these there flows the counter equatorial current Explanation: Rotation of Earth results in day from West towards east. The piling up of waters and night and not the length of day and night. in the area near Brazil due to the convergence Hence option A in incorrect. Revolution of the earth around the sun results of the two equatorial currents creates the in seasons and not day and night. This makes counter current. Hence option B is correct. Although the rotation of Earth has a role in the option B as incorrect. flow of counter equatorial current, the main reason for it is the convergence of 2 equatorial currents.Hence option A is incorrect. Option C is not correct because the salinity has no role to play, as the counter current is formed as a result of piling up of water. Option D is incorrect Educational Objective: To have knowledge about the currents all over the earth. Also to

www.laexias.com Page No. 34 https://elearn.laex.in Geography UPSC Previous Year Questions

Option C is incorrect as latitudinal position of Educational Objective: To have knowledge place has role in the intensity of the sunlight about the effects of various factors like received. And has no role in the variation of temperature, pressure, rainfall etc., in different length of day and night from season to season. regions. The correct answer is option D as the tilt of the 8. Key: D earth on its own axis causes the variation in the Explanation: Thunder is the sound caused by a intensity of sun's rays falling at a place over the lightning discharge. Lightning heats the air in its span of a year leading to variation in day and path and causes a large over-pressure of the air night. within its channel. The channel expands To learn about the earth Educational objective: supersonically into the surrounding air as a movements and it’s effects shock wave and creates an acoustic signal that is heard as thunder. Hence option D is correct. 7. Key: A Educational objective: To have knowledge Explanation: The temperature in the interiors of about the events occurring in the different Continents is high when compared to the coastal spheres of Earth like atmosphere, lithosphere areas. This is for the reason that Water etc., Maintains Temperature. Interiors of the 9. Key: B continent being away from the sea are out of Explanation: Ocean currents are like river flow reach of its influence. On the other hand, the in oceans. They have definite path and direction. coastal areas being closer to water bodies are Factors influencing the ocean currents are; much influenced by them and the temperature  Winds – winds on the surface pushes the fluctuations are less. Hence statement A is water to move. And these winds are correct. permanent winds that blow from one Altitude has little role to play in annual range of pressure belt to the other. temperature in interiors and coastal areas.  Temperature - The differential heating of Oceans in the higher latitude are warmer than the Sun at the equator and the poles causes the continents. The interiors too experience a difference in the temperature of ocean much lower temperature leading to low annual water. Thereby giving rise to ocean currents. range of temperature. Hence statement 2  Earth's rotation - According to Ferrel’s incorrect. law- Coriolis forces deflect winds and freely By elimination we can arrive at the answer. moving objects to the right in the northern Statement 3 is incorrect because winds in the hemisphere and to the left in the southern interior reduces the temperature which in turn hemisphere. Therefore, the movement of leads to reduction in annual range of ocean currents in the northern hemisphere temperature is in the clockwise and in the southern Heavy rain also reduces the temperature. Hence statement 4 is also incorrect

www.laexias.com Page No. 35 https://elearn.laex.in Geography UPSC Previous Year Questions

hemisphere it is in the anti-clockwise Educational Objective: To study the heat direction. budget and its effect. Also learn about the  Other factors are salinity, density, pressure temperature variations.

gradient and melting of ice, precipitation. 11. Key: D From the above information it is clear that Explanation: These places are located near the revolution of the earth does not have my equator, and they lie between the Southern and effect on ocean currents. the Northern Tropics. Climate has alternate wet Hence B is correct. and dry seasons similar to monsoon climate but Educational Objective: To learn about the has considerably less annual rainfall. Also, ocean currents and related details. there is no distinct rainy season like in monsoon climate. 10. Key: C Hence option D is correct Explanation: The temperature of the earth Educational Objective: To have thorough depends on the amount of insolation received knowledge of all the climatic conditions, their and the albedo of the earth. This is called the features, flora, fauna and related information. heat budget. 12. Key: D The atmosphere is heated by infrared radiation Explanation: El Nino and LaNina are opposite of emitted by earth not by the ultraviolet radiation each other. Warm current in El Nino, cold of sun. Hence statement 1 is correct. As we current in LaNina but both happen in Pacific. move upwards the temperature reduces at a rate Hence statement 1 is incorrect of 6degree per km. This is known as normal As they are opposite if ElNino affects monsoons lapse rate. adversely, LaNina would affect monsoon positively. Both depends on Indian Ocean dipole. Hence statement 2 is incorrect. Therefore the answer is D Educational Objective: To learn about monsoons, factors affecting monsoon, El Nino, La Nina, Indian Ocean dipole and their effects.

13. Key: A Explanation: The southern hemisphere has less Statement 2 is incorrect as the moisture landmass and this helps in stronger westerlies content in more in lower atmosphere because of as there is less friction. Hence statement 1 is the warmer temperature. correct Statement 3 is correct as the air is less dense, Statement 2 is incorrect as Coriolis force it can hold less heat and hence temperature is depends on the latitude and not on low. hemisphere.

www.laexias.com Page No. 36 https://elearn.laex.in Geography UPSC Previous Year Questions

Therefore the answer is option A 15. Key: A Educational objective: To have knowledge a Explanation: Both the regions are under the out the wind patterns and factors affecting them. influence of cold current and not warm Also learn about the Coriolis force and its effects currents. Hence statement 2 is incorrect. in different latitudes The factors facilitating desert formation are cold currents, temperature, precipitation among 14. Key: B others. Sub tropical region has sufficient high Explanation: Earth's atmosphere has 4 layers temperature. Desserts are formed in regions of and all of them has unique properties helping permanent high pressure areas. the organisms on earth. Hence statement 1 is correct. Ionosphere is a sub layer in the thermosphere Therefore answer is option A. that contains charged particles due to which To learn about the electromagnetic radio waves gets reflected and Educational Objective: various deserts in different latitudes. Also have communication becomes possible. It is not due knowledge about the factors influencing their to ozone as it present in Stratosphere. Hence formation. statement 1 is wrong Radio waves lie in the extreme of the spectrum 16. Key: C as shown in the figure. Hence it has long Explanation: The jet aircrafts need no wavelength. Statement 2 is correct for this disturbance in their flying that could create reason. turbulence. Lower stratosphere has less air Therefore answer is option B turbulence and hence suitable for flying of jet aircrafts. Hence statement 1 is correct. The temperature in lower stratosphere has a temperature similar to troposphere. Though the temperature increases in stratosphere, it is largely constant. This makes the little convection or the vertical wind. Hence statement 2 is also correct. Therefore answer is option C Educational Objective: To have knowledge about the layers of atmosphere, their characteristics and functions. Educational Objective: To learn about the 17. Key: B different layers of atmosphere and their Explanation: El Nino is formed in South- characteristics. Also learn about the Eastern Pacific Ocean, next to South America. electromagnetic waves that are present or has Here the cold Humboldt current is replaced by effect on these layers. the warm current sourced from Australia.

www.laexias.com Page No. 37 https://elearn.laex.in Geography UPSC Previous Year Questions

Whereas, El Nino Modoki is formed in Central Eastern China climate is characterised by warm Pacific Ocean. Hence statement 1 is incorrect. moist summer and a cool, dry winter. Hence Statement 2 is correct as the El Nino Modoki option B is incorrect. formed in Central Pacific causes greater number Central Asia climate is dry and continental, of hurricanes. with hot summers and cool to cold winters, Hence the answer is option B with occasional snowfall. Hence option C is Educational Objective: To study about the wrong different phenomena like El Nino, La Nina, El Atlantic coast of North America - mild Nino Modoki and their effects. and moderate climate because ocean water has a balancing effect on temperatures. Hence 18. Key: D option D is incorrect Explanation: From the table, it can be observed Educational Objective: To have knowledge that the average temperature is above 20 degree about the world Climates and their features. Celsius and the average rainfall is around 150cm. This is the characteristic feature of rain forest. Hence option D is correct. 20. Key: C Moist Temperate Coniferous forest will have Explanation: Earth's rotation produces a force average temperature of -40 to 20 degree Celsius. on all bodies moving relative to the Earth. Due Hence option A is incorrect to Earth's approximately spherical shape, this Montane sub tropical forest – average force is greatest at the poles and least at the temperature of 17-21 degree Celsius and average Equator. The force, called the "Coriolis effect," rainfall of 125cm.Hence option B is incorrect causes the direction of winds and ocean Temperate forest – average temperature is 10 currents to be deflected. In the Northern degree Celsius and average rainfall is 75-150cm. Hemisphere, wind and currents are deflected Hence option C is incorrect. toward the right, in the Southern Hemisphere Educational Objective: To have knowledge they are deflected to the left.Hence option C is about the different climatic conditions and their correct. features.

19. Key: A Explanation: From statement 2 it is clear that the given climate is Mediterranean as wet winters are characteristic feature of this climate. Evergreen oak trees are found here. This climate is found on the western margins of the countries in sub tropical region. Hence the answer is option A.

www.laexias.com Page No. 38 https://elearn.laex.in Geography UPSC Previous Year Questions

Educational Objective: To have knowledge become devoid of moisture, deserts are formed. about the wind systems of the world. Hence statement 1 is correct. Eastern Himalayas receive rain from north east 21. Key: D monsoon predominantly and not North east Explanation: Soot is a mass of winds. Hence statement 2 is incorrect. impure carbon particles resulting from the incomplete combustion of hydrocarbons. It Therefore the answer is option A increases air and leads to global Educational Objective: To have complete knowledge about the monsoon pattern. Also the warming. It causes cancer. pattern of all types of winds and their effects are Sulphur hexafluoride is an inorganic, colorless, to be studied. odorless, non-flammable, non-toxic but extremely potent greenhouse gas, and an excellent electrical insulator. 23. Key: A Water vapor is transparent, like most Explanation: Albedo is the fraction of the constituents of the atmosphere. It is less dense incident sunlight that is reflected. When an than most of the other constituents of air and object reflects most of the light that hits it triggers convection currents that can to and looks bright then it has high albedo. Hence, clouds. Water vapor is the most statement 1 is correct. potent greenhouse gas owing to the presence of The lower an object's albedo is, the better it is at the hydroxyl bond which strongly absorbs in absorbing light, which means the higher the infra-red region of the light spectrum. the albedo, the less sunlight actually gets Hence the answer is option D. absorbed. Mercury being the hottest planet in Educational Objective: To learn about the the solar system has lower albedo than other types of pollution, Pollutants, their sources and planets and turns out to be similar to the Moon effects. at 0.119. This means that earth has higher albedo than Mercury. Hence statement 2 is 22. Key: A incorrect. Explanation: Most of the deserts are on the To study about the western sides of the continents in the subtropics Educational Objective: solar system, planets and the albedo of planets because of the prevailing easterly winds of the with respect to earth. tropics, which are the trade winds. The trade winds prevail in the lower portion of the earth’s 24. Key: A atmosphere, in the lower section of the Explanation: The pressure belts of the earth are troposphere. as shown in the figure. The latitudes from 30° to These winds become dry when they reach the 35° in both the hemispheres are called horse western sides of the continents, and hence latitudes. don’t bring any rain with them. As these regions These latitudes have diverging winds are the result of an area of high pressure, which is

www.laexias.com Page No. 39 https://elearn.laex.in Geography UPSC Previous Year Questions

characterized by calm winds, sunny skies, and Hemisphere the deflection of the air is to the little or no precipitation. Hence statement 1 left of the direction of air movement. This is is correct. called the Coriolis effect.

Therefore, the winds in the northern hemisphere as it seen from the Statement 2 is incorrect has clockwise direction whereas winds in the figure that it is a high pressure region. southern hemisphere is anti clockwise. Educational Objective: To learn about the Hence, the reason is the correct explanation wind systems and the pressure belts of Earth of the assertion. along with the prevailing weather conditions. Educational Objective: To have knowledge 25. Key: B about the forces generated due to earth's Explanation: Bigger the size of Ocean lesser is rotation and their effects. the annual range of temperature, as water 27. Key: A maintains temperature. That’s why, Atlantic Explanation: The characteristics of cirrus Ocean records higher annual range of clouds are; temperature than the Pacific Ocean because the Cirrus clouds are formed at high altitudes of size of Atlantic is smaller than that of Pacific. 8,000 – 12,000m. Hence, they are high Hence statement 1 is incorrect. altitudinal clouds. This makes Option D Statement 2 correct because northern incorrect. hemisphere has more landmass. As the land Cirrus clouds are detached thin clouds. They heats up fast and takes longer time to cool, the have a feathery appearance. They are always annual range of temperature will be greater. white in colour.This makes option C incorrect. Educational Objective: To have knowledge Cirrus clouds have doesn’t have proper flat base about the pattern of temperature around world and their appearance is feathery. Hence option over Oceans and Continents. A comparative B is incorrect. (cumulusclouds have flat base study is needed in the subject. with rising dome structure) 26. Key: A Explanation: Due to the earth’s rotation, the air mass curves relative to the earth’s surface. In the Northern Hemisphere this effect causes the air to deflect to the right of the direction of the air movement and in the Southern

www.laexias.com Page No. 40 https://elearn.laex.in Geography UPSC Previous Year Questions

Option A is correct as cirrus clouds are formed everyday. This rain is known as 4’o’ clock of ice crystals that originate from the freezing of shower. Therefore it can be said that the super cooled water droplets. reason explains the given statement. Educational Objective: To have thorough Educational Objective: To know about the knowledge of the equatorial region and the different types of clouds and their characteristics processes occurring in the region.

28. Key: A 30. Key: D Explanation: The atmosphere has different Explanation: The equatorial region layers troposphere being the inner most. experiences high sun position throughout the Troposphere has different thickness at the year and hence receives almost same amount of insolation. Season cycle is because of the tilt of Equator and the poles. This is because of the the axis and the direction and amount of strong rising convection near the equator which sunlight received. As the sun is always high, pushes the troposphere upwards. there is no season cycle in the region. Hence The strong convection over the equator is statement 1 is wrong. because of the high insolation the region Mediterranean region is well known for its receives. winter rain, a unique characteristic of it. The Hence reason given is the correct region is dry in summer. The reason for it is the explanation of the statement. direction of the westerlies. Statement 2 also stands incorrect. By elimination we can arrive at the answer as D

Educational Objective: To study thoroughly the layers of atmosphere and their characteristics. Also to study about the causes and effects and China type of climate - Rainfall is more than conduction, convection and radiation in the moderate, anything from 25 inches to 60 inches. atmosphere. Important feature is the fairly uniform 29. Key: A distribution of rainfall throughout the year. Explanation: Equatorial region lies between 5° There is rain every month, except in the to 10° on either side of the equator. This region interior of central China, where there is a has strong rising convection because of high distinct dry season. Rain comes either from insolation. This high insolation leads to hot and convectional sources or as orographic rain in humid climate. Hence, it receives rain almost

www.laexias.com Page No. 41 https://elearn.laex.in Geography UPSC Previous Year Questions

summer, or from depressions in prolonged The winds of the cyclone blow in a circular showers in winter. Statement 3 is correct. manner and the flow of air is convergent. Statement 4 is correct and can be explained Consequently, the air rises in the central part through the figure shown. and it results in heavy rain. Hence, reason Hence correct answer is option D statement is wrong Educational Objective: To study about all the Therefore, the answer is assertion is true but climatic conditions of the world along with their the reason is false unique features. Educational Objective: To study the origin,

31. Key: B formation and structure of cyclones thoroughly. Explanation: We know that the earth is divided 33. Key: B into different pressure zones. The latitudes from Explanation: Cyclones have strong winds. 60°-65° in both the hemispheres are known as Accompanying these strong winds are torrential Circum-Polar Low Pressure Belts. Low rains and a devastating phenomenon known as pressure is produced in this area because the the storm surge, an elevation of the sea surface surface air spreads outward from this zone due that can reach 6 metres (20 feet) above normal to rotation of the earth. Hence, assertion levels. stands right. Reason is not the correct explanation for it. The statement in reason is correct as the water maintains temperature, the pressure developed also lasts long which can be said to be permanent. On the other hand on the land the temperature keeps fluctuating and the pressure too varies. Therefore, both the statements are correct In the given figure point B lies between two independently. But, reason is not the correct cyclones A and D. hence point B would be explanation for the assertion. accompanied by heavy torrential short duration Educational Objective: To learn about the rain. various pressure belts and their characteristics Educational Objective: to have complete including their origin. knowledge about the cyclones including their structure, origin, formation and effects. 32. Key: C Explanation: The cyclones are formed when the 34. Key: D surface winds spiral inwards upon the centre Explanation: Thunder clouds are formed from creating a low pressure. Hence assertion is the cumulus clouds. As the cumulus clouds correct. grows it gives rise to Thunder clouds. The growing of cumulus cloud is facilitated by the

www.laexias.com Page No. 42 https://elearn.laex.in Geography UPSC Previous Year Questions

upward draft. Sometimes air is forced up the El Nino is also a condition where the Indian side of a mountain. Air is also forced upward at Ocean develops high pressure as the Pacific weather fronts, where warm and cool air masses Ocean becomes warmer and low pressure region. collide. Hence, orography is also important As wind blows from high pressure region to low factor for the air is to rise. pressure region, there is weak monsoon in India. The water droplets inside the cumulus cloud Hence, option C stands right. starts growing. The growing droplets that make La Nina is a phenomenon where the Indian up the cloud become heavy. Raindrops start to Ocean will have low pressure and hence the fall through the cloud when the rising air can no wind is sourced towards India leading to longer hold them up. Meanwhile, cool dry air stronger monsoon. Option A is incorrect. flows downward in the cloud, called a downdraft, Movement of Jetstream affects only pulling water downward as rain. This means northwestern part of India. And it has no effect condensation. Hence statement 1 is correct. on the mosoon rains. This checks the frost in Therefore, the answer is D winter. It facilitates winter precipitation in the Educational Objective: To have knowledge region. Hence, option B is incorrect about the clouds, their formation and lifecycle Greenhouse effect and global warming affects along with features. monsoons indirectly by causing excess rains, floods or drought. Hence, option D stands 35. Key: C incorrect Explanation: Southern Oscillation Educational Objective: To study about the (SO): Normally when the tropical eastern south different phenomena and their effect on Indian Pacific Ocean experiences high pressure, the sub continent. tropical eastern Indian Ocean experiences low pressure. But in certain years, there is a reversal 36. Key: B in the pressure conditions and the eastern Explanation: Foehn is a local wind in Pacific has lower pressure in comparison to near the Alps Mountains. This is favourable the eastern Indian Ocean. This periodic change wind. By this, option A and C is eliminated. in pressure conditions is known as the SO. Santa Ana is a descending wind in California. It is unfavourable for the region as it makes the region more dry.In the given options C is to be matched with 3. Hence, option B is correct. Samunis a local wind in Kurdistan. It so also called as simoom. This blows in Egypt and adjoining parts of west asia. It has no moisture and makes the place of destination drier.

www.laexias.com Page No. 43 https://elearn.laex.in Geography UPSC Previous Year Questions

Zonda is wind in Argentina. The name is the Calm weather will have a slowly rising regional term for Foehn. It descends from slopes movement. This makes Option B incorrect. of . It is a dry wind. Cold and dry weather means barometer Educational Objective: To study the different indicates a high pressure i.e., rise in the reading types of local winds, their source and and not fall. Therefore, option C is incorrect. destination regionand their effects on the region Option D is also incorrect for the fact that the hot and sunny weather will not have a low 37. Key: B pressure and the barometer doesn’t fall Explanation: Clouds being made up of water suddenly. vapouract as large blankets over the Earth's Educational Objective: To learn about the surface which keep the heat trapped inside devices that are used for measuring atmospheric and let very little out. This is what keeps the conditions. temperature warmer when compare to clear nights. They check the radiation so that the it 39. Key: A does not radiate the heat thoroughly. Hence, Explanation: AverageRai option B is correct as clouds reflect back the Region Average heat radiated by earth. Temp nfall A cloud does not produce any heat. They reflect Monsoon 27°C 125cm back the radiated heat of Earth. Hence, option Steppe 21°C-23°C 25-50cm North West C is incorrect. 10°C-21°C 50-75cm European Option D is incorrect because if clouds absorb Mediterranean 18° C 50-55cm heat, dew drops should form which lowers the From the table it is clear that option A is temperature making the nights cooler. But this correct does not happen. Educational Objective: To know about the Educational Objective: To have knowledge different climatic conditions and their features about the effects of radiation, convection and like temperature, rainfall, flora and fauna. conduction. Also learn how the formation of clouds influences the atmospheric conditions. 40. Key: A Explanation: Equatorial region has no season. 38. Key: A It is hot and humid. The annual range of Explanation: Barometer is a device that temperature is low as the rainfall is well measures atmospheric pressure. A slow downfall distributed throughout the region. The rainfall is in this indicates a low pressure region nearby. A around 200 cm per year. In the given table, small amount of rain may occur. rainfall and temperature both are low and hence, A sudden fall in it indicates a storm. This is option B is incorrect. because, as the storm rises, the temperature and hence pressure reduces suddenly. Hence option A is correct.

www.laexias.com Page No. 44 https://elearn.laex.in Geography UPSC Previous Year Questions

Option C is also wrong as hot desert will have less than 180C but greater than –30C. By this high range of temperature around 45°C. But, the information option D is incorrect. table shows the temperature around 25°C West European type of climate - This climate is Monsoon type of climate though has characterized by cool summer and mild winters. temperature range of about 25° - 27° C, the Average temperature during summer season rainfall is around 125cm per year. Hence, ranges between 15°C and 21°C. The regional option D is ruled out. distribution of precipitation is highly controlled China type of climate - average January by topographic factor. Hence, option C is temperature is below -10 °C (14 °F), and even correct. below -20 °C (-4 °F) in the northern part. Cold Educational Objective: To know about the records are around -40 °C (-40 °F). The different climatic conditions and their features temperature is around 25°C in July. Hence, like temperature, rainfall, flora and fauna. option A is correct. 42. Key: B Educational Objective: To know about the Explanation: African Savannah –extended different climatic conditions and their features periods of wet and dry climate. The savanna like temperature, rainfall, flora and fauna. climate has a temperature range of 18° – 30° C. 41. Key: C Rainfall is around 100cm. As rainfall is not Explanation: St. Lawrence type of climate - The scanty and the climate is not extreme, option Cool Temperate Eastern Margin climate is also A is not the answer. known as the Laurentian climate. It is the Central Asian steppe - The summers are hot and intermediate between the British and Siberian the winters are cold. Temperatures vary type of climates. The climate of this type has greatly in summer and winter making the cold, dry winters and warm, wet summers. climate extreme. The average rainfall may be Summers are as warm as the tropics (~25 °C). taken as about 45 cm which is scanty. Because Rainfall is around 125cm Option A is incorrect of such extreme climate people followed based on these details nomadic herding and are known as Kirgiz. China type - average January temperature is Hence, option B is correct. below -10 °C (14 °F), and even below -20 °C (-4 North American prairie – they are also temperate °F) in the northern part. Cold records are around grassland like the steppes. Thought the -40 °C (-40 °F). The temperature is around 25°C temperature and rainfall is nearly same, in July. Hence, Option B is incorrect. nomadic herding is not practices here. People Mediterranean region has wet winters. Mean here practice mechanised farming. Hence, annual precipitation ranges from 35 – 90 cm. Option C is incorrect Temperature of warmest month greater than or Siberian Tundra –is a polar type of climate. equal to 10⁰ C. Temperature of coldest month is very low mean annual temperature. In mid- winter temperatures are as low as 40 – 50 °C

www.laexias.com Page No. 45 https://elearn.laex.in Geography UPSC Previous Year Questions

below freezing. Summers are relatively warmer. Educational Objective: To have knowledge Precipitation is mainly in the form of snow and about the wind systems and their region of sleet. Human activities of the tundra are largely effectivity along with their characteristics. confined to the coast. People live a semi- 44. Key: D nomadic life. Hence, option D is incorrect. Explanation: The common known phenomenon To know about the Educational Objective: is that the sun rises is the east and set’s in the different climatic conditions and their features west but actually it’s is the earth’s rotation on like temperature, rainfall, flora and fauna. it’s own axis which is responsible for day and 43. Key: B night on the earth. Earth rotates from west to Explanation: Westerlies has different names as east that places on the east receive the first of it crosses different latitudes. It is roaring forties sun i.e., countries to the west of 180 degree in 40° latitudes, furious fifties in 50° and meridian receive the first sunlight. Thus the shrieking sixties in 60° latitude. 92’30’ E Meridian which lies nearer to the west Westerlies blow from the sub-tropical high- of 180 degree receives the first sun light. pressure regions to the sub-polar low-pressure Educational objective: to know about rotation regions in both the hemispheres. They and revolution and understand how they are interrupted in between by the trade winds. responsible for day and night and seasons. Hence, statement 1 is incorrect. 45. Key: C By elimination, the answer is B. Explanation: Methane explosions occur in In the northern hemisphere, land masses cause mines when a build up of methane gas, a by a lot of disruption and so the winds become product of , comes into contact with a heat weak. In the southern hemisphere, between the source, and there is not enough air to dilute latitudes 40° S and 60° S there is a continuous the gas to levels below its explosion point. and the expanse of ocean Westerlies gain Hence, option C is correct. considerable strength and persistence. Hence, Hydrogen and oxygen when combined forms Statement 2 is correct. water which is not explosive. Hence, option A is In the northern hemisphere they blow from the incorrect. south-west to the north-east and in the Oxygen with acetylene is usually used in welding southern hemisphere they blow from the of metals. Therefore option B rules out. north-west to the south-east. Hence, Carbon di oxide with ethane leads to statement 3 is correct. dehydrogenation of ethane forming ethylene. They bring plenty of rain most of the year Hence option D is also incorrect. because having originated in the warmer regions Educational Objective: To learn about basic these winds bears plenty of moisture. Hence, chemical reactions occurring in atmosphere. statement 4 is also correct.

www.laexias.com Page No. 46 https://elearn.laex.in Geography UPSC Previous Year Questions

46. Key: C 48. Key: B Explanation: Land covered with fresh snow Explanation: The International Date Line, reflects back more sunlight as compared to other established in 1884, passes through the mid- Pacific Ocean and roughly follows a 180 degrees three. Fresh snow and ice reflect 80 - 85% of longitude north-south line on the Earth. the sunlight. Therefore, the skin of skiers is Straits of Malacca is a narrowstretch of water irradiated by sunlight from all sides, since the between the and the snow does not take up the light but reflects it Indonesian island of Sumatra. and remains cold. Hence, option C is correct. The Bering Strait is a strait of the Pacific, which Sand deserts absorb the sunlight and hence are separates and the United States (Alaska) slightly south of the . International very hot. Hence, option A is incorrect. Date Line passes through it. Hence B option is Paddy crop land and Prairies have plants over the correct answer. them and plants absorb some sunlight for Florida Strait is a strait located south-southeast photosynthesis. Hence, option B and D are of the North American mainland, between the incorrect. and the Atlantic Ocean. Educational Objective: To know about the Educational Objective: To know the Location of albedo of the earth at different places. International date Line and straits all over the 47. Key: C world. Explanation: The earth is divided into 24 time 49. Key:D zone based on the longitudes with 15 degree Explanation: The IST is the Green which longitude at every 1 hour. And as one moves meridian near London which represent 0 degree from 0 degree longitude one loose time as he longitude and Places east of it are advance in moves left and gains time as the moves right time by 1 hour for every fifteen degree and respectively of 1 hour for every . At 180 degree similarly places west of it are back ward in time longitude or International Date Line there is by 1 hour for every 15 degree. Thus when it is total difference of 24 hours. 12 pm at 0 degree meridian midway between o Thus when one moves from west to east of degree and 180 w is the place where it will be International date line one gains 24 hours and if morning 6 am . he crosses it from east to west he looses 24 Thus option D is the correct answer hours. Educational Objective: To understand concept Aleutian islands is situated to west of IDL and of International Date line and how it’s Dutch to east thus there is movement from west responsible for date changes confusingly. to east and hence 24 hours are added and hence as he is seeing time after 1 hour the current 50. Key: A answer is option C 0:30 HOURS Jan 3 Explanation: Global warming is the increase in Educational Objective: To understand concept the average temperature of Earth's near-surface of International Date Line and how it’s air and oceans by increasing concentrations of responsible for date changes confusingly. greenhouse gases, which result from human

www.laexias.com Page No. 47 https://elearn.laex.in Geography UPSC Previous Year Questions

activity such as the burning of fossil fuel and Select the correct answer using the code deforestation. Climate change mitigation is the given below: action to decrease the potential effects of global (a) 1 only warming. At the present time, rate of mitigation (b) 2 & 3 is less in India because appropriate technologies (c) 2 only are limited and costly. Moreover India doesn’t (d) 1 & 3 have huge funds required for research and 3. Consider the following pairs: development in that area. So, Statement 1 and 2 Town sometimes Country are correct. mentioned in news Statement 3 is wrong as setting up of polluting 1) Aleppo industries in India by developed countries 2) Kirkuk cannot be considered as a limitation of India in 3) Mosul mitigating the global warming challenge. 4) Mazar- i- Sharif Educational Objective: Learning about global Which of the pairs given above are correctly warming and India’s planning to tackle it. matched? 5. World Physical Features (a) 1 & 2 (b) 1 & 4 1. Consider the following pairs: (c) 2 & 3 Sea Bordering country (d) 3 & 4 1) Albania 4. Consider the following pairs: 2) Black Sea Croatia Regions sometimes Country 3) Caspian Sea mentioned in news 4) 1) Catalonia 5) Syria 2) Hungary Which of the pairs given above are correctly 3) Philippines matched? 4) Oromia Nigeria (a) 1, 2 and 4 only Which of the pairs given above are correctly (b) 1, 3 and 4 only matched? (c) 2 and 5 only (a) 1, 2 &3 (d) 1, 2, 3, 4 and 5 (b) 3 &4 only 2. Which of the following has/have shrunk (c) 1 & 3 only immensely/dried up in the recent past (d) 2 & 4 only due to human activities? 1) 2) Black Sea 3)

www.laexias.com Page No. 48 https://elearn.laex.in Geography UPSC Previous Year Questions

5. Mediterranean Sea is a border of which of 9. What is the correct sequence of the following countries? occurrence of the following cities in 1) Jordan South-East Asia as one proceeds from 2) Iraq south to north? 3) Lebanon 1) Bangkok 4) Syria 2) Hanoi Select the correct answer using the code 3) Jakarta given below: 4) (a) 1, 2 & 3 only Select the correct answer using the code (b) 2 & 3 only given below (c) 3 & 4 only (a) 4-2-1-3 (d) 1 ,3 & 4 only (b) 3-2-4-1 6. Consider the following pairs: (c) 3-4-1-2 Community In the affairs of (d) 4-3-2-1 sometimes mentioned in the news 1) Kurd 10. Which one of the following can one 2) Madhesi comes across if one travels through the 3) Rohingya Myanmar Strait of Malacca? Which of the pairs given above is/are (a) Bali correctly matched? (b) (a) 1 and 2 (c) Java (b) 2 only (d) Singapore (c) 2 and 3 11. Consider the following regions: (d) 3 only 1) Eastern Himalayas 2) region 7. The area known as '' sometimes appears in the news in the 3) North- context of the events related to Which of the above is/are Biodiversity (a) Central Asia Hotspot(s)? (b) (a) 1 only (c) South-East Asia (b) 1 and 2 only (d) (c) 2 and 3 only (d) 1, 2 and 3 8. Turkey is located between (a) Black Sea and Caspian Sea 12. Consider the following pairs: (b) Black Sea and Mediterranean Sea Famous Place Country (c) and Mediterranean Sea 1) Cannes France (d) and 2) Davos Denmark

www.laexias.com Page No. 49 https://elearn.laex.in Geography UPSC Previous Year Questions

3) Roland Garros The Netherlands (b) London Thames Which of the pairs given above is/are (c) New York Hudson correctly matched? (d) Vienna (a) 1 only 17. Cape Canaveral, the site from which (b) 1 and 2 only space shuttles are launched is located on (c) 2 and 3 only the coast of (d) 1, 2 and 3 (a) Florida (b) Virginia 13. Which one of the following is the (c) North Carolina country's administrative capital/new (d) South Carolina federal administrative centre of Malaysia? (a) Kota Bharu 18. In which one of the following is Malta (b) Kuala Terengganu located? (a) Baltic Sea (c) Putrajaya (b) Mediterranean Sea (d) Taiping (c) Black Sea 14. Elephant pass, which is frequently in the (d) news, is mentioned in the context of the affairs of which one of the following? 19. Which of the following cities is nearest to (a) Bangladesh the equator? (b) India (a) Colombo (c) Nepal (b) Jakarta (d) (c) Manila (d) Singapore 15. Consider the following countries: 1) Australia 20. Which of the following countries share 2) Namibia borders with Moldova? 3) Brazil 1) Ukraine 4) Chile 2) Romania Through which of the above does the Tropic 3) Belarus of Capricorn pass? Select the correct answer using the code (a) 1 only given below: (b) 2, 3 and 4 Codes: (c) 1, 2 and 3 (a) 1 & 2 only (d) 1, 2, 3 and 4 (b) 2 & 3 only 16. Which one of the following pairs is not (c) 1 & 3 only correctly matched? (d) 1, 2 & 3 City River 21. Consider the following statements: (a) Berlin Rhine

www.laexias.com Page No. 50 https://elearn.laex.in Geography UPSC Previous Year Questions

1) Ajman is one of the seven Emirates of the (c) Strait of Dover UAE. (d) Strait of Gibraltar 2) Ras al-Khaimah was the last Sheikhdom to 25. Other than India and China, which of the join the UAE following groups of countries border Which of the statements given above is/are Myanmar? correct? (a) Bangladesh, Thailand and Vietnam (a) 1 only (b) Cambodia, Laos and Malaysia (b) 2 only (c) Thailand, Vietnam and Malaysia (c) Both 1 and 2 (d) Thailand, Laos and Bangladesh (d) Neither 1 nor 2 26. Through which one of the following 22. Where is Copacabana Beach located? groups of countries does the Equator (a) Buenos Aires pass? (b) Hawaiian Islands (a) Brazil, Zambia and Malaysia (c) Rio de Janeiro (b) Colombia, Kenya and Indonesia (d) Valletta (c) Brazil, and Malaysia 23. Match List I with List II and select the (d) Venezuela, Ethiopia and Indonesia correct answer using the code given 27. Consider the following statements: below the list: 1) Length of a terrestrial mile is lesser than List-I (City) List-II (River) that of a nautical mile. 2) Harmattan is a dusty land-wind of the East A. Bangkok 1. Irrawaddy African Coast B. Phnom-Penh 2. Mekong 3) Greece and Albania form a part of the C. Hanoi 3. Menam (Chao Phraya) Which of the statements given above is/are D. Yangon 4. Red River correct? Codes: (a) 1, 2 and 3 (a) A-3; B-2; C-4; D-1 (b) 2 and 3 only (b) A-4; B-1; C-3; D-2 (c) 3 only (c) A-3; B-1; C-4; D-2 (d) 1 only (d) A-4; B-2; C-3; D-1

28. Claims to the historical Macedonian 24. Through which one of the following territory have been a bone of contention Straits, does a tunnel connect the United between which of the following countries? Kingdom and France? (a) Portugal and Spain (a) Davis Strait (b) Bulgaria and Greece (b) Denmark Strait (c) Romania and Bulgaria

www.laexias.com Page No. 51 https://elearn.laex.in Geography UPSC Previous Year Questions

(d) Portugal and Greece (c) Peshawar-Islamabad-Gujranwala-Multan

29. Huangpu River flows through which one (d) Islamabad-Multan-Peshawar-Gujranwala of the following cities? 34. Which one of the following pairs is not (a) Beijing correctly matched? (b) Ho Chi Minh City (a) Bahamas Nassau (c) Shanghai (b) Costa Rica San Jose (d) Manila (c) Nicaragua Belmopan

30. Bermuda triangle extends up to which of (d) Dominican Republic Santo Domingo the following places? 35. Match List-I (City) with List II (River) and 1) Southern Florida select the correct answer using the code 2) Puerto Rico given below the lists: 3) Hawaii Islands List-I (City) List-II (River) Select the correct answer using the code A. Washington D C 1. River Manzanares given below: B. Berlin 2. River Seine (a) 1,2 & 3 C. Paris 3. River Spree (b) 1 & 2 only D. Madrid 4. River Potomac (c) 2 & 3 only Codes: (d) 1 & 3 only (a) A-2; B-3; C-4; D-1

31. Which one of the following countries (b) A-4; B-1; C-2; D-3 does not border Lithuania? (c) A-2; B-1; C-4; D-3 (a) Poland (d) A-4; B-3; C-2; D-1 (b) Ukraine 36. Which one of the following is the correct (c) Belarus sequence of the given Continents in the (d) Latvia decreasing order of their percentage of

32. Where are the Balearic Islands located? Earth's land? (a) Mediterranean Sea (a) NorthAmerica-Africa-South America-Europe (b) Black Sea (b) Africa-North-America-South-America- (c) Baltic Sea Europe (d) North Sea (c) North America-Africa-Europe-South America 33. Which one of the following is the correct (d) Africa-North America-Europe-South sequence of the given towns of Pakistan America while moving from the North towards the South? 37. For which one of the following countries, (a) Islamabad Gujranwala-Peshawar-Multan is Spanish not an ? (b) Peshawar-Gujranwala-Multan-Islamabad (a) Chile

www.laexias.com Page No. 52 https://elearn.laex.in Geography UPSC Previous Year Questions

(b) Colombia 2) England covers less than 60% of the total (c) Republic of Congo area of the United Kingdom (d) Cuba Which of the statements given above is/are correct: 38. Which one of the following pairs is not (a) 1 only correctly matched? (b) 2 only (a) Seikan Rail Tunnel: China (c) Both 1 and2 (b) Petronas Towers: Malaysia (d) Neither 1 nor 2 (c) Appalachian Trail: United States of America 43. The great Asian river Mekong does not (d) : run through 39. Which one of the following cities is not a (a) China former capital of the given country (b) Malaysia (Country given in the brackets)? (c) Cambodia (a) Karachi (Pakistan) (d) Laos (b) Auckland (New Zealand) 44. Latvia does not share its borders with (c) Kyoto (Japan) which one of the following countries? (d) Brisbane (Australia) (a) Russia

40. Itaipu Dam built on the River Parana is (b) Estonia one of the largest dams in the world. (c) Lithuania Which of the following two countries (d) Poland have this as a joint project? 45. Match List-I (State/Province/Overseas (a) Brazil and Peru Territory) with List-II (Country) and (b) Paraguay and Ecuador select the correct answer using the codes (c) Brazil and Paraguay given below the lists: (d) Colombia and Paraguay List-I List-II A. British Colombia 1. USA 41. Which one of the following pairs is not B. Bavaria 2. UK correctly matched? C. Gibraltar 3. Canada Current name Old Name D. Rhode Island 4. Germany (a) Harare Salisbury 5.Denmark (b) Ethiopia Abyssinia (c) Ghana Dutch Guiana Codes: (a) A-1; B-2; C-5; D-3 (d) Kinshasa Leopoldville (b) A-3; B-4; C-2; D-1 42. Consider the following statements: (c) A-1; B-4; C-2; D-3 1) Great Britain comprises England, Wales, (d) A-3; B-2; C-5; D-1 Scotland and Northern Ireland.

www.laexias.com Page No. 53 https://elearn.laex.in Geography UPSC Previous Year Questions

46. Match List-I (Sea) with List-II (Country) (d) Turkey, Syria, Iraq and Yemen and select the correct answer using the 51. Which one of the following countries is codes given below the Lists: landlocked: List-I List-II (a) Bolivia A. Black Sea 1. Bulgaria (b) Peru B. Red Sea 2. China (c) Surinam C. 3. Eritrea (d) Uruguay D. Caspian Sea 4. Kazakhstan 52. In the map given below four islands of Codes: Indian Ocean region i.e., (A) Seychelles, (a) A-1; B-4; C-2; D-3 (B) Chagos, (C) Mauritius and (D) Socotra (b) A-2; B-3; C-1; D-4 are marked as 1, 2, 3 and 4. (c) A-1; B-3; C-2; D-4 (d) A-2; B-4; C-1; D-3 Match them and select the correct answer from the codes given below: 47. Which one of the following does not border Panama? (a) Costa Rica (b) Pacific Ocean (c) Colombia (d) Venezuela

48. The waterfall 'Victoria' is associated with Codes: the river (a) A-1; B-3; C-4; D-2 (a) Amazon (b) A-3; B-1; C-2; D-4 (b) Missouri (c) A-1; B-3; C-2; D-4 (c) St. Lawrence (d) A-3; B-1; C-4; D-2 (d) Zambeji

53. Consider the following countries: 49. Which one of the following countries 1) Albania does NOT border the Caspian Sea? 2) Bosnia Herzegovina (a) Armenia 3) Croatia (b) 4) Macedonia (c) Kazakhstan Which of these countries has/have Adriatic (d) Sea as a boundary? 50. Israel has common borders with: (a) 1 and 2 (a) Lebanon, Syria, Jordan and Egypt (b) 1, 2 and 3 (b) Lebanon, Syria, Turkey and Jordan (c) 4 only (c) Cyprus, Turkey, Jordan and Egypt (d) 3 and 4

www.laexias.com Page No. 54 https://elearn.laex.in Geography UPSC Previous Year Questions

(b) Santos 54. The given map shows four towns of (c) Rio de Janerio Central Asian region marked as 1, 2, 3 (d) Buenos Aires and 4. Identify these from the following list and select the correct answer using 57. Which one of the following lakes forms as the codes given below: international boundary between Tanzania A. Bishkek and Uganda. B. Ashkhabad (a) Chad C. (b) Malawi D. (c) Victoria (d) Zambezi

58. Assertion: In Australia, Cattle rearing is done more for meat than for milk. Reason: Australians are traditionally non- vegetarians. Codes: (a) Both A are R are true R is the correct (a) A-3; B-1; C-2; D-4 explanation of A (b) A-3; B-1; C-4; D-2 (b) Both A and R are true but R is not a correct (c) A-1; B-3; C-2; D-4 explanation of A (d) A-1; B-3; C-4; D-2 (c) A is true but R is false 55. At which one of the cities labelled as A, B, (d) A is false but R is true

C and D on the given map of Europe was 59. Match the drainage basins labelled as A, the historic treaty between NATO and B, C and D with the names listed below Warsaw Pact countries signed in 1998? and select the correct answer using the codes given below the names of the drainage basins. 1. Ganga-Brahmaputra 2. Indus 3. Parana 4. Zambezi (a) A (b) B (c) C (d) D

56. Which one of the following is known as the 'Coffee Port’ of the world? (a) Sao Paulo

www.laexias.com Page No. 55 https://elearn.laex.in Geography UPSC Previous Year Questions

Codes: 5. Singapore (a) A-3; B-1;C-2; D-4 Codes: (b) A-1; B-3; C-4; D-2 (a) A-1; B-2; C-4; D-3 (c) A-1; B-3; C-2; D-4 (b) A-2; B-1; C-4; D-3 (d) A-3; B-1; C-4; D-2 (c) A-1;B-4; C-5; D-2 (d) A-4; B-3; C-5; D-2 60. Which one of the following port cities in Venezuela been developed as an oil port? 63. Match List-I with List-II and select the (a) Caracas correct answer using the codes given below the lists: (b) Maracaibo List-I (Timber) List-II (Country) (c) Maracay A. Cedar 1. Myanmar (d) Carupano B. Douglas Fir 2. Canada 61. Assertion: Chile continues to be an C. Mahogany 3. Mexico important producer of in the world. D. Teak 4. Honduras Reason: Chile is endowed with the world's Codes: largest deposit of porphyry copper. (a) A-3; B-2; C-1; D-4 (a) Both A are R are true R is the correct (b) A-3; B-2; C-4; D-1 explanation of A (c) A-2; B-3; C-4; D-1 (b) Both A and R are true but R is not a correct (d) A-2; B-3; C-1; D-4 explanation of A (c) A is true but R is false 64. The physical regions marked as 1, 2, 3 (d) A is false but R is true and 4 on the given map are respectively

62. Match the cities labelled as A, B, C and D in the given map with the names of cities and select the correct answer using the codes given below the names of cities

(a) Andes, Brazilian Shield, Guyana Highlands and (b) Andes, Guyana Highlands, Brazilian Shield Names of Cities and Amazon Basin 1. Darwin (c) Amazon Basin, Guyana Highlands, Brazilian 2. Kuala Lumpur Shield and Andes 3. Lagos (d) Guyana Highlands, Brazilian Shield, Andes 4. Nairobi and Amazon Basin

www.laexias.com Page No. 56 https://elearn.laex.in Geography UPSC Previous Year Questions

 65. Which one of the following pairs is not Black sea is bordered by Ukraine to the north, to the northeast, to correctly matched? Russia Georgia (a) Solvenia Bratislava the east, Turkey to the south, (b) Seychelles Victoria and Bulgaria and Romania to the west. (c) Sierra Leone Freetown Educational Objective:To know about the important places, around the world which (d) Tashkent are in news due to clashes between 66. Which one of the following countries of countries, conflict, etc South-West Asia does not open out to the 2. Key: D Mediterranean Sea? : The Aral sea in central Asia used (a) Syria Explanation to be the fourth largest lake in the world after (b) Jordan the Caspian sea, and Superior and Victoria. Now (c) Lebanon barely 10% of it is left. While Lake Baikal has (d) Israel been crippled by a series of phenomenon like 5. Key and Explanation rapid growth of putrid algae and the death of endemic species of sponges across its vast 3.2 1. Key: B ml hectare area. Explanation: The capsian Sea is a landlocked Shrinking in various lakes of world: Lake body of water between Europe and Asia i.e., Five Poopó – Bolivia: Lake Poopó was a large saline Countries Russia, Kazakhistan, Turkmenistan, lake located in the in Bolivia. Glacial Iran, and Azerbaijan. Thus 3 is correctly melting and the diversion of the lake’s matched. tributaries contributed to its decline and now  The countries with coasts on the Adriatic the lake is almost dry. are Albania, Bosnia Dead Sea – Jordan, Israel, Palestine: The Dead &Herzegovina, , , a Croatia Italy Montenegro Sea, the lowest point on Earth, has been nd . The southern boundary of the Slovenia shrinking at a rate of around one metre each sea ends in the Strait of Otranto year, for the last 50 years. One of the main between and Salento Peninsula, Albania Italy's reasons behind this depletion is that some of the thus 1 is correctly matched. So option b water sources it relied on were diverted in the is right. 1960s.  The countries surrounding Aral Sea – Kazakhstan and Uzbekistan: The Aral the Mediterranean in clockwise order are Sea has been shrinking steadily since the 1960s Spain, France, Monaco, Italy, Slovenia, after its tributaries were diverted by Soviet Croatia, Bosnia and Herzegovina, irrigation projects. By 2007, the lake had Montenegro, Albania, Greece, Turkey, Syria, declined to 10% of its original size and had split Lebanon, Israel, Egypt, Libya, Tunisia, into four separate lakes. The eastern basin of Algeria, and Morocco

www.laexias.com Page No. 57 https://elearn.laex.in Geography UPSC Previous Year Questions

the former has now completely Kirkuk is in Iraq, which was in news due to ISIS dried up and is called the . controlled territory. Poyang Lake – China: China’s largest freshwater Thus option B is right. lake has been shrinking dramatically since the Some of important places in Syria start of the century. At one point, the lake covered 4,500 square kilometres but its surface has been recently recorded as low as 200 square kilometres. The main reasons for this dramatic decrease are diversions from the River, a prolonged dry season, and industrial activities. Hulun Lake – : The Mongolian Plateau has seen dramatic shrinkage of its lakes Educational Objective:To know about the over the last few decades, mainly due to important places, around the world which are in intensive human activities and climate change. news due to clashes between countries, conflict, One of the biggest lakes on the plateau, Hulun, etc has lost 291 square kilometres of surface area. 4. Key: C The smaller lake of Xinkai to the east of Hulun Explanation: Referendum was held in October Lake had dried out completely by 2010. 2017 in which 90% of Catalonian voted for Lake Chad – Chad, Niger, Nigeria, Cameroon: independence but the voter turnout was only Once one of Africa’s largest bodies of fresh water, 43%.It was alleged that Spanish National Police Lake Chad is now a ghost of its former self. tried to stop people from voting which led to According to a study by University of Wisconsin- widespread violence. However, the referendum Madison researchers, the lake is now one 20th was declared illegal by Spain’s central of the size it was 35 years ago. Massive irrigation government as it was marred by wide scale projects, an increasingly dry climate, and violence. The referendum has thrown the declining rainfall have all contributed to its country into its worst constitutional crisis. decline.  Ukraine was one of the republics within the Educational Objective:To know about the USSR during the cold war days, and has important places sea which are in news due to remained a strong ally of Russia ever since, ecological importance. till 2013. While it was planning to sing an 3. Key: B association agreement with the European Explanation: Aleppo is in Syria, it was in news Union in 2013, Russia sternly objected to it, due to Syrian conflict, Mazar-i-Sharif is in leading to tensions. Russia subsequently Afganistan where 140 soldiers were killed in annexed “Crimea” (Russian speaking terror attack, our Prime minister condemned province in Ukraine) by force and declared this act.

www.laexias.com Page No. 58 https://elearn.laex.in Geography UPSC Previous Year Questions

its sovereignty over it with people’s known as Kurdistan, which spans southeastern support. Turkey, northwestern Iran, northern Iraq, and  Marawi City, is the capital and the largest northern Syria. There are also exclaves city of the province of Lanao del Sur on the of Kurds in central and Khorasan. island of Mindanao in the Philippines. Marawi siege started in May 2017 when Philippine government security forces began to take on heavily armed militants linked to the Islamic State. Educational Objective: to know about the important places, around the world which are in news due to clashes between The Madhesi are an of Nepal. countries, conflict, etc are Muslims from the Rakhine 5. Key: C state of Mynmar. Explanation: The countries surrounding Educational Objective:To know about the the Mediterranean in clockwise order are important places, around the world which are in Spain, France, Monaco, Italy, Slovenia, news due to clashes between countries, conflict, Croatia, Bosnia and Herzegovina, etc Montenegro, Albania, Greece, Turkey, Syria, Lebanon, Israel, Egypt, Libya, Tunisia, 7. Key: B Algeria, and Morocco; Malta and Cyprus are Explanation: The Golan Heights is a rocky island countries in the sea. plateau with an area of 1,800km² on the border between Israel and Syria in south-western Syria. Dispute Israel occupied the Golan Heights, West Bank, East Jerusalem, and the in the 1967 Six-Day War. An armistice line was established and the region came under Israeli military control. Syria tried to retake the Golan Heights during the 1973 Middle East war. Syria was defeated in Educational Objective:To know about the its attempt. Both countries signed an armistice countries around the sea, bordering in 1974 and a UN observer force has been in countries, landlocked countries etc place on the ceasefire line since 1974. In 1981, Israel permanently acquired the 6. Key: C territory of the Golan Heights and East Explanation: The Kurds are an ethnic group native to a mountainous region of Jerusalem in moves never recognized by most

www.laexias.com Page No. 59 https://elearn.laex.in Geography UPSC Previous Year Questions

countries. The international community regards as disputed territory occupied by Israel whose status should be determined by negotiations between Israel and Syria. Attempts by the international community to bring Israel and Syria for negotiations have failed.

Educational Objective: to know about the important places and their direction with respect north and south.

10. Key: D Educational Objective: To know about the Explanation: important places, around the world which are in news due to clashes between countries, conflict, etc

8. Key: B Explanation: Turkey is located between Black Sea and Mediterranean Sea. Educational Objective: To know about the countries around the sea, bordering countries, landlocked countries and important cannels, straits etc

11. Key: A Explanation: Eastern Himalayas and Eastern Mediterranean region are Biodiversity hotspots. Educational Objective: To know about the South west part of Australia has hotspots. countries around the sea, bordering countries,  A biodiversity hotspot is a biogeographic landlocked countries etc region that is both a significant reservoir 9. Key: C of biodiversity and is threatened with destruction. The term biodiversity Explanation: hotspot specifically refers to 25 biologically

www.laexias.com Page No. 60 https://elearn.laex.in Geography UPSC Previous Year Questions

rich areas around the world that have lost Educational Objective: To know about the at least 70 percent of their original habitat. important places, around the world which are in  India shares its territories into news due to clashes between countries, conflict, three biodiversity hotspots viz. Eastern etc Himalaya, and Indo- Burma. Biodiversity Hotspots also work as 15. Key: D funding regions for Conservation Explanation: The Tropic of Capricorn passes International for its Critical Ecosystem through a number of countries including Partnership Fund (CEPF) Argentina, Australia, Botswana, Brazil, Chile, Educational Objective: to know about the , Mozambique, Namibia, and important places sea which are in news due to Paraguay. If you consider its starting place to be ecological importance. the Prime Meridian, it first makes landfall on the 12. Key: A coast of Namibia. Explanation: Davos is in Switzerland and Roland Garros is in England. Educational Objective:To know about the important places, around the world which are in news due to clashes between countries, conflict, etc

13. Key: C Explanation: The 3 federal territories were formed for different purposes: Kuala Lumpur is the national capital, Putrajaya is Educational Objective:To know about the the administrative centre of important places and their direction with respect the federal government, and Labuan serves as north and south. And countries which are in the an offshore financial centre. places where important latitudes passes. Educational Objective:To know about the important places and their direction with respect 16. Key: A north and south. Explanation: Berlin is located in north eastern Germany on the banks of the rivers spree and 14. Key: D Havel. Explanation: Elephant Pass, Northern Province, Educational Objective: To know about the Sri Lanka is located in the gateway of Jaffna important places and their direction with respect Peninsula. There are about 340km north from north and south cities located on the banks of capital to here. It has an important military base the rivers. The countries which are in the places and used to be the island's largest salt field. It where important latitudes passes through. has regularly been the site of battles during the civil war.

www.laexias.com Page No. 61 https://elearn.laex.in Geography UPSC Previous Year Questions

17. Key: A 20. Key: A Explanation: Launch operations for the Apollo, Explanation: Located in , Skylab and Space Shuttle programs were Moldova is bordered on the west and southwest carried out from by Romania and on the north, south, and east Kennedy Space Centre Launch Complex 39 and by Ukraine. managed by KSC. Located on the east coast of Educational Objective: To know about the Florida, KSC is adjacent to Cape Canaveral Air countries around the sea, bordering countries, Force Station (CCAFS). landlocked countries and important cannels, Educational Objective:To know about the straits etc important places and their direction with respect 21. Key: C north and south cities located on the banks of Explanation: Ajman is one of the seven emirates the rivers. The countries which are in the places constituting the United Arab Emirates with an where important latitudes passes through. area of 260 sq kilometer. Ras al-Khaimah was the 7th and last Sheikhdom to join UAE in 1972. 18. Key: B Educational Objective: To know about the The island-state of Explanation: Malta countries around the sea, bordering countries, is in the Mediterranean Sea, south of located landlocked countries and important cannels, Sicily (Italy), it consists of three islands: , Malta straits etc Gozo, and Comino, of which Malta is the largest 22. Key: C island. In its history, the Maltese archipelago Explanation: Copacabana located in the South was always strategically significant for the Zone of the city of Rio de Janeiro, Brazil. It is domination of the Mediterranean. known for its 4 km (2.5 miles) balneario beach, Educational Objective: To know about the which is one of the most famous in the world. countries around the sea, bordering countries, Educational Objective: To know about the landlocked countries and important cannels, countries around the sea, bordering countries, straits etc landlocked countries and important cannels, 19. Key: D straits etc Explanation: Some of the cities that 23. Key: A are near the equator include Bogota, Singapore, Explanation: and Nairobi. Bangkok - Menam (Chao Phraya) Educational Objective: To know about the Phnom - Penh -Mekong important places and their direction with respect Hanoi - Red River north and south cities located on the banks of Yangon - Irrawaddy the rivers. The important countries, places To know about the where important latitudes passes through. Educational Objective: important places and their direction with respect north and south cities located on the banks of

www.laexias.com Page No. 62 https://elearn.laex.in Geography UPSC Previous Year Questions

the rivers. The important countries, places Uganda, Kenya, Somalia, , where important latitudes passes through. Indonesia and .

24. Key: C Explanation: The Channel Tunnel or Chunnel is a 50km-long undersea rail tunnel below the Strait of Dover in the . It is one of the longest underwater tunnels in the world and connects Folkestone in Kent, UK, with Coquelles in Pas-de-Calais, France Educational Objective: To know about the countries around the sea, bordering countries, landlocked countries and important cannels, Educational Objective: To know about the straits etc important places and their direction with respect north and south. Cities located on the banks of 25. Key: D the rivers. The important countries, places Other than India and China Explanation: where important latitudes passes through. Thailand, Laos and Bangladesh are countries 27. Key: D bordering Myanmar. Explanation: In statement 1st one nautical mile is 1852 mtrs and one terrestrial mile is 1609 metres. So, statement '1' is correct. In statement 2nd Harmattan is a dry and dusty wind of West Africa. So, statement '2' is wrong. In statement '3' Iberian peninsula includes Portugal, Spain, Andorra and Gibraltar. Greece and Albania are not a part of Peninsula. So, statement '3' is wrong. Educational Objective: To know about the Educational Objective: To know about the important places and their direction with respect countries around the sea, bordering countries, north and south. cities located on the banks of landlocked countries and important cannels, the rivers. The important countries, places straits etc where important latitudes passes through. 26. Key: B 28. Key: B Explanation: The equator passes Explanation: Macedonian territory is disputed through 13 countries: Ecuador, Colombia, between Bulgaria & Greece. Brazil, Sao Tome & Principe, Gabon, Republic of Educational Objective: To know about the the Congo, Democratic Republic of the Congo, important places, around the world which are in

www.laexias.com Page No. 63 https://elearn.laex.in Geography UPSC Previous Year Questions

news due to clashes between countries, conflict, Educational Objective: To know about the etc important places and their direction with respect north and south cities located on the banks of 29. Key: C the rivers. The important countries, places Explanation: The Huangpu river is the largest where important latitudes passes through. river in Shanghai in China. It is 113.07 km long 400 metres wide and 9 metres deep. Shanghai gets most of its drinking water from Huangpu. 31. Key: B Explanation: Lithuania is situated on the eastern shore of the Baltic Sea and borders Latvia on the north, Belarus on the east and south, and Poland and the Kaliningrad region of Russia on the southwest.

Educational Objective: To know about the important places and their direction with respect north and south. cities located on the banks of the rivers. The important countries, places where important latitudes passes through.

30. Key: B Explanation: Bermuda Triangle is a strange and mysterious place in the southern Atlantic Ocean. It is roughly the shape of a triangle. The triangle Educational Objective: To know about the extends upto south Florida, Pruto Rico and countries around the sea, bordering countries, Bermuda Island. landlocked countries and important cannels, straits etc

32. Key: A Explanation: Balearic Islands are located in Mediterrian sea. It is an autonomous division of Spain and the North African Coast. Educational Objective: To know about the countries around the sea, bordering countries, landlocked countries and important cannels, straits etc

www.laexias.com Page No. 64 https://elearn.laex.in Geography UPSC Previous Year Questions

landlocked countries and important cannels, 33. Key: C straits etc Explanation: While moving from the north to south the correct sequence of the towns is 36. Key: B Peshawar, Islamabad, Gujranwala, and Multan. Explanation: Percent of Area in Square Miles Total Land Continent (Square Km) Area on Earth 57,308,738 Sq. Miles The World 100% (148,429,000 Sq. Km) Asia (including 17,212,000 Sq. Miles 30.0% the Middle East) (44,579,000 Sq. Km) 11,608,000 Sq. Miles Africa 20.3% (30,065,000 Sq. Km) 9,365,000 Sq. Miles North America 16.3% (24,256,000 Sq. Km) 6,880,000 Sq. Miles South America 12.0% Educational Objective: To know about the (17,819,000 Sq. Km) important places and their direction with respect 5,100,000 Sq. Miles 8.9% north and south. cities located on the banks of (13,209,000 Sq. Km) 3,837,000 Sq. Miles the rivers. The important countries, places Europe 6.7% (9,938,000 Sq. Km) where important latitudes passes through. Australia (plus 2,968,000 Sq. Miles 5.2% 34. Key: C ) (7,687,000 Sq. Km)

Explanation: Managua is the capital of 37. Key: C Nicaragua, not Balmopan. Balmopan is the Explanation: The official language of the capital city of Balize. Republic of Congo is French. Other languages Educational Objective: To know about the are mainly Bantu languages, and the two countries around the sea, bordering countries, national languages in the country landlocked countries and important cannels, are Kituba and Lingala (13%). straits etc Educational Objective: To know about 35. Key: D languages of the world.

Explanation: 38. Key: A A. Washington D C 1. River Potomac Explanation: Seikan Rail Tunnel is world's B. Berlin 2. River Spree longest tunnel with length of 34 miles is located C. Paris 3. River Seine in Japan. Seikan railway tunnel is Japan's D. Madrid 4. River Manzanares 53.85 km long tunnel beneath the Tsugaru Educational Objective: To know about the Strait and connects the Aomori Prefecture on countries around the sea, bordering countries, Honshu Island and the Hokkaido Island. With

www.laexias.com Page No. 65 https://elearn.laex.in Geography UPSC Previous Year Questions

its track located 140m below the seabed, the Cambodia Kampuchea Seikan tunnel is the world's deepest and longest Ethiopia Abyssinia railway tunnel. Ghana Coast Educational Objective: To know about the Harare Salisbury countries around the sea, bordering countries, landlocked countries and important cannels, Indonesia Dutch straits etc Iran Persia/

39. Key: D Istanbul Constantinople/Byzaiu Explanation: Karachi was the capital of Jakarta Batavia Pakistan from 1947 – 1959. Kyoto was the Japan Nippon capital of Japan before Tokyo. Brisbane was the Malaysia Malaya capital of Australia before Cannbera in 1842 – Myanmar Burma 1859. Educational Objective: To know about the Sri Lanka Ceylon countries around the sea, bordering countries, St Petersburg Leningrad landlocked countries and important cannels, Taiwan Formosa straits etc Thailand Siam 40. Key: C Togo Togoland Explanation: The project is located on United Arab Republic Egypt the Parana River, at the border between Brazil Zaire Congo and Paraguay. Itaipu dam with an installed capacity of 14,000MW ranks as Zambia Northern the world's second largest plant. Zimbabwe To know about the Educational Objective: Educational Objective: To know about the countries around the sea, bordering countries, countries around the sea, bordering countries, landlocked countries and important cannels, landlocked countries and important cannels, straits etc straits etc 41. Key: C 42. Key: B Explanation: Ghana and Dutch Guiana are Explanation: The total area of United Kingdom different. is 94, 226 sq miles (244, 044 sq km), where Some of the country’s names and their earlier England covers 50, 344 sq miles (130, 365 sq names km) i.e. less than 60 % of total land area of New Names Old Names United Kingdom. Great Britain donot comprise Bangladesh East Pakistan of Northern Ireland. Beijing Peking

www.laexias.com Page No. 66 https://elearn.laex.in Geography UPSC Previous Year Questions

Educational Objective: To know about the UK and Rhoda Island is the smallest state in countries around the sea, bordering countries, USA. andlocked countries and important cannels, Educational Objective: To know about the straits etc countries around the sea, bordering countries, 43. Key: B landlocked countries and important cannels, Explanation: It runs through Yunan province, straits etc China, forms the border between Myanmar and 46. Key: C Laos and most of the border between Laos and Explanation: Thailand and flows across Cambodia and A. Black Sea 1. Bulgaria Southern Vietnam before emptying to South B. Red Sea 2. Eritrea China. C. Yellow Sea 3. China Educational Objective: To know about the D. Caspian Sea 4. Kazakhstan countries around the sea, bordering countries, Educational Objective: To know about the landlocked countries and important cannels, countries around the sea, bordering countries, straits etc landlocked countries and important cannels, 44. Key: D straits etc Explanation: Latvia does not share its 47. Key: D border with Poland because it is located Explanation: Panama shares its land between Estonia, Russia, Belarus, Lithuania, boundaries with two of Costa Gulf of Riga and Baltic Sea. Rica and Colombia. Eastern side Caribean sea and west side Pacific ocean

Educational Objective: To know about the To know about the countries around the sea, bordering countries, Educational Objective: countries around the sea, bordering countries, landlocked countries and important cannels, landlocked countries and important cannels, straits etc straits etc 45. Key: B 48. Key: D Explanation: British Columbia is western most The ' provinces of Canada, Bavaria is a state of Explanation: waterfall Victoria' is Zambeji. Germany. Gibralter is the overseas territory of associated with the river Victoria Falls, spectacular waterfall located about

www.laexias.com Page No. 67 https://elearn.laex.in Geography UPSC Previous Year Questions

midway along the course of the Zambezi River, Educational Objective: To know about the at the border between Zambia to the north and countries around the sea, bordering countries, Zimbabwe to the south. landlocked countries and important cannels, Educational Objective: To know about the straits etc countries around the sea, bordering countries, 51. Key: A landlocked countries and important cannels, Explanation: Bolivia is a land locked country, in straits etc South America the surrounding countries are 49. Key: A Peru, Chile, Argentina, Brezil, and Paruguay Explanation: The Caspian Sea is a landlocked body of water between Europe and Asia. Five countries Russia, Kazakhstan, Turkmenistan, Iran and Azerbaijan border the inland sea

Educational Objective: To know about the countries around the sea, bordering countries, landlocked countries and important cannels, straits etc

50. Key: A Educational Objective: To know about the Explanation: Israel has borders with four countries around the sea, bordering countries, neighbouring countries: Lebanon on the landlocked countries and important cannels, north, Syria and Jordan on the east side, straits etc and Egypt on the South side. 52. Key: D Explanation: In the given figure, four islands of Indian ocean region are Chagos, Socotra, Sechyells and Mouritus. Educational Objective: To know about islands of Indian ocean.

www.laexias.com Page No. 68 https://elearn.laex.in Geography UPSC Previous Year Questions

53. Key: B and Uganda. It is the largest lake in Africa and Explanation: Bosnia, Hercegovina, Albania, second widest fresh water body in the Croatia have Adriatic sea as a boundary. world. Lake Victoria is one of the African Macedonia is far from Adriatic sea. Great Lakes. Educational Objective: To know about the Educational Objective: To know about the countries around the sea, bordering countries, countries around the sea, bordering countries, landlocked countries and important cannels, landlocked countries and important cannels, straits etc straits etc

54. Key: A 58. Key: B Explanation: Explanation: Assertion is correct, but reason is 1. Ashkhabad not the correct explanation of assertion but the 2. Tashkent statement is true. In Australia farmers rear 3. Bishkek cattle for meat export, and most of the 4. Dushanbe Australian people are non-vegetarian. Educational Objective: To know about the Educational Objective: To know about the countries around the sea, bordering countries, countries around the sea, bordering countries, landlocked countries and important cannels, landlocked countries and important cannels, straits etc straits etc

55. Key: C 59. Key: D Explanation: In the given figure of Europe the C Explanation: marked city is Brussels, where historic treaty The basin is a part of the Ganges- between NATO and warsaw pact countries was Brahmaputra basin draining 1,086,000 square signed in 1998. kilometres in Tibet, Nepal, India and Bangladesh. Educational Objective: To know about Brussels. To the north, the Himalaya or lower parallel ranges beyond form the Ganges-Brahmaputra 56. Key: B divide. On the west the borders Explanation: The Port of Santos is located in the Indus basin and then the Aravalli ridge. the city of Santos, state of São Paulo, Brazil. It The Basin is one of the most is the busiest container port in . important water systems in the world. From its Educational Objective: To know about the headwaters on the , the Indus countries around the sea, bordering countries, River flows for 3,200 km across northern India landlocked countries and important cannels, and the length of Pakistan before emptying into straits etc the .The second largest river in 57. Key: C South America, the Parana River, originates at Explanation: Lake Victoria acts as the confluence of the Grande and Paranaiba the international boundary between Tanzania rivers in South Eastern Brazil.

www.laexias.com Page No. 69 https://elearn.laex.in Geography UPSC Previous Year Questions

The Zambezi River rises in the Kalene hills in 63. Key: B north-western Zambia and flows northwards for Explanation: Pseudotsuga menziesii is an about 30 km. It then turns west and south to evergreen conifer species in the pine family, run over about 280 km through Angola and Pinaceae. It is native to western North America reenters Zambia with an annual discharge of and is known as Douglas fir nearly 18 km3. It then flows southwards Mahogany is a straight-grained, reddish-brown through marshy plains. timber of three tropical hardwood species of the To learn about the Educational Objective: genus Swietenia, indigenous to the drainage basins of the rivers. Teak is a tropical hardwood tree species placed 60. Key: B in the flowering plant family Lamiaceae. Some Explanation: Maracaibo is the oil-port of forms of teak are known as Burmese teak, Venezuela. Maracaibo is a city in northwestern Central Province teak as well as Nagpur teak. Venezuela and the capital of Zulia state, known Educational Objective: To learn about the type as the center of Venezuela’s oil industry. Its old of timber and the country to which timber town is marked by colonial buildings, including belongs to. the colorful houses lining Calle Carabobo. 64. Key: A Educational Objective: To learn about the oil Explanation: Andes: The Andes Mountain port and their locations. ranges South America are the longest 61. Key: A continental mountain range in the world. They Explanation: Assertion and reason both are form a continuous highland along the western correct and reason is correct explanation of edge of South America. assertion. Chile is the largest copper producer in Brazilian Shield: Brazilian Shield is a part of the world. 35% of total global production is Amazonian Craton, and it is found in north produced by Chile and the major deposit is western part of South America. poryphyry copper. Guyana Highlands: Guiana Highlands or Educational Objective: To learn about the plateau is a region in South America located largest copper producer Chile and its major north of the Amazon river and south of the deposit. Orinoco River. The Amazon basin is drained by 62. Key: A Amazon Basin: the Amazon River and its tributaries. It'sdrinage Explanation: ‘A’ marked city in map is Darwin basin is located in the countries of Bolivia, Brazil, in Australia. ‘B’ marked city in the given map is Colombia, Ecuador, Guyana, Peru, Suriname Kualalumpur in Malayasia. ‘C’ marked city in and Venezuela. The basin is covered by the map is Nairobi in Kenya. The city marked ‘D’ in evergreen rainforest called , the map is Lagos in Nigeria. also known as Amazonia. Educational Objective: To learn about the map location of major cities in the world.

www.laexias.com Page No. 70 https://elearn.laex.in Geography UPSC Previous Year Questions

Educational Objective: To learn about the map Educational Objective: To learn about locations of mountains, basin and highlands. countries bordering Mediterranean sea.

65. Answer: A Explanation: Bratislava is the capital of Slovakia hence, (a) is not correctly matched. The capital of Slovenia is Ljublijana. Educational Objective: To know about the countries around the sea, bordering countries, landlocked countries and important cannels, straits etc. 66. Key: B Explanation: The following countries have a coastline on the Mediterranean Sea: Northern shore (from west to east): Spain, France, Monaco, Italy, Slovenia, Croatia, Bosnia and Herzegovina, Montenegro, Albania, Greece and Turkey. Eastern shore (from north to south): Turkey, Syria, Lebanon, Israel. Southern shore (from west to east): Morocco, Algeria, Tunisia, Libya, Egypt. Island nations: Malta, Cyprus.

www.laexias.com Page No. 71 https://elearn.laex.in Geography UPSC Previous Year Questions

www.laexias.com Page No. 72 https://elearn.laex.in Geography UPSC Previous Year Questions

II. Indian Geography (a) Bhavnagar (b) Bheemunipatnam 1. Indian Physiography (c) Chandipur (d) 1. Consider the following statements: 1) The Barren Island volcano is an active 4. Consider the following pairs: volcano located in the Indian territory. Famous Place Region 2) Barren Island lies about 140 km east of 1) Bodhgaya Baghelkhand Great Nicobar. 2) Khajuraho Bundelkhand 3) The last time the Barren Island volcano 3) Shirdi Vidarbha erupted was in 1991 and it has remained 4) Nasik (Nashik) inactive since then. 5) Tirupati Rayalaseema Which of the statements given above is/are Which of the pairs given above are correctly correct? matched? (a) 1 only (a) 1,2& 4 (b) 2 & 3 (b) 2,3,4& 5 (c) 3 only (c) 2 & 5 only (d) 1 & 3 (d) 1,3,4& 5

2. Consider the following statements: 5. Consider the following pairs: 1) In India, the Himalayas are spread over five Place of Pilgrimage Location States only 1) Srisailam Nallamala Hills 2) Western Ghats are spread over five States 2) Omkareshwar Satmala Hills only. 3) Pushkar Mahadeo Hills 3) Pulicat Lake is spread over two States only. Which of the above pairs is/are correctly Which of the statements given above is/are matched? correct? (a) 1 only (a) 1 & 2 only (b) 2 & 3 only (b) 3 only (c) 1 & 3 only (c) 2 & 3 only (d) 1,2& 3 (d) 1 & 3 only 6. Which one of the following pairs of 3. At one of the places in India, if you stand Islands is separated from each other by on the seashore and watch the sea, you the “Ten degree channel”? will find that the sea water recedes from (a) Andaman & Nicobar the shore line a few kilometres and (b) Nicobar & Sumatra comes back to the shore, twice a day, (c) Maldives & and you can actually walk on the sea (d) Sumatra & Java floor when the water recedes. This unique phenomenon is seen at

www.laexias.com Page No. 73 https://elearn.laex.in Geography UPSC Previous Year Questions

7. Consider the following pairs: (a) 1 only Hills Region (b) 1 & 3 only 1) (c) 2 & 3 only 2) Kaimur Hills Coast (d) 1,2& 3 3) Mahadeo Hills Central India 10. The latitudes that pas through 4) Mikir Hills North- also pass through Which of the above pairs are correctly (a) Rajasthan matched? (b) (a) 1 & 2 (c) (b) 2 & 3 (d) Jammu & (c) 3 & 4 11. In India which one of the following states (d) 2 & 4 has the largest inland saline wetland? 8. A state in India has the following (a) characteristics: (b) Haryana 1) Its northern part is arid and semi-arid. (c) 2) Its central part produces . (d) Rajasthan 3) Cultivation of cash crops is predominant 12. Which of the following Hills are found over food crops where the & the Western Which one of the following states has all of Ghats meet? the above characteristics? (a) (a) (b) Cardamom Hills (b) Gujarat (c) Nilgiri Hills (c) (d) Shevoroy Hills (d) 13. Which of the following pairs are correctly 9. If there were no Himalayan ranges, what matched? would have been the most likely Waterfalls River geographical impact on India? 1) Kapildhara Falls Godavari 1) Much of the country would experience the 2) cold waves from Siberia 3) Sivasamudram Falls Cauvery 2) Indo-gangetic plain would be devoid of such Select the correct answer using the code extensive alluvial soils given below 3) The pattern of monsoon would be different Codes: from what it is at present (a) 1 & 2 only Which of the statements given above is/ are (b) 2 & 3 only correct? (c) 1 & 3 only (d) 1,2& 3

www.laexias.com Page No. 74 https://elearn.laex.in Geography UPSC Previous Year Questions

Nallamalai Hills 14. In India, how many states share the coast line? (c) Nallamalai Hills-Javadi Hills- Nilgiri Hills (a) 7 Anaimalai Hills (b) 8 (d) Anaimalai Hills - Nilgiri Hills - Javadi Hills (c) 9 Nallamalai Hills (d) 10 19. Which one of the following statement is

15. Where are located? not correct? (a) Andhra Pradesh (a) The Western Ghats are relatively higher in their northern region (b) Karnataka (b) The Anai Mudi is the highest peak in the (c) Western Ghats (d) Tamil Nadu (c) Tapi river lies to the south of Satpura 16. In which State is the Guru Shikhar Peak (d) The Narmada & the Tapi river valleys are located? said to be old rift valleys. (a) Rajasthan 20. Consider the following: (b) Gujarat 1) Mahadeo Hills (c) Madhya Pradesh 2) Sahyadri Parvat (d) Maharashtra 3) Satpura Range What is the correct sequence of the above 17. Match List-I (Valley) with List-II (State) from the north to the south? and select the correct answer using the (a) 1-2-3 code given below the lists: (b) 2-1-3 List-I List-II (State) (c) 1-3-2 A. Markha Valley 1. Sikkim B. Dzukou Valley 2. Himachal Pradesh (d) 2-3-1 C. Sangla Valley 3. Jammu & Kashmir 21. Which one of the following statements is D. Yumthang Valley 4. not correct? Codes: (a) Gulf with narrow fronts and wider rears (a) A-2; B-4; C-3; D-1 experience high tides (b) A-3; B-1; C-2; D-4 (b) Tidal currents take place when a gulf is (c) A-2; B-1; C-3; D-4 connected with the open sea by a narrow (d) A-3; B-4; C-2; D-1 channel 18. Which one of the following is the correct (c) Tidal bore occurs when a tide enters the sequence of the given hills starting from narrow and shallow estuary of a river the north and going towards the south? (d) The tidal nature of the mouth of the river (a) Nallamalai Hills- Nilgiri Hills-Javadi Hills Hooghly is of crucial importance to Kolkata Anaimalai Hills as port (b) Anaimalai Hills- Javadi Hills- Nilgiri Hills

www.laexias.com Page No. 75 https://elearn.laex.in Geography UPSC Previous Year Questions

22. Match List-I (Beaches in India) with List- 26. The approximate age of the Aravallis II (States) and select the correct answer range is using the codes given below the Lists: (a) 370 million years List-I List-II (b) 470 million years A. Gopnath Beach 1.Andhra Pradesh (c) 570 million years B. Lawsons Bay Beach 2.Kerala (d) 670 million years C. Devbagh Beach 3.Gujarat 27. Assertion: Ganga Plain is in the most D. Sinquerim Beach 4. densely populated part of India 5.Karnataka Reasoning: Ganga is the most harnessed Codes: river of India. (a) A-5; B-4; C-2; D-1 (a) Both A and R are true and R is the correct (b) A-3; B-1; C-5; D-4 explanation of A. (c) A-5; B-1; C-2; D-4 (b) Both A and R are true but R is not the (d) A-3; B-4; C-5; D-1 correct explanation of A. 23. Nanda Devi peak forms a part of (c) A is true but R is false. (a) Himalayas (d) A is false but R is true. (b) Kumaon Himalayas 28. Assertion: The frequency of floods in North (c) Nepal Himalayas India plains has increased during the last (d) Punjab Himalayas couple of decades. 24. The sea coast of which one of the Reason: There has been reduction in the following states has become as a nesting depth of river valleys due to deposition of place for the giant Olive Ridley turtles silt. from South America? (a) Both A and R are true and R is the correct (a) Goa explanation of A. (b) Gujarat (b) Both A and R are true but R is not the (c) Orissa correct explanation of A. (d) Tamil Nadu (c) A is true but R is false. (d) A is false but R is true. 25. Which one of the following is not a lagoon 29. Some people in live in houses (a) Ashtamudi Lake built on floating islands of weeds and (b) Chilka Lake decaying vegetation held together by (c) Periyar Lake suspended slit. These islands are called: (d) Pulicat Lake (a) Tipis (b) Barkhans (c) Phumdi (d) Izba

www.laexias.com Page No. 76 https://elearn.laex.in Geography UPSC Previous Year Questions

30. Among the following cities, which one Which one among the following States has all lies on a longitude closest to that of the above characteristics? Delhi? (a) Arunachal Pradesh (a) Bengaluru (b) Assam (b) Hyderabad (c) Himachal Pradesh (c) Nagpur (d) Uttarakhand (d) Pune 35. The Stilwell Road, built in 1940s, which 31. Which of the following is geographically was recently in news, connects which of closest to Great Nicobar? the following? (a) Sumatra (b) (a) Agartala in India and Yangon in Myanmar (c) Java via Bangladesh (d) Sri Lanka (b) Ledo in India and Kunming in China via

32. If you travel by road from Kohima to Myanmar Kottayam, what is the minimum number (c) Kalimpong in India and Lhasa in Tibet via of States within India through which you Bhutan can travel, including the origin & the (d) Imphal in India and Bangkok in Thailand destination? via Myanmar (a) 6 36. Out of the four southern States: Andhra (b) 7 Pradesh, Karnataka, Kerala and Tamil (c) 8 Nadu, which shares boundaries with the (d) 9 maximum number of Indian States? (this

33. Which one of the following pairs of States question was asked before the formation of India indicates the easternmost & of . Hence answer has been westernmost State? changed to Karnataka only) (a) Assam and Rajasthan (a) Andhra Pradesh only (b) Arunachal Pradesh and Rajasthan (b) Karnataka only (c) Assam and Gujarat (c) Each of Andhra Pradesh and Karnataka (d) Arunachal Pradesh and Gujarat (d) Each of Tamil Nadu and Kerala

34. A particular State in India has the 37. Consider the following statements: following characteristics: 1) Sikkim has the minimum area among the 1) It is located on the same latitude which 28 Indian States (Delhi and Pondicherry not passes through northern Rajasthan. included) 2) It has over 80% of its area under forest 2) Chandigarh has the highest literacy rate cover. among Pondicherry, NCT of Delhi and other 3) Over 12% of forest cover constituted UnionTerritories. Protected Area Network in this State. 3) Maharashtra has the highest population

www.laexias.com Page No. 77 https://elearn.laex.in Geography UPSC Previous Year Questions

after Uttar Pradesh among the 28 Indian (a) States (Delhi and Pondicherry not included). (b) Delhi Which of the statements given above is/are (c) Jodhpur correct? (d) Nagpur (a) 1 &2 41. Consider the following statements: (b) 2 &3 1) Longitude of Jabalpur's location is between (c) 1 only those of and Bhopal (d) 3 only 2) Latitude of Aurangabad's location is between those of Vadodara and Pune 38. Consider the following statements: 3) Bangalore is situated more southward than 1) Assam shares a border with Bhutan and Chennai Bangladesh. Which of these statements is/are correct? 2) West shares a border with Bhutan (a) 1 & 3 and Nepal. (b) 2 only 3) shares a border with Bangladesh (c) 2 & 3 and Myanmar. (d) 1,2& 3 Which of the statements given above are 42. Which one among the following states is correct? smallest in area? (a) 1,2 and 3 (a) Andhra Pradesh (b) 2 and 3 (b) Gujarat (c) 2 and 3 only (c) Karnataka (d) 3 only (d) Tamil Nadu 39. Which of the following States border 43. Among the following cities, which one is Uttar Pradesh? nearest to the Tropic of Cancer? 1) Punjab (a) Delhi 2) Rajasthan (b) Kolkata 3) (c) Jodhpur 4) (d) Nagpur Select the correct answer using the code 44. Match List I with List II and select the given below: correct answer using the codes given (a) 1, 2, 3 and 4 below the lists: (b) 2, 3 and 4 List-I (Resorts) List-II (States) (c) 1 and 4 A. Chakrata 1. Assam (d) 1 and 3 B. Haflong 2. 40. Among the following cities, which one is C. Kalimpong 3. Uttar Pradesh at the highest altitude above mean sea D. Kufri 4. Himachal Pradesh level?

www.laexias.com Page No. 78 https://elearn.laex.in Geography UPSC Previous Year Questions

Codes: (c) Lucknow (a) A-1; B-3; C-2; D-4 (d) Bengaluru (Bangalore) (b) A-3; B-1; C-4; D-2 48. For which one of the following items, is (c) A-3; B-1; C-2; D-4 Tirupur well-known as a huge exporter to (d) A-1; B-3; C-4; D-2 many parts of the world. 45. In the rough outline map of a part of (a) Gems and Jewellery Jammu and Kashmir shown in the figure, (b) Leather goods places marked, A, B, C and D represent (c) Knitted garments respectively. (d) Handicrafts

49. Which one of the following cities does not have the same clock time as that of the other three cities at any given instant? (a) London (UK) (b) Lisbon (Portugal) (c) Accra (Ghana) (d) Addis Ababa (Ethiopia) (a) Anantnag, Baramula, & 50. Which two countries follow China and (b) Baramula, Srinagar, Kargil & Anantnag India in the decreasing order of their (c) Baramula, Srinagar, Anantnag & Kargil ? (d) Srinagar, Baramula, Kargil & Anantnag (a) Brazil and USA (b) USA and Indonesia 46. The above map is the of (c) Canada and Malaysia (d) Russia and Nigeria

1. Key and Explanation

1. Key: A Explanation: Barren Island is an island located (a) Chandigarh in the Andaman Sea. This volcanic island stands (b) Daman and Diu in the midst of a volcanic belt on the edge of the (c) Dadra and Nagar Haveli Indian and Burmese tectonic plates. (d) Pondicherry It is the only confirmed active volcano in and the only active volcano along a chain of 47. Which one among the following major volcanoes from Sumatra to Myanmar. Thus Indian cities is most eastward located? statement 1 is correct. (a) Hyderabad (b) Bhopal

www.laexias.com Page No. 79 https://elearn.laex.in Geography UPSC Previous Year Questions

It is a part of the Indian Union Territory of Educational Objective: To learn about the Andaman and , and lies about and spread of important 138 km northeast of the territory's capital, Port features like mountains, lakes and ghats. Blair. Thus statement 2 is incorrect. 3. Key: C The first recorded eruption of the volcano dates Explanation: Chandipur, is a beach in Baleswar back to 1787.After nearly one and half centuries District, , India. The beach is located on of dormancy, the island had another eruption in the shore of 1991 that lasted six months and caused The beach is unique in a way that the water considerable damage. recedes up to 5 kilometers during the ebb tide. The 1991 eruption was particularly harmful to Due to its unique circumstances, the beach the island's fauna. A team from the Geological supports biodiversity. Survey of India visited Barren Island on April Chandipur is also the location of the Indian 1993 to assess the impact of the eruption on the Strategic Forces Command's Integrated Test distribution, habit, and abundance of animal Range (ITR) at Abdul Kalam Island, formerly species. A team from the National Institute of known as Wheeler Island. Oceanography spotted the volcano erupting on Educational Objective: To learn about the 23 January 2017 important unique feature of Chandipur beach Educational Objective: To learn about the and its location. Barren island volcano and its location. 4. Key: C 2. Key: B Explanation: Bodh Gaya is a religious site and Explanation: The Indian Himalayan Region is place of pilgrimage associated with the spread over 10 states namely, Jammu & Mahabodhi Temple Complex in Gaya district in Kashmir, Himachal Pradesh, Uttarakhand, the Indian state of . It is famous as it is Sikkim, Arunachal Pradesh, , the place where Gautama Buddha is said to Nagaland, Manipur, Mizoram, and . Thus have attained Enlightenment (Pali: bodhi) under statement 1 is incorrect. what became known as the Bodhi Tree. The Western Ghats in the Indian peninsula is Mahabodhi Temple, located in Bodh Gaya is a one of the eight hotspots of biological diversity in UNESCO World Heritage Site. Bodhgaya does the world and is spread across six states Gujarat, not come under Baghelkhand region. Hence 1 is Maharashtra, Goa, Karnataka, Tamil Nadu and wrongly matched. Kerala. Thus statement 2 is incorrect. The Khajuraho Group of Monuments is a group Pulicat lake spread over two states, Andhra of Hindu temples and Jain temples in Pradesh and Tamil Nadu. Thus statement 3 is Chhatarpur district, Madhya Pradesh, India. correct. It is part of Bundelkhand Region. Hence option 2 is correct. They are a UNESCO World Heritage Site. The temples are famous for their

www.laexias.com Page No. 80 https://elearn.laex.in Geography UPSC Previous Year Questions

nagara-style architectural symbolism and their erotic sculptures. From the below maps it is clear that Shirdi is not part of vidarbha plateau

While the fifth pair Tirupathi and Rayalaseema are correct Educational Objective: To know the geographical locations of famous pilgrimage centres in India.

5. Key: A Explanation: Srisailam is a census town in Kurnool district of the Indian state of Andhra Pradesh. The town is famous for Mallikarjuna Jyotirlinga Temple and is one of the holy pilgrimage site for Saivism and Shaktism sects of . The town is classified as both Jyotirlinga and Shakti Peetha. It is located in the From the below two maps it is clear that nasik is Nallamala hills of Andra Pradesh. Hence option not part of malwa plateau. 1 is correct. Omkareshwar is a Hindu temple dedicated to God Shiva. It is one of the 12 revered Jyotirlinga shrines of Shiva. It is on an island called Mandhata or Shivapuri in the Narmada river, Madhya Pradesh. Hence it is not located on any hills. Option 2 is wrong. Pushkar is a city in the Ajmer district in the Indian state of Rajasthan. It is a pilgrimage site for and . Pushkar has many temples. Most of the temples and ghats in Pushkar are from the 18th century and later,

www.laexias.com Page No. 81 https://elearn.laex.in Geography UPSC Previous Year Questions

because many temples were destroyed during India. They are part Western ghats. Hence 1st Muslim conquests in the area. Pushkar is in pair is incorrect. centre-east part of Rajasthan, on the western Kaimur Range is the eastern portion of the side of Aravalli Mountains. Hence option 3 is Vindhya Range, extending from around Katangi wrong. in Jabalpur district of Madhya Pradesh to Educational Objective: To know the famous around Sasaram in of Bihar. temples like 12 jyotirlingas and their Hence 2ndpair is incorrect. Western coast geographical importance along Karnataka, Goa and Maharashtra is known as Konkan coast. 6. Key: A The Mahadeo Hills are a range of hills in Explanation: Madhya Pradesh state of central India. The hills are situated in the northern section of the Satpura Range Mikir Hills are a group of hills located to the south of the Kaziranga National Park, Assam. It is part of the Karbi Anglong Plateau. Educational Objective: To learn about the major hills and mountains along with their locations.

8. Key: B The Ten Degree Channel is a channel that Explanation: No effective barrier is there to separates the and Nicobar block Southwest (SW) monsoon wind. SW Islands from each other in the Bay of Bengal. monsoon wind flows parallel to Aravalli range. This channel is 150 kilometres (93 mi) wide from So Northern part of Gujarat is arid and semi- north to south, and approximately 10 kilometres arid. (6.2 mi) long from east to west. It is so named as Large part of northwest plateau covered with it lies on the 10-degree line of latitude, north of black lava soil which is very suitable for growing the equator. cotton. Central and south part of Gujarat To know about the Educational Objective: covered with this black soil. Gujarat is the top important islands in the territory of India and producer of groundnut and castor. More than their geographical importance. 30% of total cotton produced from India is 7. Key: C Gujarat. Gujarat plains lie east of Kachchh and Explanation: Cardamom hills are mountain Kathiawar. This plain is enough to support range of southern India and part of the southern agriculture. Western Ghats located in southeast Kerala Educational Objective: Gujarat state location, () and southwest Tamil Nadu in physical geography, Economic geography

www.laexias.com Page No. 82 https://elearn.laex.in Geography UPSC Previous Year Questions

9. Key: D From the figure it is visible that the latitude that Explanation: The Himalayas is the highest passes through Sikkim also passes through mountain range in the world, and has 9 out of Indian states of Rajasthan and Uttar Pradesh. 10 of the world’s highest peaks, including Mount Educational Objective: Learn about the Everest. These mountains, referred to as the latitudes and longitudes passing through the Third Pole, are the source of some of Asia’s important cities. major rivers and also help to regulate our 11. Key: D planet’s climate.They save us from cold waves Explanation: North of Luni River in Rajasthan from Siberia. there is inland drainage having several saline The most characteristic features of the lakes which is source of common salt. Some of Himalayas are their soaring heights, steep-sided them are Sambhar, Didhwana, Degana, jagged peaks, river valley and alpine glaciers. Kuchaman. The topography of Himalayas are deeply cut by The largest is Sambhar Lake which is near to erosion, resulting in deep river gorges and Jaipur city. deposition of such sediments in plains, Indo- Educational Objective: Rajasthan state location, gangetic plain would be devoid of such extensive physical geography alluvial soils without such erosion in upper region of Himalayas . 12. Key: C Complex geologic structure, and series of Explanation: The form part of elevational belts,compression and faults due to the Western Ghats in western Tamil Nadu of collision of Indian plate with Eurasian plate Southern India. It is the meeting point of resulted in mighty parallel mountain ranges Eastern and Western ghats. It is famous for which highly influences the monsoonal pattern. Nilikurinjiflower that blooms once in 12 years. Nilgiri hills is home for Today tribes. 10. Key: A Three national parks border portions of the Explanation: Nilgiri mountains. Mudumalai National Park Lies in the northern part of the range where Kerala, Karnataka, and Tamil Nadu meet. MukurthiNational Park lies in the southwest part of the range, in Kerala, which includes intact sholagrassland mosaic, habitat for the Nilgiritahr. Silent Valley National Park lies just to the south and contiguous with those two parks. Educational Objective: To learn about the Major hills ranges, their location, national parks and tribes associated with it.

www.laexias.com Page No. 83 https://elearn.laex.in Geography UPSC Previous Year Questions

13. Key: B 16. Key: A Explanation: Kapil Dhara Waterfall: River Explanation: Guru Shikhar Peak is located in Narmada plunge from a height of about 100 feet Rajasthan--- Aravalli range, which is the oldest to form Kapil Dhara Waterfalls. It is located in fold mountain in India. They are aligned in the Amarkantak district of Madhya Pradesh. the northeast to southwest direction. At that Waterfall gets its name from the famous sage southwest extreme the range rises to over Kapil. 1000m. Here mount Abu small hilly block is Jog Falls: Jog Falls is located in Siddapur in separated from the main range by the valley of Uttara Kannada district of Karnataka is created flowers. by the Sharavathi River. It is also known as Educational Objective: Places and location Gerusoppe falls. River Sharavathi, rises at 17. Key: D Ambutirtha in the Thirthahalli taluk shivamogga. Explanation: Markha Valley is in Union Shivanasamudra Falls: Territory. The , which flows is located in the District in the through the markha Valley, is a river in Ladakh, state of Karnataka the falls is on the river Kaver. India. It is a tributary of the River. Shivanasamudra Falls is one of the first hydro- Dzukou Valley is a valley located in borders of electric power stations in Asia the state of Nagaland and Manipur in northeast To learn about the Educational Objective : India. This valley is well known for its natural rivers and its waterfalls. environment, seasonal flowers and flora and 14. Key: C fauna. Explanation: In india 9 states and 4 union Sangla is a town in the Baspa Valley, also territories share the coastal line. India has coast referred to as the Sangla valley, in the Kinnaur line of 7516km. (6100km of mainland and District of Himachal Pradesh, India, close to the 1197km of island) Tibetan border. Maharashtra and Goa Konkan coast Yumthang Valley or Sikkim Valley of Flowers Kerala and Karnataka Malabar coast sanctuary, is a nature sanctuary with river, hot Tamilnadu Coromondal coast springs, yaks and grazing pasture on rolling Educational Objective: Places and location meadows surrounded by the Himalayan mountains in the North Sikkim district of 15. Key: D Sikkim state in India. Explanation: Shevaroy Hills, are a towering Educational Objective: Places and location mountain range (1620 m) near the city of Salem, in Tamil Nadu state, southern India. 18. Key: C Refer question no. 11 for map Explanation: Nallamala hills situated in Educational Objective: To learn about the Andhra Pradeshare a section of the Eastern major hills ranges, their location, national parks Ghats. They run in a nearly north-south and tribes associated with it. alignment, parallel to the Coromandel Coast for

www.laexias.com Page No. 84 https://elearn.laex.in Geography UPSC Previous Year Questions

close to 430 km between the rivers, Krishna and is spread across six states Gujarat, Maharashtra, Pennar. Goa, Karnataka, Tamil Nadu and Kerala. The Javadhu Hills ( Rainfall clouds producing The Western Ghats are relatively lower in their Hills For North Tamilnadu) ( Ooty of Vellore ) are northern region. The site’s high montane forest an extension of the Eastern Ghats spread ecosystems influence the Indian monsoon across parts of Vellore and Tiruvannamalai weather pattern. Moderating the tropical climate districts in the northern part of the state of of the region, the site presents one of the best Tamil Nadu in southeastern India. examples of the monsoon system on the planet. The Anai Mudi is the highest peak in the Western Ghats Narmada River: The Narmada River, also called by the name Rewa rises from Amarkantak Plateau. The river is known as "Life Line of Madhya Pradesh and Gujarat". Narmada is one of the only three major rivers in peninsular India that run from east to west. It is one of the rivers in India that flows in a rift valley (between the Satpura and Vindhya ranges). Geologically the Narmada Valley is a graben. It occupies a linear rift valley sloping towards west. Tapti: Tapti is another west flowing river of The Nilgiri Mountains form part of the Western Madhya Pradesh. The Ukai Dam also known as Ghats in western Tamil Nadu of Southern India. Vallabh Sagar is constructed across the Tapi The Anaimalai or Anamala Hills, also known as River is the second largest reservoir in Gujarat the Elephant Mountains, are the range of after the Sardar Sarovar. mountains that form the southern portion of Educational Objective: To learn about the the Western Ghats and span the border of Western Ghats and its highest peak. Also to Tamil Nadu and Kerala in Southern India. know about west flowing rivers. Educational Objective: To learn about the major hills ranges, their location, national parks 20. Key: C and tribes associated with it. The north- south Explanation: The Mahadeo Hills are a range of order and the east- west order of such hills in Madhya Pradesh state of central India. mountains are also important. The hills are situated in the northern section of the Satpura Range. 19. Key: A Satpura ranges are towards the south of Explanation: Western Ghat: The Western Mahadeo hills Ghats in the Indian peninsula is one of the eight hotspots of biological diversity in the world and

www.laexias.com Page No. 85 https://elearn.laex.in Geography UPSC Previous Year Questions

zones and breeding grounds of several forms of wildlife. Educational Objective: To learn about the tides and its tidal nature.

22. Key: B Explanation: Gopnath Beach is a beach situated in the Bhavnagar district of Gujarat state of India. It is located on the coast of the Gulf of Kambhat. Sahyadris is the other name for Western ghats, Lawsons Bay Beach is located in the city of that runs through6 states; Gujarat, Visakhapatnam state of Andhra Pradesh, India. Maharashtra, Goa, Karnataka, Kerala and Tamil Devbagh Beach is in (formerly Carwar) Nadu. which is a city in Uttara Kannada district in the Educational Objective: To learn about the Indian state of Karnataka. major hills ranges, their location, national parks Sinquerim Beach is located near Panaji in Goa. and tribes associated with it. The north- south Educational Objective: Places and location order and the east- west order of such mountains are also important. 23. Key: B Explanation: Nanda Devi is the second highest 21. Key: A mountain in India after Kangchenjunga and the Explanation: Tides are the rise and fall of sea highest located entirely within the country.The levels caused by the combined effects of the peak forms a part of Kumaon Himalayas. gravitational forces exerted by the Moon and the TheNanda Devi National Park is established in Sun, and the rotation of the Earth. Gulfs with 1982. It is situated around the peak of Nanda narrow fronts and wider rears would experience Devi (7816 m) in the state of Uttarakhand in highest tides. (Example - Gulf of Mexico) Thus northern India. It was tagged as a World statement 1 is correct. Heritage Site by UNESCO in 1988. A tidal bore is a tidal phenomenon in which the leading edge of the incoming tide forms a wave (or waves) of water that travels up a river or narrow bay against the direction of the river or bay's current. Tidal bore occurs when a tide enters the narrow and shallow estuary of a river Bores occur in relatively few locations worldwide, usually in areas with a large tidal range. Tidal bores can be dangerous. On the other hand, tidal bore-affected estuaries are rich feeding

www.laexias.com Page No. 86 https://elearn.laex.in Geography UPSC Previous Year Questions

Educational Objective: To know about the Bengal. It has been listed as a UNESCO World important peaks and the mountain ranges in Heritage site. which they are situated. is known for migratory birds on the Indian sub-continent. In 1981, it was designated 24. Key: C as the first Indian wetland of international Explanation: The olive ridley sea turtle also importance under the Ramsar Convention. known commonly as the Pacific ridley sea turtle, Periyar Lake is located in is a species of turtle Periyar Lake: Thekkady, Kerala. The Periyar Lake was formed The species is the second smallest and most during the construction of the dam across the abundant of all sea turtles found in the world. Mullaperiyar river in the year 1895. The lake is The olive ridley turtle has a circumtropical the part of the Periyar Wildlife Sanctuary in distribution, living in tropical and warm waters Kerala. It is fed by river. of the Pacific and Indian OceansIn the Indian Pulicat Lagoon is the second Ocean, the majority of olive ridleys nest in two or Pulicat Lake: largest brackish water lagoon in India, after three large groups near Gahirmatha in Odisha. Chilika Lake. The two rivers which feed the Olive-ridleys face serious threats across their lagoon are the Arani River at the southern tip migratory route, habitat and nesting beaches, and the Kalangi River from the northwest. The due to human activities such as turtle lake and its river basins are spread across the unfriendly fishing practices, development and Andhra Pradesh and Tamil Nadu states. The exploitation of nesting beaches for ports, and lake also includes the Pulicat Lake Bird tourist centres. Sanctuary. The Buckingham Canal, a navigation Educational Objective: To learn about the olive channel, is part of the. ridley turtles and its nesting locations. Educational Objective: To learn about the 25. Key: C lagoons and their locations. Explanation: Ashtamudi Lake: Ashtamudi 26. Key: D Lake is located in the Kollam District in the state Explanation: Aravalli Range is an eroded stub of of Kerala. For the conservation and sustainable ancient mountains, is the oldest range of Fold utilization of wetlands the Ashtamudi Wetland Mountains in northern India (670 million years), was included in the Ramsar Convention. Kallada running northeasterly for 350 miles (560 km) River is a major river discharging into the through Rajasthan state. Ashtamudi Lake. The Aravalli Range starts near Delhi, passing Chilika Lake: Chilika Lake is a brackish water through southern Haryana and Rajasthan, and lagoon, It is the largest coastal lagoon in India ending in Gujarat. and the second largest brackish water lagoon in Mount Guru Shikhar, the highest peak in the the world situated in Odisha. The lake is at the Aravalli Range. mouth of the , flowing into the Bay of

www.laexias.com Page No. 87 https://elearn.laex.in Geography UPSC Previous Year Questions

Educational Objective: To learn about the fresh water in and it is famous Aravalli range and its spread. for phumdis floating over it. Keibul lamjao National park is the only floating national park 27. Key: C in the world. The park is famous for endangered Explanation: The Ganga Plain of northern India Sangai— Manipur state animal. It plays an region is drained by Ganga and many other important role for Manipur economy by prominent rivers. Which has mainly made of generating hydropower, irrigation, drinking fertile alluvial soil which has resulted in water supply, and fishing. This lake is increased agricultural activities. designated as wetland of international The fertile soil, presence of many rivers, importance under Ramsar Convention and favourable climate and the availability of flat Montreux record. terrain have made this region one of the most Educational Objective: Manipur location and densely populated areas of the world. Thus economic geography of India and plant statement 1 is correct. vegetation The river in southern India is the most harnessed river in India not the river Ganga 30. Key: A thus statement 2 is incorrect. Explanation: From the map it is clear that the Educational Objective: To learn about the city that lies on a longitude closest to that of Ganga river and the reasons why Ganga region Delhi is Bangalore. has got densely population. Delhi – 77.08” E longitude

28. Key: A Explanation: In The north Indian rivers systems such as Indus and Ganga, the rivers carry huge volume of water for a very long distance. Due to the excessive siltation in the river course there has been reduction in the depth of river valleys. As result of shallow river plains the north Indian rivers are very prone to flash floods. Especially in the year of excessive rainfall these rivers overflow above their banks and creat huge economic loss. Bangalore – 77.56” E longitude Educational Objective: To understand why Hyderabad – 78.27” E longitude north Indian rivers are very prone to flash floods. Nagpur – 79.10” E longitude Pune – 73.86” E longitude 29. Key: C Learn about the Explanation: Phumdi—Heterogeneous mass of Educational Objective: vegetation, soil, and organic matter at various latitudes and longitudes passing through the stages of decomposition. Loktak lake largest important cities.

www.laexias.com Page No. 88 https://elearn.laex.in Geography UPSC Previous Year Questions

33. Key: D 31. Key: A Explanation: Vijaynagar in Changlang district of Explanation: Sumatra is the western most Arunachal Pradesh is the eastern-most island among the Indonesian islands. It is the inhabited locality of the country. The famous largest island that is entirely governed by Namdapha National Park lies at the Indonesia and the sixth-largest island in the international border between India and world. It is nearest to the Great Nicobar. Myanmar in the state of Arunachal Pradesh. The westernmost point of India is the small inhabited village of Ghuar Moti, located in the Kutch District of Gujarat Thus Arunachal Pradesh and Gujarat are the eastern most and western most states of the county respectively. Educational Objective: To learn about the states and its locations in Indian map. Educational Objective: To know about the neighbouring countries of India along with their 34. Key: A location. Explanation: Arunachal pradesh is located on the same latitude which passes through 32. Key: B northern Rajasthan. It has over 80% of its area Explanation: To travel by road from Kohima to under forest cover. Kottayam one needs to cross Nagaland (origin), The state has also got vast network of Protected Assam, WB, Odisha, Andhra Pradesh and then Areas. either through Tamilnadu to Kerala (Kottayam), Educational Objective: States and location. or through AP to Karnataka to Kerala. In either way a person needs to travel through 7 states. 35. Key: B Educational Objective: To learn about the Explanation: Stilwell Road, original name Ledo states and its locations in Indian map. Road, highway 478 mi (769 km) long that links northeastern India with the Burma Road , which runs from Burma to China (Ledo in India and Kunming in China via Myanmar). During World War II the Stilwell Road was a strategic military route. U.S. Army engineers began construction of the highway in December 1942 to link the railheads of Ledo (Assam, now in Arunachal Pradesh, India) and Mogaung (Burma), and Chinese troops later aided in the project.

www.laexias.com Page No. 89 https://elearn.laex.in Geography UPSC Previous Year Questions

Initially called the Ledo Road, it was named after Uttar Pradesh — 19.09crore the American general Joseph Stilwell who Maharashtra — 11.2crore oversaw its construction. Bihar — 10.4crore Educational Objective: To learn about the West Bengal — 9.10crore Stilwell road and its strategic importance. Educational Objective: Economic geography of 36. Key: B India. Explanation: 38. Key: A Explanation: Assam has land frontier with Bhutan and Bangladesh

Kerala being the southernmost state, shares borders with only Karnataka and Tamil Nadu. Tamil Nadu shares borders with Karnataka, West Bengal bordered with Bangladesh and Kerala and Andhra Pradesh. Nepal. Andhra Pradesh is bordered by Tamil Nadu, Mizoram has border with Bangladesh and Karnataka, Telangana and Odisha. Myanmar. Karnataka shares borders with Tamil Nadu, Other Northeastern states that shares border Kerala, Andhra Pradesh, Telangana, with foreign countries are; Arunachal Pradesh – Maharashtra and Goa. with China, Bhutan and Myanmar. Nagaland Educational Objective: To know about the and Manipur– with Myanmar. Tripura – with states and their bordering states. Bangladesh and Myanmar. To learn about the 37. Key: D Educational Objective: international frontiers of India and the states Explanation: that shares border with them. Goa area — 3702sqkm Sikkim — 7096sqkm 39. Key: B Tripura — 10,486sqkm Explanation: Punjab lies on northwestern part Nagaland — 16,579sqkm of India. Uttar Pradesh is situated in North According to census 2011, Kerala has highest Central India. Hence, only Rajasthan, literacy rate 93.91%. Chattisgarh, Delhi, Haryana, Jharkhand, Literacy rate Puducherry>>Chandigarh>>Delhi. Uttarakhand, Madhya Pradesh and Bihar Population According to census 2011 report: shares borders with UP.

www.laexias.com Page No. 90 https://elearn.laex.in Geography UPSC Previous Year Questions

Educational Objective: To know about the important cities, their longitude and the surrounding cities.

42. Key: D Explanation: Gujarat - 196,024sqkm Karnataka - 191,791sqkm

Educational Objective: To learn about the Andhra Pradesh - 162,970sqkm borders of each state. Tamil nadu - 130,058sqkm Educational Objective: Physical Geography of 40. Key: A India Explanation: Bangalore lies in the area and hence among the given options, 43. Key: B Bangalore is correct. 921m above sea level. Explanation: From the map it is visible that Delhi-216m above sea level; Jodhpur –395m among the given options, Kolkata is nearest to above sea level; Nagpur –310.5m above sea level. Tropic of Cancer. Educational Objective: To learn about the important feature of Indian cities.

41. Key: C Explanation: From the below map it is clear that the Jabalpur's longitude is not between the Bhopal and Indore. Hence statement 1 is wrong. From the below two maps it is clear that Aurangabad is between Pune and Vadodara. Hence 2 is correct. Educational Objective: To know in detail about And also the option 3 is correct, as Bengaluru is the Tropic of cancer and it’s relation with the more southward than Chennai important cities.

44. Key: C Explanation: The places mentioned are all famous hill stations Chakrata is a cantonment town in Dehradun district in the state of Uttarakhand, India. It is between the Tons and rivers, at an elevation of 2118 m.

www.laexias.com Page No. 91 https://elearn.laex.in Geography UPSC Previous Year Questions

Haflong is a town and headquarters of in the state of Assam in India. It is the only hill station in Assam. Haflong is a Dimasa word meaning ant hill. Kalimpong is an east Indian hill town in the Himalayan foothills of West Bengal. Perched on a ridge above the Teesta River, it’s home to colonial-era buildings. Kufri is a small hill station in Shimla district of Educational Objective: To learn about the Himachal Pradesh state in India. union territories, as new union territories have Educational Objective: Places and location been formed.

45. Key: C 47. Key: C Explanation: From the below map it is clear Explanation: Lucknow is the most eastward city that, point A is Baramula, point B is Srinagar, among Hyderabad, Bhopal, Lucknow and point C is Anantnag and Point D is Kargil. Bengaluru.

Educational Objective: To have idea about

geographical locations of major cities in India. Educational Objective: To know about major 46. Key: C Indian cities and their locations

Explanation: Dadra and Nagar Haveliis 48. Key: C composed of two separate geographical entities: Explanation: Tirupur located in Tamil Nadu is Nagar Haveli, wedged between Maharashtra and the garment hub that makes exports worth Rs Gujarat. The union territory was merged with, 26,000 crore annually of knitted wear products. Daman and Diu to form new union territory of Educational Objective: To know about Tirupur, Dadra and Nagar Haveli and Daman and Diu on Global exporter of knitted garments. 26 January 2020.

www.laexias.com Page No. 92 https://elearn.laex.in Geography UPSC Previous Year Questions

49. Key: D c. 1 and 3 only Explanation: The clock time of any location on d. 1, 2 and 3 the earth depends on the location of the place of 3. Consider the following pairs: longitude i.e., places located on same longitude Glacier River have same time and thus London, Lisbon and 1. Bandarpunch Yamuna Accra have a same longitude passing through 2. Bara Shigri Chenab them i.e., 51.5 N AND 0.1278 W. Thus they 3. Milam Mandakini have same clock time and not local time which 4. Siachen depends on country’s standard time. 5. Zemu Manas Educational Objective: To study latitudes and Which of the pairs given above are correctly longitudes and understand their importance in matched? finding the exact location of a place on earth. a. 1,2 and 4 50. Key: B b. 1,3 and 4 Explanation: USA(28,23,38,631) and c. 2 and 5 Indonesia(21,38,29,469) are the two countries d. 3 and 5 that follow China and India in the decreasing 4. What is common to the places known as order of their populations. Aliya, Isapur and Kangsabati? Educational Objective: The top ten countries a. Recently discovered deposits with the highest population. b. Tropical rain forests c. Underground cave systems 2. Rivers d. Water reservoirs

1. Which one among the following rivers is 5. Which one of the following is an artificial the longest lake? a. Amazon a. Kodaikanal (Tamil Nadu) b. b. Kolleru (Andhra Pradesh) c. Congo c. Nainital (Uttarakhand) d. d. Renuka (Himachal Pradesh)

2. Consider the following pairs: 6. With reference to river Teesta, consider Famous Place River the following statements: 1. Pandharpur Chandrabhaga 1. The source of river Teesta is the same as 2. Cauvery that of Brahmaputra but it flows through Sikkim. 3. Hampi Malaprabha 2. River Rangeet originates in Sikkim and it is Which of the pairs given above are correctly a tributary of river Teesta. matched? 3. River Teesta flows into Bay of Bengal on the a. 1 and 2 only border of India and Bangladesh. b. 2 and 3 only

www.laexias.com Page No. 93 https://elearn.laex.in Geography UPSC Previous Year Questions

Which of the following statements given Select the correct answer using the codes above is / are correct? given below. a. 1 & 3 only a. 1 only b. 2 only b. 2 & 3 c. 2 & 3 only c. 1 & 3 d. 1,2 & 3 d. None

7. Which of the following is/are tributary/ 10. When you travel in Himalayas, you will tributaries of Brahmaputra? see the following 1. Dibang 1. Deep gorges 2. Kameng 2. U-turn river courses 3. Lohit 3. Parallel mountain ranges Select the correct answer using the code 4. Steep gradients causing land-sliding given below: Which of the above can be said to be the a. 1 only evidences for Himalayas being young fold b. 2 & 3 only mountains? c. 1 & 3 only a. 1 & 2 only d. 1,2 & 3 b. 1, 2 & 4 only c. 3 & 4 only 8. Consider the following rivers: d. 1,2,3 & 4 1. Vamsadhara 2. Indravati 11. Two important rivers one with its source 3. Pranahita in Jharkhand (and known by a different 4. Pennar name in Odisha), and another, with its source in Odisha -merge at a place only a Which of the above are tributaries of short distance from the coast of Bay of Godavari? Bengal before flow into the sea. This is a. 1,2 & 3 an important site of wildlife and b. 2,3 & 4 biodiversity and a protected area. Which c. 1,2 & 4 one of following could be this? d. 2 & 3 only a. Bhitarkanika b. Chandipur-on-sea 9. The Narmada river flows to the west, c. Gopalpur-on-sea while most other large peninsular rivers d. Simlipal flow to the east. Why? 1. It occupies a linear rift valley. 12. Rivers that pass through Himachal 2. It flows between the Vindhyas and the Pradesh are Satpuras. a. Beas & Chenab only 3. The land slopes to the west from Central b. Beas &Ravi only India c. Chenab, Ravi & Satluj only

www.laexias.com Page No. 94 https://elearn.laex.in Geography UPSC Previous Year Questions

d. Beas, Chenab, Ravi , Satluj & Yamuna 2. There are no west flowing rivers in Madhya Pradesh. 13. With reference to the river Luni, which Which of the statements given above is/are one of the following statements is correct? a. It flows into correct? a. 1 only b. It flows into Gulf of Kuchchh b. 2 only c. It flows into Pakistan & merges with a c. Both 1 & 2 tributary of Indus d. Neither 1 nor 2 d. It is lost in the marshy land of the Rann of Kuchchh 18. Consider the following pairs:

14. Which one of the following pairs is not Tributary River Main River 1. Chambal Narmada correctly matched? 2. Sone Yamuna Dam/Lake River a. Govind Sagar Satluj 3. Manas Brahmaputra b. Kolleru Lake Krishna Which of the pairs given is/are correctly c. Ukaireservoir Tapi matched? d. Jhelum a. 1,2 & 3 b. 1 & 2 only c. 2 & 3 only 15. Which one of the following rivers does d. 3 only not originate in India? 19. Which of the following pairs are correctly a. Beas matched? b. Chenab Irrigation Project State c. Ravi 1. Damanganga Gujarat d. 2. Gima Maharashtra 16. At which one of the following places do 3. Pamba Kerala two important rivers of India originate; Select the correct answer using the code while one of them flows towards north and merges with another important given below rivers flowing towards Bay of Bengal, the Codes: other one flows towards Arabian Sea? a. 1 & 2 only a. Amarkantak b. 2 & 3 only b. Badrinath c. 1 & 3 only c. d. 1,2 & 3 d. Nasik 20. Assertion: River Kalinadi is an east-flowing 17. Consider the following statements: river in the southern part of India. 1. There are no east flowing rivers in Kerala.

www.laexias.com Page No. 95 https://elearn.laex.in Geography UPSC Previous Year Questions

Reason: The Deccan Plateau is higher along 24. Which one of the following statements is its western edge and gently slopes towards not correct the Bay of Bengal in the east. a. River rises in Chattisgarh a. Both A and R are true and R is the correct b. rises in Maharashtra explanation of A. c. Cauvery River rises in Andhra Pradesh b. Both A and R are true but R is not the d. rises in Madhya Pradesh correct explanation of A. 25. What is the correct sequence of the c. A is true but R is false. rivers Godavari, Mahanadi, Narmada and d. A is false but R is true. Tapi in the descending or their lengths? 21. Which one of the following rivers a. Godavari-Mahanadi-Narmada-Tapi originates at Amarkantak? b. Godavari-Narmada-Mahanadi-Tapi a. Damador c. Narmada-Godavari-Tapi-Mahanadi b. Mahanadi d. Narmada-Tapi-Godavari-Mahanadi c. Narmada 26. Assertion: West-flowing rivers of Peninsular d. Tapi India have no deltas. 22. Match List I with List II and select the Reason: These rivers do not carry any correct answer using the code alluvial sediments. List-I (Town) List-II (River Nearer to it) a. Both A and R are true and R is the correct A. Betul 1. Indravati explanation of A. B. Jagdalpur 2. Narmada b. Both A and R are true but R is not the C. Jabalpur 3. Shipra correct explanation of A. D. Ujjain 4. Tapti c. A is true but R is false. Codes: d. A is false but R is true. a. A-1; B-4; C-2; D-3 27. The correct sequence of the eastward b. A-4; B-1; C-2;D-3 flowing rivers of the penisular India from c. A-4; B-1; C-3; D-2 north to south is d. A-1; B-4; C-3; D-2 a. Subarnarekha, Mahanadi, Godavari, 23. From north towards south, which one of Krishna, Pennar, Cauvery and Vaigai the following is the correct sequence of b. Subarnarekha, Mahanadi, Krishna, the given rivers in India? Godavari, Cauvery, Vaigai and Pennar a. -Spiti-Zaskar-Satluj c. Mahanadi, Subarnarekha, Godavari, b. Shyok-Zaskar--Spiti-Satluj Krishna,Cauvery, Pennar and Vaigai c. Zaskar-Shyok-Satluj-Spiti d. Mahanadi, Subarnarekha, Krishna, d. Zaskar-Satluj-Shyok-Spiti Godavari, Cauvery, Vaigai and Pennar

www.laexias.com Page No. 96 https://elearn.laex.in Geography UPSC Previous Year Questions

28. Which one of the following statements is 32. Match List I with List II and select the not true? correct answer using the codes given a. Ghaggar's water is utilized in the Indira below the lists: Gandhi Canal List-I (Rivers) List-II (Dams) b. Narmada raised from Amarkantak region A. Cauvery 1. Alamatti c. Nizam Sagar is situated on the Manjra river B. Krishna 2. d. Penganga is a tributary of the Godavari C. Narmada 3. Gandhi Sagar D. Chambal 4. Sardar Sarovar 29. Which one of the following east flowing rivers of India has rift valley due to down Codes: a. A-1; B-4; C-2; D-3 warping a. Damodar b. A-2; B-1; C-4; D-3 b. Mahanadi c. A-2; B-1; C-3; D-4 c. Sone d. A-1; B-3; C-4; D-2 d. Yamuna

30. Lake Sambhar is nearest to which one of 2. Key and Explanation the following cities of Rajasthan? 1. Key: A a. Bharatpur Explanation: The Amazon River in South b. Jaipur America is the largest river by discharge volume c. Jodhpur of water in the world, covers about 40% of South d. Udaipur Americaand it is the second longest river in the world, after the River.The Amazon basin is

31. In the given figure, the site of the Tehri the largest drainage basin in the world, with an dam has been labelled as area of approximately 7,050,000 square kilometers. It finally discharges into the Atlantic Ocean.It drains from west to east, from Iquitos in Peru, across Brazil to the Atlantic.The Atlantic has sufficient wave and tidal energy to carry most of the Amazon's sediments out to sea, thus the Amazon does not form a true delta. Morethan one-third of all known species in the world live in the Amazon rainforest a. A Educational Objective: To learn about the b. B Amazon river and its basin. c. C d. D

www.laexias.com Page No. 97 https://elearn.laex.in Geography UPSC Previous Year Questions

2. Key: B It is located just northeast of the point NJ9842 Explanation: Pandharpu: Pandharpur in where the LOC () between India Solapur district of Maharashtra is a well known and Pakistan ends. The melting waters of pilgrimage town on the banks of River Bhima 's are the main source of the (Chandrabhaga). The principal deity is lord Nubra River. Vithoba temple. Zemu: Zemu Glacier is one of the largest glacier Tiruchirappalli: Tiruchirappalli also called in the Eastern Himalaya. The glacier is the Trichy (City of heaven), is a major city in the source of water for the Teesta River. state of Tamil Nadu, on the bank of river Educational Objective: To learn about the Cauvery. The Cauvery Delta begins west of the glaciers and rivers. city where the Kaveri river splits into two, 4. Key: D forming the island of . Explanation: Aliyar Reservoir: Aliyar Reservoir Hampi: Hampi is a small town in Ballari district, across the Aliyar river, located in Aliyar village Karnataka, on the banks of the Tungabhadra near Pollachi town in Coimbatore District of River. It was the capital of Tamil Nadu. The dam was constructed mainly in the 14th century. The Group of Monuments at for irrigation purposes. Hampi is recognised as UNESCO World Heritage Isapur Dam is an earthfill dam near Kalamnuri Site. in the state of Maharashtra. built across the Educational Objective: Famous places and River Penganga. rivers. Educational Objective: To learn about the 3. Key: A water reservoirs and their locations. Explanation: Bandarpunch: Bandarpunch 5. Key: A Glacier is located in the Garhwal division of the Explanation: Kodaikanal Lake: Kodai Lake, Himalayas, in the state of Uttarakhand. The also known as Kodaikanal Lake is one of the Bandarpunch glacier acts as major watershed manmade (Artifical) lake located in the for the headwaters of the Yamuna River, whose Kodaikanal city in Dindigul district in the state source lies above Yamnotri. of Tamil Nadu.The Palani Hills range to Bara Shigri: Bara Shigri located in the state of northwest of lake, is the main watershed for the Himachal Pradesh is one of the largest glacier in lake Himalayas. The glacier feeds the River Chenab. Kolleru: Kolleru Lake is one of the largest Milam: Milam Glacier is one of the major glacier freshwater natural lakes in India located in state of the Kumaon Himalaya, in the Pithoragarh of Andhra Pradesh and it is also the largest district of Uttarakhand. The glacier is the source shallow freshwater lake in Asia it is located of the River Goriganga. between Krishna and Godavari deltas. Siachen: The Siachen Glacier is the longest Nainital: Nainital Lake is one of the natural glacier in the range in the Himalayas. freshwater lake, located in the township of

www.laexias.com Page No. 98 https://elearn.laex.in Geography UPSC Previous Year Questions

Nainital in Uttarakhand, The lake istectonic in Meghna. It flows as Meghna river before origin. emptying into the Bay of Bengal. Renuka: Renuka lake is located in the Sirmaur The River Brahmaputra is a classic example of a district of Himachal. It is the largest lake in braided river channel. Himachal Pradesh. And It is designated as is a created by the Ramsar site since November 2005. Brahmaputra River in Assam and it became the Educational Objective: To learn about the first island to be made a district in India in 2016. artificial and natural lakes. Dibang, Kameng, Lohit, Manas River, , Teesta River and Kopili River are the 6. Key: B tributaries of Brahmaputra. Explanation: The Teesta River originates from To learn about the river the Pahunri (or Teesta Kangse) glacier in Educational Objective: Brahmaputra and its important tributaries. Himalaya's. It is one of the longest river that rises in the eastern Himalayas. 8. Key: D It flows through the Indian states of Sikkim and Explanation: The Godavari is east flowing West Bengal, and it forms the border between peninsular river originates in the Western Ghats these two states for some distance. at Triambakeshwar in Maharashtra. It is India's It joins the Jamuna River at Fulchhari in second longest river after the Ganga. Bangladesh and enters in to the Bay of Bengal. It has its drainage basin in the states of The Teesta River is known for changing its Maharashtra, Telangana, Andhra Pradesh, course. And the river has remained a subject of Chhattisgarh, Odisha, ultimately emptying into conflict for almost 35 years between India and the Bay of Bengal. Bangladesh, over the appropriate allocation of The Godavari Water Disputes Tribunal governs its water resource and development on the river. the water allocation from the river among the Educational Objective: To learn about the riparian states the Purna, Pranhita, Indravati Teesta river and its origin along with its course and Sabari River are the left bank tributaries of flow. whereas the Pravara, Manjira, Manair are the right bank tributaries. 7. Key: D Polavaram Project: in Andhra Pradesh a multi- Explanation: purpose National project on the Godavari River Explanation: The River Brahmaputra has its is under construction in the name of Polavaram origin in the Manasarovar Lake. It is a trans- Project in the West Godavari District and East boundary river which flows through China, India Godavari Districts. and Bangladesh. It is called Yarlung Tsangpo in To learn about the river Tibet and Dihang River in Arunachal Pradesh. Educational Objective: Godavari and its important tributaries. It merges with the Padma (River Ganges is popularly known by the name Padma in 9. Key: A Bangladesh) and from here it is called as

www.laexias.com Page No. 99 https://elearn.laex.in Geography UPSC Previous Year Questions

Explanation: The Narmada River, also called by Explanation: The Brahmani is a major seasonal the name Rewa rises from Amarkantak Plateau. river in the state of Odisha. The Brahmani is The river is known as "Life Line of Madhya formed by the confluence of the rivers South Pradesh and Gujarat". Koel and Sankh near the major industrial town Narmada is one of the only three major rivers in of . Whereas the Baitarani is one of six peninsular India that run from east to west. It is major rivers of Odisha. one of the rivers in India that flows in a rift Together with the river Baitarani, river valley (between the Satpura and Vindhya ranges). Brahmani forms a large delta before emptying Geologically the Narmada Valley is a graben. It into the Bay of Bengal at Dhamra. occupies a linear rift valley sloping towards west. Bhitarkanika National Park is a large national Sardar Sarovar Dam: The Sardar Sarovar Dam park in northeast district in Odisha. is a gravity dam built across Narmada river near The Bhitarkanika National Park is home to Navagam in Gujarat. Saltwater crocodile. Educational Objective: To learn about the The Bhitarkanika Mangroves is a mangrove Narmada river and understand its course of flow. wetland in Odisha, India, covering an area of 650 km in the Brahmani and 10. Key: D deltasIt was designated as Ramsar site. Explanation: The Himalayas is the highest Educational Objective: To learn about the mountain range in the world, and has 9 out of presence of river flow in the important sites such 10 of the world’s highest peaks, including Mount as National parks/wildlife sanctuaries. Everest. These mountains, referred to as the Third Pole, are the source of some of Asia’s 12. Key: D major rivers and also help to regulate our Explanation: Beas, Chenab, Ravi and Satluj planet’s climate. rivers are the tributaries of Indus river where as The most characteristic features of the the Yamuna is the tributary of Ganga. Himalayas are their soaring heights, steep-sided rises in the Himalayan ranges, in the jagged peaks, river valley and alpine glaciers. state Himachal Pradesh and merges to the Sutlej The topography of Himalayas are deeply cut by River in Punjab. Beas river is home to some erosion, resulting in deep river gorges, complex isolated population of the Indus dolphin geologic structure, and series of elevation belts, is one of the major transboundary compression and faults due to collision of Indian river that flows between India and Pakistan. The plate with Eurasian plate resulted in parallel river is formed by the confluence of two rivers, mountain ranges Chandra and BhagaIt rises in the upper Educational Objective: To learn about the Himalayas in the Lahaul and Spiti district of important characteristic features of young fold Himachal Pradesh. mountains. The Ravi is one of the major transboundary river that flows between India and Pakistan, under 11. Key: A

www.laexias.com Page No. 100 https://elearn.laex.in Geography UPSC Previous Year Questions

the Indus Water Treaty the waters of the Ravi Reservoir is formed by the Bhakra Dam on the are allocated to India River Sutlej. It is one of the world's highest The Sutlej River is one of the longest tributary of gravity Indus. The source of the Sutlej is west of Kolleru lake: Kolleru Lake is one of the largest catchment area of the Lake Rakshastal in Tibet. freshwater natural lakes in India located in state It enters Indian state of Himachal Pradesh of Andhra Pradesh and it is also the largest through the Shipki La pass under the Indus shallow freshwater lake in Asia.it is located Waters Treaty between India and Pakistan the between Krishna and Godavari deltas. waters of the Sutlej are allocated to India. Some The lake is fed directly by water from the of the major hydroelectric projects on the Sutlej, seasonal Budameru and Tammileru rivers, and including Bhakra Dam, Karcham Wangtoo it is also connected to the Krishna and Godavari Hydroelectric Plant, Ropar barrage, Nathpa irrigation systems by over 67 major and minor Jhakri Dam. irrigation canals. The Yamuna is the second largest tributary river Ukai Dam: The Ukai Dam also known as of the Ganges, Originating from the Yamunotri Vallabh Sagar is constructed across the Tapi Glacier River is the second largest reservoir in Gujarat Educational Objective: To learn about the after the Sardar Sarovar. rivers that passes through Himachal Pradesh. Wular Lake: Wular Lake in Bandipora district in Jammu and Kashmir is one of the largest fresh 13. Key: D water lakes in Asia. The lake basin was formed Explanation: Luni River originates in the as a result of tectonic activity and is fed by the Pushkar valley of the Aravalli Range, near Ajmer, River Jhelum, which is a tributary of Indus. Rajasthan. Luni is largest river which flows To learn about the river through the . Educational Objective: and its dams. Its major tributaries are the Sukri, Mithri River, Bandi, Khari, Jawai, Guhiya and Sagi from the 15. Key: D left, and the Jojari from the right. Explanation: Beas, Chenab, Ravi and Satluj The river flows in the southwest direction rivers are the tributaries of Indus River. through the hills and plains of the Marwar and Beas River: Beas River rises in the Himalayan enters the Thar Desert before dissipating into ranges, in the state Himachal Pradesh and the Rann of Kutch. merges to the Sutlej River in Punjab. Beas river Educational Objective: To learn about the river is home to some isolated population of the Indus Luni and its origin and flow. dolphin Chenab River: Chenab River is one of the major 14. Key: B transboundary river that flows between India Explanation: Gobind Sagar Lake: Gobind Sagar and Pakistan. The river is formed by the Lake is a Reservoir located in the Una and confluence of two rivers, Chandra and Bhaga. It Bilaspur Districts of Himachal Pradesh. The

www.laexias.com Page No. 101 https://elearn.laex.in Geography UPSC Previous Year Questions

rises in the upper Himalayas in the Lahaul and Educational Objective: To know about the Spiti district of Himachal Pradesh. rivers and their origin points along with the : The Ravi is one of the major states they flow. transboundary rivers that flows between India 17. Key: D and Pakistan, under the Indus Water Treaty the Explanation: Narmada is one of the only three waters of the Ravi are allocated to India major rivers in peninsular India that run from The Sutlej River is one of the Sutlej River: east to west. The river is known as "Life Line of longest tributary of Indus. The source of the Madhya Pradesh and Gujarat". Sutlej is west of catchment area of the Lake Tapti is another west flowing river of Madhya . It enters Indian state of Rakshastal in Tibet Pradesh. The Ukai Dam also known as Vallabh Himachal Pradesh through the Shipki La pass. Sagar is constructed across the Tapi River is the To learn about the Educational Objective: second largest reservoir in Gujarat after the source/origination of rivers. Sardar Sarovar. 16. Key: A In Kerala there are more than 44 major rivers of Explanation: Amarkantak is a pilgrim which 3 are east flowing. Kabani, Paampar and town in Madhya Pradesh, India. The Bhavani are the east flowing rivers. Periyar is Amarkantak region is a unique natural the longest west flowing river in Kerala followed heritage area and is the meeting point of by Bharathapuzha and Pampa. the Vindhya and the Satpura Ranges, with Educational Objective: To learn about the east the Maikal Hills being the fulcrum. This is where and west flowing rivers. the Narmada River, the and Johila 18. Key: D River emerge. Explanation: The River Chambal is a tributary of River Yamuna and thus forms part of Gangetic drainage system. The Chambal originates at janapav, south of town, near Indore. Gandhisagar Dam, the Rana Pratap Sagar Dam, the Jawahar Sagar Dam, Kota Barrage are some of the major projects on Chambal River. The Son River originates near Amarkantak in of Madhya PradeshSon River of central India is the second largest tributary of the Ganges' southern tributaries after Yamuna River. Manas River is a transboundary river between Bhutan and India. Manas River passes through

www.laexias.com Page No. 102 https://elearn.laex.in Geography UPSC Previous Year Questions

Manas National Park, a UNESCO Natural World extremity is the Vindhya Range. The average Heritage site, a Project Tiger reserve, an elephant elevation of Deccan plateau is about 2,000 feet reserve and a biosphere reserve in Assam, India. sloping gentally eastward. Educational Objective: To learn about the river The Godavari, Krishna, and Kaveri are the and its tributaries. principal rivers which flows from the Western Ghats toward the Bay of Benga. Tiruvannamalai 19. Key: D in Tamil Nadu is often regarded as the Southern Explanation: Damanganga-Pinjal river linking gateway of Deccan plateau. project proposes to divert excess water from : To learn about the Kali reservoirs in the Damanganga basin to Educational Objective river and Deccan plateu. through the Pinjal dam, built on the Pinjal river in Vaitarna basin. Daman Ganga also called 21. Key: C Dawan River is a river in . The Explanation: Amarkantak (Amarkantak Plateau) river's headwaters are on the western slope of in Anuppur, Madhya Pradesh is a major pilgrim the Western Ghats range, and it flows west into town and place of origin of two important rivers the Arabian Sea. The river flows through Narmada & the Son River. Maharashtra and Gujarat states. The Narmada River, also called by the name Gima irrigation project in Maharashtra. Rewa rises from Amarkantak Plateau. The river Pamba Irrigation scheme in Pattanamthitta is known as "Life Line of Madhya Pradesh and district aims at the utilization of the tail race Gujarat". water of Sabarigiri Hydro Electric project for Narmada is one of the only three major rivers in irrigation purpose. The tail race water is let into peninsular India that run from east to west.It is the river Kakkad and is picked up at Maniyar by one of the rivers in India that flows in a rift a barrage. The water thus collected is diverted valley (between the Satpura and Vindhya ranges). through a canal on the left bank of the river. Educational Objective: To learn about the Educational Objective: Places and location rivers which originate at Amarkantak.

20. Key: D 22. Key: B Explanation: : The Kali River flows Explanation: Betul: Betul is a district in the through Uttara Kannada district of Karnataka state of Madhya Pradesh. the city is located on The Kali River has its origin in the Western the bank of river Tapti. Ghats and flows eastwards into the Jagdalpur: Jagdalpur is a city in Bastar district Reservoir. The river flows entirely through the in the state of Chhattisgarh. The city is located district of Uttara Kannada and finally joins the on the bank of river Indravati. Nagarnar Steel Arabian Sea near the town of Karwar. Plant, being set up at its outskirts of the city. Deccan Plateau: The Deccan Plateau is a large Jabalpur: Jabalpur city in the state of Madhya plateau located in between the Western Ghats Pradesh is located on the bank of river Narmada. and the Eastern Ghats, while its northern

www.laexias.com Page No. 103 https://elearn.laex.in Geography UPSC Previous Year Questions

Jabalpur is also known for the Dhuandhar Falls 24. Key: C and the white Marble Rocks at Bhedaghat. Explanation: Mahanadi: Mahanadi rises as a Ujjain: Ujjain district in the state of Madhya combination of many mountain streams however Pradesh is a famous Hindu pilgrimage centre. its farthest headwaters lie from Sihawa in The Kumbh Mela is held in Ujjain every 12 years. of Chhattisgarh. River Seonath, As an ancient city it is situated on the eastern Jonk and Hasdeo are the major tributaries of bank of the Kshipra River. Historically It was the Mahanadi. The largest earthen dam in the world, capital of the ancient Avanti kingdom, one of the the is built across the river sixteen Mahajanapadas. Mahanadi. Educational Objective: To learn about the Kaveri: River Kaveri originates in a place called rivers and towns nearer to it. . In the hills in Kodagu, flowing through the states of Karnataka and 23. Key: B Tamil Nadu. It is the third largest river in Explanation: Shyok River: The Shyok River peninsular India after Godavari and Krishna. Shyok River originates at the Rimo Glacier, one The Kaveri is also one of the seven holy rivers of of the tongues of Siachen Glacier.It is a tributary India. Harangi, Hemavati, and of the Indus River that flows through northern Arkavathy are its left bank tributaries where as Ladakh in India,the river joins the Indus at Keris. , Kabini, Bhavani, Noyyal, : The Zanskar River is a north- Amaravati and Moyar are its right bank flowing one of the major tributaries of the Indus tributaries. river. Zanskar River is formed by the confluence River Kaveri forms two islands in of and Lungnak River the Zanskar district's and Shivanasamudra. river meets Indus river at Nimmu village in The Godavari is east flowing Ladakh, and the place is popularly known as Godavari: peninsular river originates in the Western Ghats Sangam. at Triambakeshwar in Maharashtra. It is India's Spiti River: Spiti River is a tributary of Sutlej second longest river after the Ganga. river.It originates from the Kunzum Range of the It has its drainage basin in the states of Himalayas. The Spiti River flows through the Maharashtra, Telangana, Andhra Pradesh, valleys of Lahaul-Spiti district of Himachal Chhattisgarh, Odisha, ultimately emptying into Pradesh. the Bay of Bengal. Sutlej River: The Sutlej River is one of the The Godavari Water Disputes Tribunal governs longest tributary of Indus. The source of the the water allocation from the river among the Sutlej is west of catchment area of the Lake riparian states. Rakshastal in Tibet. It enters Indian state of Himachal Pradesh through the Shipki La pass Tapti: Tapti is a west flowing river of Madhya Pradesh. The Ukai Dam also known as Vallabh Educational Objective: To learn about the map Sagar is constructed across the Tapi River, is the locations of rivers.

www.laexias.com Page No. 104 https://elearn.laex.in Geography UPSC Previous Year Questions

second largest reservoir in Gujarat after the second largest reservoir in Gujarat after the Sardar Sarovar. Sardar Sarovar. Educational Objective: To learn about the river Educational Objective: To learn about the and location of its origin. length of the rivers in order.

25. Key: B 26. Key: C Explanation: Godavari: The Godavari is east Explanation: West-flowing rivers of Peninsular flowing peninsular river originates in the India have no large deltas because most of the Western Ghats at Triambakeshwar in Western flowing rivers originate in Western ghat Maharashtra. it is India's second longest river so the rivers flow from very high gradient and after the Ganga. the rivers travel for very short distance to carry It has its drainage basin in the states of any large amount of sediments. Maharashtra, Telangana, Andhra Pradesh, The most active South-west monsoons create a Chhattisgarh, Odisha, ultimately emptying into very high waves which carry or wash away the the Bay of Bengal. whatever little sediments brought by the rivers. The Godavari Water Disputes Tribunal governs Educational Objective: To learn about the west the water allocation from the river among the flowing rivers of peninsular India. riparian states. 27. Key: A Narmada: The Narmada River, also called by the Explanation: Subarnarekha: The River name Rewa rises from Amarkantak Plateau. The Subarnarekha originates at a village named river is known as "Life Line of Madhya Pradesh piska near in . And and Gujarat". it flows through the states of Jharkhand, West Narmada is one of the only three major rivers in Bengal and Odisha. It is majorly a rain-fed river peninsular India that run from east to west.It is Mahanadi: Mahanadi rises as a combination of one of the rivers in India that flows in a rift many mountain streams. However its farthest valley (between the Satpura and Vindhya ranges). headwaters lie from Sihawa in Dhamtari district Mahanadi rises as a combination of Mahanadi: of Chhattisgarh. River Seonath, Jonk and many mountain streams However its farthest Hasdeo are the major tributaries of Mahanadi. headwaters lie from Sihawa in Dhamtari district The largest earthen dam in the world, the of Chhattisgarh River Seonath, Jonk and Hasdeo Hirakud Dam is built across the river Mahanadi. are the major tributaries of Mahanadi. The : The River Krishna originates in largest earthen dam in the world, the Hirakud the Western Ghats near Mahabaleshwar, in the Dam is built across the river Mahanadi. state of Maharashtra. It is the fourth-biggest Tapti is a west flowing river of Madhya Tapti: river in terms of water inflows and river basin Pradesh. The Ukai Dam also known as Vallabh area in India. Its basin spread across Sagar is constructed across the Tapi River is the Maharashtra, Karnataka, Telangana and Andhra Pradesh.The largest tributary of the Krishna

www.laexias.com Page No. 105 https://elearn.laex.in Geography UPSC Previous Year Questions

River is the where as the the canal terminates in the Thar Desert in longest tributary is the the north west of Rajasthan state. Godavari: The Godavari is east flowing Narmada: The Narmada River, also called by the peninsular river originates in the Western Ghats name Rewa rises from Amarkantak Plateau. The at Triambakeshwar in Maharashtra. it is India's river is known as "Life Line of Madhya Pradesh second longest river after the Ganga. and Gujarat". It has its drainage basin in the states of Nizam Sagar Dam: Nizam Sagar Dam located in Maharashtra, Telangana, Andhra Pradesh, Telanganais a reservoir constructed across the Chhattisgarh, Odisha, ultimately emptying into . Manjira River is a tributary of the the Bay of Bengal. Godavari River. The Godavari Water Disputes Tribunal governs Educational Objective: To learn important facts the water allocation from the river among the about rivers. riparian states. 29. Key: A Kaveri: River Kaveri originates in a place called Explanation: Damodar River: Damodar River Talakaveri. In the Brahmagiri hills in Kodagu, originates at Chota Nagpur Plateau in flowing through the state of Karnataka and Jharkhand.it flows across the Indian states of Tamil Nadu. It is the third largest river in Jharkhand and West Bengal. Damodar is a rift peninsular India after Godavari and Krishna. valley river. The Kaveri is also one of the seven holy rivers of Earlier the river is known as the Sorrow of India. Harangi, Hemavati, Shimsha and Bengal because of its ravaging floods in the Arkavathy are its left bank tributaries where as plains of West Bengal. Barakar, Konar, Bokaro, Lakshmana Tirtha, Kabini, Bhavani, Noyyal, are major tributaries a of Damodar river Amaravati and Moyar are its right bank The Damodar valley is rich in coal. And it is tributaries. considered as the prime centre of coking coal in River Kaveri forms two islands in Mandya the country. The Damodar Valley is one of the district's Srirangapatna and Shivanasamudra. most industrialised parts of India. Thus it is To learn about the map Educational Objective: called as “the Ruhr of India”. locations/sequence of the eastward flowing Other rift valley rivers india are Narmada,Tapi rivers. and Mahi. 28. Key: A Educational Objective: To learn about the east Explanation: Indira Gandhi Canal: The Indira flowing rivers and rift valleys.

Gandhi Canal is the longest canal of India. 30. Key: B It starts from the Harike Barrage at Harike, Explanation: The Sambhar Salt Lake, India's Punjab a few kilometers below the largest inland salt lake, is located 80 km confluence of the Satluj and Beas rivers and (50 mi) southwest of the city of Jaipur and 64 km (40 mi)northeast of Ajmer, Rajasthan.

www.laexias.com Page No. 106 https://elearn.laex.in Geography UPSC Previous Year Questions

Explanation: Chambal: The River Chambal is a tributary of River Yamuna and thus forms part of Gangetic drainage system. The Chambal originates at janapav, south of Mhow town, near Indore. Gandhisagar Dam, the Rana Pratap Sagar Dam, the Jawahar Sagar Dam, Kota Barrage are some of the major projects on Chambal River. Krishna River: The River Krishna originates in the Western Ghats near Mahabaleshwar, in the state of Maharashtra. It is the fourth-biggest river in terms of water inflows and river basin Educational Objective: To learn about the area in India. Its basin spread across major lakes and the cities around them. Maharashtra, Karnataka, Telangana and Andhra 31. Key: C Pradesh. Alamatti dam is located on the krishna Explanation: The Tehri Dam is the highest river in Karnataka. dam in India and one of the highest in the world. Narmada: The Narmada River, also called by the It is a multi-purpose rock and earth- name Rewa rises from Amarkantak Plateau. The fill embankment dam on the BhagirathiRiver river is known as "Life Line of Madhya Pradesh near Tehri in Uttarakhand, India. From the and Gujarat". Sardar Sarovar Dam is the larges below map it is clear that point C is Theri Dam. dam in India built on Narmada. Kaveri: River Kaveri originates in a place called Talakaveri. In the Brahmagiri hills in Kodagu, flowing through the states of Karnataka and Tamil Nadu. It is the third largest river in peninsular India after Godavari and Krishna. KRS and Mettur Reservoirs are built on this river. Educational Objective: To learn about the river 3. Indian Climate and its dams.

1. With reference to 'Indian Ocean Dipole Educational Objective: To have idea about the important dams built in India, the rivers that (IOD)' sometimes mentioned in the news they are built on and their geographical while forecasting Indian monsoon, which locations of the following statements is/are correct? 1) IOD phenomenon is characterised by a 32. Key: B difference in sea surface temperature

www.laexias.com Page No. 107 https://elearn.laex.in Geography UPSC Previous Year Questions

between tropical Western Indian Ocean and 4. Which one of the following is the correct tropical Eastern Pacific Ocean. sequence of the given Indian cities in the 2) An IOD phenomenon can influence an El decreasing order of their normal annual Nino's impact on the monsoon’ rainfall? Select the correct answer using the code (a) Kochi-Kolkata-Delhi-Patna given below: (b) Kolkata-Kochi-Patna-Delhi (a) 1 only (c) Kochi-Kolkata-Patna-Delhi (b) 2 only (d) Kolkata-Kochi-Delhi-Patna (c) Both 1 & 2 5. Assertion: Bangalore receives much higher (d) Neither 1 nor 2 average annual rainfall than that of 2. Consider the following statements: Mangalore. 1. The winds which blow between 30°N and Reason: Bangalore has the benefit of 60°S latitudes throughout the year are receiving rainfall both from south-west and known as westerlies north-east monsoons. 2. The moist air masses that cause winter (a) Both A and R are true and R is the correct rains in North-Western region of India are explanation of A. part of westerlies. Which of the statements given above is / are (b) Both A and R are true but R is not the correct? correct explanation of A. (a) 1 only (c) A is true but R is false. (b) 2 only (d) A is false but R is true. (c) Both 1 & 2 6. The average annual temperature of a (d) Neither 1 nor 2 meteorological station is 26 C, its 3. Consider the following statements: average annual rainfall is 63cm and the 1. The duration of the monsoon decreases annual range of temperature is 9°C. The from Southern India to Northern India. station in question is 2. The amount of annual rainfall in the (a) Allahabad Northern plains of India decreases from east (b) Chennai to west. (c) Cherrapunji Which of the statements given above is/are (d) Kolkata correct? 7. With reference to India, which one of the (a) 1 only following statements is not correct? (b) 2 only (a) About one-third of the area of the country (c) Both 1 & 2 records more than 750 millimetres of (d) Neither 1 nor 2 annual rainfall (b) The dominant source of irrigation in the country is wells

www.laexias.com Page No. 108 https://elearn.laex.in Geography UPSC Previous Year Questions

(c) Alluvial soil is the predominant type of soil 10. Consider the following statements in the northern plains of the country regarding environmental issues of India: (d) The mountain areas account for about 1. is one of the biosphere thirty percent of the surface area of the reserves country 2. The Gangs Action Plan, phase II has been merged with the National River 8. In the shaded area of the map given Conservation Plan. below the mean temperature for the 3. At New Delhi imparts non-formal education month of July varies between in environment and conservation 4. Environmental Information System (ENVIS) acts as a decentralized information network for environmental information Which of these statements are correct? (a) 1, 2 & 4 (b) 1,2,3 &4 (c) 2 & 3 (d) 1, 3 & 4

(a) 22.5°C-25.0C 3. Key and Explanation (b) 25.0°C-27.5"C 1. Key: B (c) 27.5 C-30.0'C Explanation: The Indian Ocean Dipole is (d) 30.0°C-32.5 C characterised by a difference in sea surface

9. Assertion: Anti cyclonic conditions are temperature between tropical western Indian formed in winter season when atmospheric ocean and tropical eastern Indian ocean. Hence, pressure is high and air temperatures are statement 1 is wrong. A positive IOD occurs low. when sea surface temperature is higher than Reason: Winter rainfall in Northern India normal in the Arabian Sea and less than normal cause development of anti cyclonic in the tropical eastern Indian Ocean. The conditions. opposite is true in case of a negative IOD. (a) Both A and R are true and R is the correct An IOD can either aggravate or weaken the explanation of A. impact of El Nino on Indian monsoon. If there is (b) Both A and R are true but R is not the a positive IOD, it can bring good rains to India correct explanation of A. despite of an El Nino year. Similarly, During a (c) A is true but R is false. negative IOD and El Nino can cooperatively (d) A is false but R is true. produced efficient rainfall. Hence, statement 2 is correct.

www.laexias.com Page No. 109 https://elearn.laex.in Geography UPSC Previous Year Questions

Educational Objective: To learn about Indian owing to the progressive decrease in humidity of Ocean Dipole and its impact on Indian monsoon. the winds. Educational Objective: To learn about Indian 2. Key: B monsoons and retreat of monsoons. Explanation: The winds are described as Westerlies or Easterlies based on the direction 4. Key: C from which they are moving. Explanation: The rain bearing monsoonal The winds which blow between 30°N and 60°N clouds come in two main branches the Arabian latitudes and between 30°S and 60°S are known sea branch and Bay of Bengal branch. as westerlies, Thus statement 1 is wrong. The Arabian sea branch hits the Kerala coast A extratropical cyclones/storm originating in the (Kochi) by the early first week of June with Mediterranean region move towards east under maximum intensity. the influence of westerlies and brings sudden The Bay of Bengal branch reaches the north winter rain to the north western parts of the eastern India including West Bengal (Kolkata) by Indian. They are also known as Western the end of first week of June and the amount of Disturbance. Thus statement 2 is correct. rainfall in Kolkata is less compared to Kochin. Educational Objective: To know about the The amount of rainfall decreases from east to difference between easterlies and westerlies, west owing to the progressive decrease in their direction of movements and understand humidity of the winds in the Patna and Delhi about western disturbances. experience even less rainfall. To learn about Indian 3. Key: C Educational Objective: monsoons and important/main branches of Explanation: The rain bearing monsoonal monsoons. clouds come in two main branches the Arabian sea branch and Bay of Bengal branch. The 5. Key: D Arabian sea branch hits the Kerala coast by the Explanation: Mangalore being located in the first week of June, where as the it reaches the coastal Karnataka and to the west of Western north Indian plains at the end of second week, Ghats it gets rainfall from the Arabian sea and covers whole by the first of July. branch monsoonal winds with full intensity. But the retreat of monsoon begins in the north Whereas the Bangalore is located in the rain India by the second week of September and by shadow region of Western Ghats it get very less the December it retreaths from whole Indian rainfall compared to Mangalore. Thus statement subcontinent thus the duration of the monsoon 1 is wrong decreases from Southern India to Northern India Bangalore has the benefit of receiving rainfall The rain bearing monsoonal clouds with high both from south-west monsoon and north- humidity hits southern part of India first. The eastern or retreating monsoons. thus Statement amount of rainfall decreases from east to west 2 is correct.

www.laexias.com Page No. 110 https://elearn.laex.in Geography UPSC Previous Year Questions

Educational Objective: To learn about Indian wells irrigation, tanks irrigation, canals, monsoons and understand retreating monsoons. perennial canal, multi-purpose river valley projects. But the dominant source of irrigation 6. Key: A in the country is wells. The predominant type of Explanation: Allahabad is a city in the Indian soil in the northern plains of India is Alluvial soil. state of Uttar Pradesh. Allahabad has a humid Near thirty percent of the geographical area of subtropical climate with the annual mean country is covered by mountains temperature is 26.1 °C with maximum To learn about Indian temperature experienced in summer the annual Educational Objective: monsoons and types of irrigation systems which range of temperature is 90C its average annual are practised in india. rainfall is 63 cm. Chennai being a coastal city in Tamil Nadu has 8. Key: B very less annual temperature range. And it gets Explanation: The Inflow of the south westerly its maximum rainfall from North Eastern monsoons in to India brings about total change Monsoon rather than south western monsoon. in the weather condition. The very hot Cherrapunji in the state Meghalaya experience temperature experienced in summer due to heavy rainfall during monsoonal season from the intense heating of land suddenly drops by the Bay of Bengal branch of south west mansoonal inflow of the south westerly monsoons. But the winds. The funal shape of the Kasi hills lock the humidity of the also increases due to the inflow rain bearing clouds from moving out which south westerly monsoons. results in excessive rainfall. Educational Objective: To learn about the The Bay of Bengal branch of south west changes in weather condition after the inflow of monsoon reaches the north eastern India the south western monsoons.

including West Bengal (Kolkata) by the end of 9. Key: B first week of June and the amount of rainfall in Explanation: In the winter seasons due to low Kolkata is less compared to Kochin. temperature subsidence of air takes place thus Educational Objective: To understand the create a high pressure condition or a Anti raining patterns in some of the important cities cyclonic condition. in India. Where as in the summer due to to the excessive 7. Key: A heating of land a low pressure is created Explanation: Average annual rainfall in India is (upliftment of air takes place) thus create 650 millimetres. One-third of the area of the cyclonic conditions. Thus statement 1 is correct. country does not records more than 750 Winter rainfall in Northern India does not create millimetres of annual rainfall. Thus statement 1 anti cyclonic condition. Thus statement 2 is is wrong. wrong. In India various types of irrigation systems are practiced in different parts of India. Such as

www.laexias.com Page No. 111 https://elearn.laex.in Geography UPSC Previous Year Questions

Educational Objective: To learn about the 3) Tropical climate differences between the cyclonic and anti- Select the correct answer using the code cyclonic conditions. given below. (a) 1 and 2 only 10. Key: B (b) 2 only Explanation: In Gulf of Mannar is a Biosphere reserve and is one of the eighteen biosphere (c) 1 and 3 only reserves in India under the UNESCO’s Man and (d) Neither 1 nor 2 Biosphere (MAB) Programme. It is located 2. Identify the correct order of the between Indian part of Gulf of Mannar between (Tamil Nadu) India and Sri Lanka. Hence, processes of soil-erosion from the statement 1 is correct. following The Ganga Action Plan (GAP) Phase – I was (a) Splash erosion, Sheet erosion, Rill erosion, aimed at preventing the pollution of river Ganga Gully erosion was started in 1985.Ganga Action Plan Phase – (b) Sheet erosion, Splash erosion, Gully erosion, II was launched in 1993 to include Yamuna and Rill erosion Gomati rivers. National River Conservation Plan (NRCP) was (c) Rill erosion, Gully erosion, Sheet erosion, launched in 1995 to reduce the pollution in Splash erosion rivers improving and improve water quality. (d) Gully erosion, Rill erosion, Sheet erosion, After launching of NRCP in 1995, it was decided Splash erosion to merge GAP II with NRCP. Hence, statement 2 is correct. 3. Contour bunding is a method of soil Statement-3: Environment Information System conservation used in (ENVIS) is central sector Scheme of the (a) Desert margins, liable to strong wind action MOEFCC implemented since 1982. It acts as (b) Low flat plains, close to stream courses, Decentralized information network for liable to flooding environmental information collection, storage, (c) Scrublands, liable to spread of weed growth collation, retrieval and dissemination. Hence, (d) None of the above statement 4 is correct. Educational Objective: To know about Gulf of 4. With reference to soil conservation, Mannar Biosphere reserve, National River consider the following practices: Conservation Plan, Ganga action plan phase II 1) Crop rotation and ENVIS - Environment Information System. 2) Sand fences 3) Terracing 4) Wind breaks 4. Soil Which of the above are considered appropriate methods for soil conservation in 1. In India, the problem of soil erosion is India? associated with which of the following? (a) 1, 2 and 3 only 1) Terrace cultivation (b) 2 and 4 only 2) Deforestation

www.laexias.com Page No. 112 https://elearn.laex.in Geography UPSC Previous Year Questions

(c) 1, 3 and 4 only Select the correct answer using the codes (d) 1, 2, 3 and 4 given below. (a) 1, 2 and 3 5. With reference to agricultural soils, (b) 2, 3 and 4 consider the following statements: (c) 1 and 4 1. A high content of organic matter in soil (d) 2 and 3 only. drastically reduces its water holding capacity. 8. Which one of the following is the 2. Soil does not play any role in the sulphur appropriate reason for considering the cycle. Gondwana rocks as most important of 3. Irrigation over a period of time can rock systems of India? contribute to the salinization of some (a) More than 90% of limestone reserves of agricultural lands. India are found in them Which of the statements given above is/are (b) More than 90% of India's coal reserves are correct? found in them (a) 1 & 2 only (c) More than 90% of fertile black cotton soils (b) 3 only are spread over them (c) 1 & 3 only (d) None of the reasons given above is (d) 1,2 & 3 appropriate in this context

4. Key and Explanation 1. Key: B 6. When you travel in certain parts of India, Explanation: The Roots of trees and plants in a you will notice red soil. What is the main area bind the soil particles and regulate the flow reason for this colour? of water, thus saving soil from erosion. (a) Abundance of magnesium Deforestation (Removal of trees) makes soil (b) Accumulated humus vulnerable to wind and water erosion. Hence, 2nd (c) Presence of ferric oxides option is correct. (d) Abundance of phosphates In terrace cultivation, a number of terraces are cut along the hill slope. These are made on the 7. Which of the following statements steep slopes so that flat surfaces are available to regarding laterite soils of India are grow crops. They can reduce surface run-off and correct? soil erosion. Hence, 1st option is wrong 1) They are generally red in colour The type of climate doesn’t really affect the rate 2) They are rich in nitrogen and potash. of erosion as much the other factors like rainfall. 3) They are well-developed in Rajasthan and Hence, 3rd option is wrong. UP. Educational Objective: To learn about the 4) Tapioca and cashew nuts grow well on these problems/causes of soil erosion soils.

www.laexias.com Page No. 113 https://elearn.laex.in Geography UPSC Previous Year Questions

2. Key: A 4. Key: D Explanation: Splash Erosion: The erosion due Explanation: Crop rotation is a practice in to the impact of falling raindrops on soil surface which different crops are grown every year, on leading to the destruction of the crumb the basis of rotation. This helps to conserve soil structure is known as the raindrop or splash fertility as different crops require different erosion. It is also called Rain drop erosion. nutrients from the soil. Crop rotation will Sheet Erosion: The detachment and provide enough time to restore lost nutrients. transportation of soil particles by flowing Sand fences are barriers made of small, evenly rainwater is called sheet or wash off erosion. spaced wooden slats or fabric. They are erected Rill erosion: It occurs mostly during heavy rain. to reduce wind velocity and to trap blowing sand. In rill erosion finger like rills appear on the Sand fences can be used as perimeter controls cultivated land after it has undergone sheet around open construction sites to keep erosion. When rills increase in size, they are sediments from being blown offsite by the wind. called gullies. Terracing: On hilly slopes, terraces act as Gully Erosion refers to the cutting of narrow bunds and prevent the soil from being washed channels called gullies. It cuts deep and removes away. Farming is done on a unique step like the surface soil as well as deeper soil that may structure and the force of water running off is still have substantial amounts of total nutrients slowed down but less compared to the surface soil. Windbreaks are the rows of trees planted to Hence, the correct order is Splash erosion, Sheet check the wind movement to protect soil cover. erosion, Rill erosion, Gully erosion (1st option) They are generally found in coastal and dry Educational Objective: To learn about the regions. process /stages of soil erosion Educational Objective: To learn about various soil conservation methods 3. Key: D Explanation: In Contour Bunding, a series of 5. Key: B checks are put in place across the slope of a Explanation: Organic matter acts like a sponge, hilly surface. with the ability to absorb and hold up to 90 The bunds divide the entire slope into numerous percent of its weight in water. It therefore smaller segments which help in reducing the increases Water-Holding Capacity of the soil. runoff along the slopes, ensure greater Hence, statement 1 is wrong absorption of rainwater on the slopes thus The sulphur reservoir is in the soil and controlling soil erosion. sediments where it is locked in organic (coal, oil Sometimes, the bunds can be in the form of and peat) and inorganic deposits (pyrite rock retaining walls. and sulphur rock) in the form of sulphates, Educational Objective: To learn about contour sulphides and organic sulphur. bounding method of soil conservation

www.laexias.com Page No. 114 https://elearn.laex.in Geography UPSC Previous Year Questions

It is released by weathering of rocks, erosional Distribution: They are well developed in south runoff and decomposition of organic matter and Maharashtra, parts of Karnataka etc. and are is carried to terrestrial and aquatic ecosystems widely scattered in other regions. UP has alluvial in salt solution. Hence, statement 2 is wrong. soils where as Rajasthan has desert soil, Hence, Salinity from irrigation can occur over time statement 3 is wrong. wherever irrigation occurs, since water contains Laterite soils lack fertility due to intensive some dissolved salts. When the plants use the leaching, so are not fit for agricultural crops. water, the salts are left behind in the soil and Plantation crops like tea, coffee, rubber, eventually begin to accumulate leading to cinchona, coconut, arecanut, etc. Can be grown. salinization of Agricultural lands. Hence, Hence, statement 4 is correct. statement 3 is correct Educational Objectives: To learn the Educational Objective: To know about impact characteristics of laterite soils of organic matter, sulphur cycle, Irrigation on 8. Key: B soil Explanation: Gondwana rock system is named after the 6. Key: C huge carbon deposits contained within them. They Explanation: Red soils are red in colour due to contain up to 98 percent of our coal deposits, Hence, are presence offerric oxides. The lower layer is the largest source of coal in India, reddish yellow or yellow. They are mainly found in Ranigunj, Jharia regions of These soils mostly occur in the regions of low Jharkhand, Damodar valley, Pench valley in Chhattisgarh rainfall. They occupy about 3.5 lakh sq km (10.6 and Madhya Pradesh, Godavari valley in Telangana and per cent) of the total area of the country. They the Rajmahal hills of West Bengal. are poor in lime, magnesia, phosphates, nitrogen They are named after the Gond (indigenous people and humus. They are rich in potash and mainly found in the Telangana and Andhra Pradesh potassium. regions). These are sedimentary deposits which were Educational Objective: To know about accumulated in the synclinal troughs on an older plateau characteristics of red soil surface. Educational Objective: To learn about the Gondwana rock system.

7. Key: C Explanation: Laterite soil occurs in the areas of 5. Minerals high temperature and high rainfall. They are formed as a result of high leaching and Red 1. Which among the following has the colour due to iron oxide.Hence, statement 1 is world's largest reserves of Uranium? correct. (a) Australia They are very poor in lime, magnesia, potash (b) Canada and nitrogen. Hence, statement 2 is wrong (c) Russian Federation

www.laexias.com Page No. 115 https://elearn.laex.in Geography UPSC Previous Year Questions

(d) USA 1. In India, State Governments do not have the power to auction non- coal mines. 2. Which one of the following countries is 2. Andhra Pradesh and Jharkhand do not the leading producer of uranium? have gold mines. (a) United States of America 3. Rajasthan has iron ore mines. (b) Canada Which of the statements given above is/are (c) Germany correct? (d) Zambia (a) 1 and 2 (b) 2 only 3. Match List-I (Minerals) with List-II (Major (c) 1 and 3 producer) and Select the correct answer (d) 3 only using the codes given below the lists: List-I List-II 6. Which of the following is/are the (a) Mineral Oil 1. Zambia characteristic/ characteristics of Indian (b) Copper 2. Guyana coal? (c) Manganese 3. Venezuela 1. High ash content (d) Bauxite 4. Gabon 2. Low sulphur content Codes: 3. Low ash fusion temperature (a) A-3; B-1; C-4; D-2 Select the correct answer using the codes (b) A-3; B-1; C-2; D-4 given below. (c) A-1; B-3;C-2; D-4 (a) 1 & 2 only (d) A-1; B-3; C-4; D-2 (b) 2 only (c) 1 & 3 only 4. Which of the following substances are (d) 1, 2 & 3 found in the beach sands of many parts of Kerala? 7. Consider the following statements: 1. Ilmenite 1. Natural gas occurs in the Gondwana beds. 2. Mica occurs in abundance in Kodarma. 2. Zircon 3. Dharwars are famous for . 3. Sillimanite Which of the statements given above is/are 4. correct? Select the correct answer using the code (a) 1 & 2 given below: (b) 2 only (a) 1,2,3 &4 (c) 2 & 3 (b) 1,2 & 3 only (d) None (c) 3 & 4 only (d) 1 & 2 only 8. With reference to the mineral resources of India, consider the following pairs: 5. Consider the following statements: Mineral 90% Natural sources in (a) Copper Jharkhand

www.laexias.com Page No. 116 https://elearn.laex.in Geography UPSC Previous Year Questions

(b) Nickel Orissa 12. Consider the following statements: (c) Tungsten Kerala 1. Balaghat is known for its diamond mines. Which of the pairs given above is/are 2. Majhgawan is known for its manganese correctly matched? deposits (a) 1 and 2 only Which of the statements given above is/are (b) 2 only correct (c) 1 and 3 only (a) 1 only (d) 1, 2 and 3 (b) 2 only (c) Both 1 and 2 9. In which of the following states are (d) Neither 1 nor 2 Namchik – Namphuk Coalfields located (a) Arunachal Pradesh 13. Shahgarh area in Jaisalmer district of (b) Meghalaya Rajasthan was in news in the year 2006 (c) Manipur because of which one of the following? (d) Mizoram (a) Finding high quality gas reserves (b) Finding uranium deposits 10. Which of the following minerals found in (c) Finding deposits a natural way in the state of (d) Installation of wind power units Chhattisgarh? 1) Bauxite 14. Match List-I (Minerals) with List-II 2) Dolomite (Location) and select the correct answer 3) Iron ore using the codes given below the Lists: 4) Tin List-I (Minerals) List-II (Location) Select the correct answer using code given A. Coal 1. Giridih below codes B. Copper 2. Jayankondam (a) 1,2 and 3 only C. Manganese 3. Alwar (b) 1 and 3 only. D. Lignite 4. Dharwar (c) 2 and 4 only Codes: (d) 1,2 3 and 4 (a) A-1; B-4; C-3; D-2 (b) A-2; B-3; C-4; D-1 11. In which one of the following districts, (c) A-1; B-3; C-4; D-2 have large reserves of diamond-bearing (d) A-2; B-4; C-3; D-1 kimberlite been discovered in the recent past? 15. HINDALCO, an aluminium factory located (a) Hoshangabad at Renukut owes its site basically to (b) Raipur (a) Proximity of raw materials (c) (b) Abundant supply of power (d) Warangal (c) Proximity to the network

www.laexias.com Page No. 117 https://elearn.laex.in Geography UPSC Previous Year Questions

(d) Proximity to the market C. Nhava Sheva 3.Petro-chemical complex 16. In the map black marks show the D. Ratnagiri 4. Port distributions of Codes: (a) A-1; B-3; C-2; D-4 (b) A-2; B-3; C-4; D-1 (c) A-3; B-4; C-2; D-1 (d) A-2; B-1; C-4; D-3

19. Commercial production of mineral oil has started recently in which one of the areas of India, labelled 1, 2, 3 and 4 in the rough map given below: (a) Asbestos (a) 1 (b) Gypsum (b) 2 (c) Limestone (c) 3 (d) Mica (d) 4

17. Match List-I with List-II and select the correct answer using the codes given below the list Lists-I (Minerals) List-II (Mining area) A. Graphite 1. Bellary B. Lead 2. Didwana

C. Salt 3. Rampa 20. Which of the following is/are the D. 4. Zawar possible consequences of heavy sand Codes: mining in riverbeds? (a) A-3; B-4; C-1; D-2 1. Decreased salinity in the river (b) A-1; B-4; C-2; D-3 2. Pollution of groundwater (c) A-3; B-1; C-4; D-2 3. Lowering of the water-table (d) A-2; B-3; C-1; D-4 Select the correct answer using the code 18. In the vicinity of Mumbai, a number of given below: specialised towns have been developed. a. 1 only Match the lists and select the correct b. 2 & 3 only answer using the codes c. 1 & 3 only List-I (Towns) List-II (Specialisation) d. 1,2 & 3 A. Alibag 1. Fishing centre B. Balapur 2. Holiday resort

www.laexias.com Page No. 118 https://elearn.laex.in Geography UPSC Previous Year Questions

forms such as uraninite, brannerite and 5. Key and Explanation carnotite. It is also found in phosphate rock and 1. Key: A monazite sands. Explanation: Uranium is radioactive metal  Canada has the fourth largest reserve of (silvery-gray metal) used as nuclear fuel to Uranium in the world. And it is the world's generate electricity in nuclear power stations. second largest producer of uranium, with  Uranium occurs naturally in several mineral approximately 15% of global production forms such as uraninite, brannerite and (2017). carnotite. It is also found in phosphate rock  In Canada uranium is currently mined from and monazite sands. the McArthur River, Cigar Lake and Rabbit  The purified oxide of uranium is also known Lake (Eagle Point) mines in northern as yellow-cake. Saskatchewan.  Australia has the world's largest reserve of  The world’s largest and Canada’s only Uranium. it possesses around 30% of the uranium refinery is located at Blind River, world's known recoverable uranium reserves. Ontario. And approximately 85 percent of  Australia is followed by Kazakhstan. It Canada’s uranium production is exported. contains about 13% of the world's  Even though the uranium deposits are also recoverable uranium reserve. found in the United States of America and  Russia, with about 9% of the world's Germany, But their production rate is not uranium possesses the third largest much. uranium reserve in the world. While Canada Educational Objective: To learn about the has the fourth largest reserve of Uranium. uranium producing countries.  Uranium deposits in India can be found in 3. Key: A Singhbhum and Hazaribagh districts of Explanation: Mineral Oil: Venezuela- The Jharkhand, Gaya district of Bihar, and in proven oil reserves in Venezuela are recognized Saharanpur district of Uttar Pradesh (In the as the largest in the world. sedimentary rock). While the largest Copper: Zambia- the Republic of Zambia is a concentration of monazite sand is found on in Southern-Central Africa, the Kerala coast (Malabar Coast). The Zambian economy has historically been Educational Objective: To learn about the based on the copper mining industry. In India reserves of uranium. Rajasthan (50%) 2. Key: B Madhya Pradesh (24%) Explanation: Uranium is radioactive metal Jharkhand (19%) (silvery-gray metal) used as nuclear fuel to Are the leading producers of Copper. generate electricity in nuclear power stations. Manganese: Gabon- Gabon is a world leader in Uranium occurs naturally in several mineral manganese. It has high-grade manganese

www.laexias.com Page No. 119 https://elearn.laex.in Geography UPSC Previous Year Questions

deposits. Reserves were estimated at 250 million Amendment Act, 2015. The non-coal mines have tons with a metal content of 48%–52%. to be auctioned by the respective state In India, Odisha, Karnataka and Madhya governments. Thus statement 1 is incorrect. Pradesh are the leading producers of manganese. ● Gold in its purest form is a bright, slightly Bauxite: Guyana- Guyana was known to have a reddish yellow, dense, soft malleable and 350-million-ton bauxite reserve, one of the ductile metal. In India both Andhra world's highest concentrations. Bauxite mining Pradesh and Jharkhand have gold mines was concentrated in northeast Guyana. [Andhra Pradesh (3%), Jharkhand (2 %)]. In India, Odisha, Andhra Pradesh, Gujarat are Other States with gold reserves are Bihar the leading producers of bauxite ore. Odisha (44%) followed by Rajasthan (25%), alone accounts for 52 per cent. Karnataka (21%), West Bengal (3%). But In Educational Objective: To learn about the terms of metal content, Karnataka remained minerals and its major producers. on top followed by Rajasthan, Andhra Pradesh, Bihar, Jharkhand, etc. Thus 4. Key: B statement 2 is incorrect. Explanation: Heavy-mineral concentrates ● Rajasthan has iron ore deposits in Jaipur, known as “Black sands” occur in layers and Bhilwara, Udaipur, Sikar and Dausa. patches all along the Kerala beaches. Educational Objective: To learn about the coal,  The Heavy Mineral Sand deposits in Kerala non-coal and iron ore mines. contain an assemblage of Ilmenite, Rutile, Leucoxene, Monazite, Zircon and Sillimanite. 6. Key: A  The State possesses one of the world class Explanation: The Coal resources of India are deposits of mineral sands in the coastal available in older Gondwana Formations of tracts. peninsular India and younger Tertiary  These deposits are commonly known as the formations of north-eastern region. Chavara deposit, The Chavara barrier beach  Jharkhand accounts for largest reserve of portion contains concentration of heavy coal deposits in India followed by Odisha, minerals above 60%. Chhattisgarh and West Bengal.  The Kerala state also owns mineral deposits  Hard coal deposits spread over 27 major like placers, china clay (kaolin), limestone, coalfields, are mainly confined to eastern lime shell, silica sand, bauxite, graphite, and south central parts of the country. iron ore, granite etc.  The lignite reserves stand at a level around Educational Objective: To learn about the 36 billion tonnes, of which 90 % occur in heavy mineral sands and its deposits. the southern State of Tamil Nadu.  5. Key: D Most of the Indian contain a higher Explanation: As per the new Mines and percentage of inorganic impurities. Minerals (Development and Regulation)

www.laexias.com Page No. 120 https://elearn.laex.in Geography UPSC Previous Year Questions

 Indian coals like any other gondwana coals district of West Bengal and Degana near Rawat are of high ash content. Hills in Rajasthan.  Indian coals also has very low sulphur Educational Objective: To learn about the content minerals and its natural sources/locations.

Educational Objective: To learn about the 9. Key: A characteristics of Indian coal.  Explanation: Namchik-Namphuk coalfields 7. Key: B are located in the state of Arunachal Explanation: The Coal resources are found in Pradesh. The mining rights of Namchik - Gondwana Formations not Natural gas. Thus Namphuk coalfield has been given to the statement 1 is incorrect. Arunachal Pradesh Mineral Development Koderma district in Jharkhand is rich in and Trading Corporation Ltd (APMDTCL). minerals. The Koderma district and the Lokai-  The Coal resources of India are available in Indarwa area covers the southern part of the older Gondwana Formations of peninsular Great Mica-Belt of Jharkhand and Bihar India. India and younger Tertiary formations of Thus statement 2 is correct. north-eastern region. Dharwars formation are famous for Iron ore  Jharkhand accounts for the largest reserve deposits and not for petroleum. Thus statement of coal deposits in India followed by Odisha, 3 is incorrect. Chhattisgarh and West Bengal. Educational Objective: To learn about the  Hard coal deposits spread over 27 major resources and its place of occurrence. coalfields, are mainly confined to eastern and south central parts of the country.  The lignite reserves stand at a level around 36 billion tonnes, of which 90 % occur in 8. Key: B the southern State of Tamil Nadu. Explanation: In India Rajasthan (50%), Madhya Educational Objective: To learn about the Pradesh (24%), Jharkhand (19%) are the leading Coal fields and its locations. producers of Copper. 10. Key: D Nickel is a lustrous, silvery-white metal having a Explanation: Chhattisgarh is one of the high melting point of 1455oC. It exhibits high foremost mineral rich States in the country. resistance to corrosion and oxidation, excellent Twenty-Eight known varieties of minerals are strength and toughness. Nickel occurs found in the State including precious stones & principally as oxides, sulphides and silicates in diamonds, iron ore, coal, limestone, dolomite, tin India. Orissa has the highest amount of Nickel ore, bauxite and gold. Ore resources.  Chhattisgarh possesses India’s only active The deposits of wolfram, the chief ore of tin mine (in Bastar district), and one of Tungsten, are found in Chendpathar in Bankura world’s best quality of iron ore deposits in

www.laexias.com Page No. 121 https://elearn.laex.in Geography UPSC Previous Year Questions

the world (at Bailadila in Dantewada diamond mines. NMDC is the only organised district). producer of diamonds in India from its  The State has high potential for mining good Majhgawan mine at Panna, Madhya quality of diamonds. Pradesh. Educational Objective: To learn about the Educational Objective: To learn about the minerals found naturally in Chattisgarh. locations of diamond and manganese mines.

11. Key: B 13. Key: A Explanation: Chhattisgarh has one of the Explanation: Natural gas consists primarily of biggest diamond mines in the country. methane. Natural gas is often found dissolved in Chhattisgarh's diamond reserve are found at oil or as a gas cap above the oil reserves. Raipur and Devbhog region of Maoist-hit  Natural gas is the cleanest fossil fuels Gariyaband district. among the available fossil fuels. ● Chhattisgarh has an estimated 13 lakh  KG basin, Assam, Gulf of Khambhat, carat diamonds in its reserves which is Cuddalore district of Tamil Nadu, Barmer in 28.26 percent of total 46 lakh carat Rajasthan are the major natural gas fields diamond reserves in India. in India. ● The diamond mine is located about 130  Shahgarh area in Jaisalmer new gas kilometres from Gariyaband headquarter reserves had found. towards the border of Odisha  According to officials. The gas reserve was ● Hoshangabad is one of the diamond mines discovered at 3,465 metres depth and is in the country. A total reserve of 976.05 stretched up to five km area. Study and thousand carats has been estimated in the experiment has started to ascertain exact Panna district. quantity oil in those wells. Educational Objective: To learn about the Educational Objective: To learn about the reserves of diamond in India. natural gas reserves in India.

12. Key: D 14. Key: C Explanation: Balaghat of Madhya Pradesh is Explanation: Giridih Coalfield is located in known for its manganese mines. State is Giridih district in the Indian state of Jharkhand. enriched with good deposits of manganese ore The Coal resources of India are available in older mainly spread over in the Balaghat, Chhindwara Gondwana Formations of peninsular India and and Jhabua districts. younger Tertiary formations of north-eastern  The "Bharveli manganese mine" in Balaghat region. district is the largest underground mine  Copper mineralization from the Alwar basin operating in Asian subcontinent. in Rajasthan (of the north Delhi fold belt)  Majhgawan is a town located in Satna found. The tentative resource of copper is District, Madhya Pradesh is famous for its about 11 million tones.

www.laexias.com Page No. 122 https://elearn.laex.in Geography UPSC Previous Year Questions

 Dharwar rocks have been formed as a result  Rajasthan and Andhra Pradesh are the only of the erosion and sedimentation of the states that produce most of the Asbestos. rocks of the Archaean system. The Hence option A is wrong. prominent metallic minerals found in these  Distribution of limestone in India Madhya rocks are iron, gold, manganese etc. Pradesh, Rajasthan, Andhra Pradesh,  Lignite is found in Jayankondam in Gujarat, and Chhattisgarh Option C relates Tamilnadu. to the above Map. Hence option C is correct. Educational Objective: To learn about the Educational Objective: To learn about the minerals and its sources/locations. distribution of mica, gypsum, limestone and asbestos in India. 15. Key: B Explanation: HINDALCO is the world’s largest 17. Key: B aluminium rolling company and one of Asia’s Explanation: Zawar is a settlement located in biggest producers of primary aluminium. Udaipur district, Rajasthan, is known for lead  HINDALCO, an aluminium factory located at mining. Zawar is a township created by mining Renukut owes its site basically to Abundant company Zinc Limited, for extraction supply of power from nearby thermal Power of zinc and lead. Stations (National Thermal Power Station,  Didwana is a town in Nagaur district of Rihand). Rajasthan in India, in the Thar Desert.is  Rihand Super Thermal Power Project is known for salt pans. located at Renukut, Sonebhadra in  Rampa (Rampachodavaram) is a census in Indian state of Uttar town in East Godavari district of the Indian Pradesh. The power plant is one of the coal state of Andhra Pradesh. based power plants of NTPC Limited. Educational Objective: To learn about the  Thus statement 2 is correct. minerals and its mining areas. To learn about the Educational Objective: 18. Key: B location of aluminium factories in India. Explanation: Alibag is a coastal town and a 16. Key: C municipal council in Raigad District of Explanation: Distribution of MICA in India Maharashtra, India. It is the headquarters Andhra Pradesh, Rajasthan, Jharkhand. Hence of the Raigad district and is south of the city the above map does not relate to Mica of Mumbai. Alibag is part of Mumbai Distribution. Option D is wrong. Metropolitan Region and is situated at a  Gypsum is mainly produced majorly in distance of about 96km from Mumbai and it Rajasthan (nearly 99% of the production) is known for Holiday resorts. hence option B is wrong.  The Thane – Belapur region is one of the largest industrial regions in India. Belapur is known for its Petro-chemical complex.

www.laexias.com Page No. 123 https://elearn.laex.in Geography UPSC Previous Year Questions

Port, also known as Explanation: Depletion of sand in the Nhava Sheva, is the largest container port in streambed and along coastal areas causes the India. Located east of Mumbai (previously deepening of rivers and estuaries, and the known as 'Bombay') in Navi Mumbai, enlargement of river mouths and coastal inlets. Maharashtra. It may also lead to saline-water intrusion from  Ratnagiri is a port city on the Arabian Sea the nearby sea. Thus there will be increased coast in Ratnagiri District in the salinity in the river. Hence statement 1 is southwestern part of Maharashtra, India. wrong. Fishing is done throughout all the coasts of Sand mining transforms the riverbeds into large Ratnagiri. and deep pits; as a result, the groundwater Educational Objective: To learn about the table drops leaving the drinking water wells on important towns in Maharashtra and its the embankments of these rivers dry. specialisation areas. Bed degradation from in-stream mining lowers the elevation of stream flow and the floodplain 19. Key: C water table which in turn can eliminate water Explanation: In Cauvery on-shore basin in table-dependent woody vegetation in riparian Tamil Nadu the commercial production of areas, and decrease wetted periods in riparian mineral oil has started. wetlands. Hence statement 2 and 3 are  Petroleum or Mineral oil is obtained from correct. sedimentary rocks of the earth Educational Objective: To learn about the  India has 26 sedimentary basins covering possible consequences of heavy sand mining an area of 3.14 million square kilometres. in riverbeds. The sedimentary basins of India, on land and offshore up to the 400m isobath, have 6. Energy an aerial extent of about 1.84 million sq. km. 1. The dulhasti power station is based on  The on-shore Oil Production In India which one of the following rivers? includes. (a) Beas  Cauvery on-shore basin in Tamil Nadu. (b) Chenab  Gujarat coast in western India. (c) Ravi  of north-east India. (d) Sutlej  Barmer area of Rajasthan.  Andhra Pradesh has both on-shore and 2. Consider the following statements 1) India does not have any deposits of offshore oil reserves. thorium (2.009) Educational Objective: To learn about the 2) Kerala's monazite sands contain Uranium locations of mineral oil production. Which of the statements given above is/are 20. Key: B correct?

www.laexias.com Page No. 124 https://elearn.laex.in Geography UPSC Previous Year Questions

(a) 1 only Select the correct answer using the code (b) 2 only given below. (c) Both 1 and 2 (a) 1 & 2 only (d) Neither 1 nor 2 (b) 3 only (c) 2 & 3 only 3. Which of the following pairs in respect of (d) 1, 2 & 3 current power generation in India is/are correctly matched? (Rounded Figure) 7. On which one of the following rivers is 1) Installed electricity 100000 MW Generation the Tehri Hydropower Complex located? capacity (a) Alaknanda 2) Electricity generation 660 billion kWh (b) Bhagirathi Select the correct answer using the code (c) Dhauliganga given below: (d) Mandakini Codes: 8. Where are Tapovan and Vishnugarh (a) 1 only Hydroelectric Project located? (b) 2 only (a) Madhya Pradesh (c) Both 1 and 2 (b) Uttar Pradesh (d) Neither 1 nor 2 (c) Uttarakhand 4. With which one of the following rivers is (d) Rajasthan the Omkareshwar Project associated? 9. For which one of the following, is Satara (a) Chambal well-known? (b) Narmada (a) Thermal power plant (c) Tapi (b) Wind energy plant (d) Bhima (c) Hydro-electric plant 5. The thermal power plant of Bokaro is (d) Nuclear power plant located in: 10. Match List I (Atomic Power Plants/Heavy (a) Bihar Water Plants) with List II (State) and (b) Chhattisgarh select the correct answer using the code (c) Jharkhand given below the lists: (d) Orissa List-I (Atomic Power Plants/ Heavy Water 6. In which of the following regions of India Plants) List-II (State) are shale gas resources found? A. Thal 1. Andhra Pradesh 1) Cambay Basin B. Manuguru 2. Gujarat 2) Cauvery Basin C. Kakrapar 3. Maharashtra 3) Krishna-Godavari Basin D. Kaiga 4. Rajasthan 5. Karnataka

www.laexias.com Page No. 125 https://elearn.laex.in Geography UPSC Previous Year Questions

Codes: (a) A (a) A-2; B-1; C-4; D-5 (b) B (b) A-3; B-5; C-2; D-1 (c) C (c) A-2; B-5; C-4; D-1 (d) D (d) A-3; B-1; C-2; D-5

11. Match items in the List I (Power Station) 6. Key and Explanation with those in the List II (State) and select 1. Key: B the correct answer using the code given Explanation: DulHasti is a 390 MW below the lists: hydroelectric power plant in Kishtwar district of List-I List-II (State) Jammu and Kashmir, India built by NHPC. (Power Station) The power plant is a run-of-the-river type on A. Kothagudem 1. Andhra Pradesh Chandra River, a tributary of Chenab River, in B. Raichur 2. Gujarat the Kishtwar region, a rugged, mountainous C. Mettur 3. Karnataka section of the Himalayas, and several hundred D. Wanakbori 4. Tamil Nadu kilometers from larger cities Codes: (a) A-4; B-2; C-1; D-3 (b) A-1; B-3; C-4; D-2 (c) A-4; B-3; C-1; D-2 (d) A-1; B-2; C-4; D-3

12. Gandhi Sagar Dam is a part of which one of the following? (a) Chambal Project The Chenab river has rich power generation (b) Kosi Project potential in India. Following are the power (c) Damodar Valley Project projects related to Chenab river. (d) Bhakra Nangal Project ● ● Tarbela Dam 13. In the given figure shows a portion of ● DulHasti Hydroelectric Plant Southern India. The proposed site ● Pakal Dul DamRatle Hydroelectric Plant (Koodankulam) for the construction of ● Kiru Hydroelectric Power Project two 1000 MW nuclear power plants has ● Kwar Hydroelectric Power Project been labelled in the map as Educational Objective: To know about run of river project.

2. Key: D Explanation: There are two sets of estimates that define world thorium reserves, one set by

www.laexias.com Page No. 126 https://elearn.laex.in Geography UPSC Previous Year Questions

the United States Geological Survey (USGS) and (utilities and non utilities) in the country was the other supported by reports from the OECD 1,547 TWh. and the International Atomic Energy Agency (the Hence statement 2 is wrong. IAEA). Under the USGS estimate, India, the The gross electricity consumption in 2018-19 United States, and Australia have particularly was 1,181 kWh per capita.In 2015-16, electric large reserves of thorium. India and Australia energy consumption in agriculture was recorded are believed to possess about 300,000 tonnes as being the highest (17.89%) worldwide. each; i.e. each has 25% of the world's thorium Educational Objective: To know about energy reserves. Hence statement 1 is wrong. consumption. The minerals found in the Monazite sands of 4. Key: B Kerala is Thorium. Hence statement 2 is also Explanation: The Omkareshwar Dam is a wrong. gravity dam on the Narmada River just Thorium: Thorium is several times more upstream of Mandhata in Khandwa district, abundant in Earth’s crust than all isotopes of Madhya Pradesh, India. It is named after the uranium combined and thorium-232 is several Omkareshwar temple located just downstream. hundred times more abundant than uranium- The dam was constructed between 2003 and 235. 2007 with the purpose of providing water for Monazite – Rare Earth Metals: Monazite is a irrigation of 132,500 ha (327,000 acres). An reddish-brown phosphate mineral containing associated hydroelectric power station located at rare earth metals. Rare earths are a series of the base of the dam has an installed capacity of chemical elements found in the Earth’s crust 520 MW. that are vital to many modern technologies. Total Catchment Area at the Dam site is 64880 Educational Objective: To know about sq.km. Generation of power at Omkareshwar thorium reserves Project is directly related to the regulated release 3. Key: D of water from Indira Sagar Project, hence it is Explanation: India is the world's third largest one of the beneficiary project of Indira Sagar producer and third largest consumer of Project. electricity. The national electric grid in India has Educational Objective: To know about hydro an installed capacity of 368.79 GW as of 31 electric project. December 2019. Renewable power plants, which 5. Key: C also include large hydroelectric plants, Explanation: Bokaro Thermal Power Station is constitute 34.86% of India's total installed located at Bokaro district in Jharkhand. capacity. Hence statement 1 is wrong. Thermal Power Plants in India During the 2018-19 fiscal year, the gross electricity generated by utilities in India was 1,372 TWh and the total electricity generation

www.laexias.com Page No. 127 https://elearn.laex.in Geography UPSC Previous Year Questions

on the Bhagirathi River near Tehri in Uttarakhand, India. It is the primary dam of the THDC India Ltd. and the Tehri hydroelectric complex.

The BTPS has achieved a reputation of being the first and the largest thermal power station in Asia after its inauguration in 1953. Educational Objective: It is important to have a Educational Objective: To know about thermal power plants of India. glance at new hydro-electricity projects in news. 6. Key: D 8. Key: C Explanation: Explanation: The Tapovan-Vishnugad Hydropower Plant is a 520 MW run-of-river hydroelectric project being constructed on Dhauliganga River in Chamoli District of Uttarakhand, India. The plant is expected to generate over 2.5k GWh of electricity annually. Educational Objective: It is important to have a To strengthen its hydrocarbon resource base, glance at new hydro-electricity projects in news India has identified six basins as areas for shale 9. Key: B gas exploration: Cambay (Gujarat), Assam- Explanation: Satara well-known for Thermal Arakan (North East), Gondwana (Central India), power plant Hussain Sagar Thermal Power Krishna Godavari onshore (East Coast), Cauvery Station is the historic thermal power plant that onshore, and Indo-Gangetic basins. was located in Hyderabad, Telangana on the Shale gas is an unconventional source of energy banks of Hussain Sagar. It was India's first found in non-porous rocks. It has become an thermal power station, opened in 1920 by the important source of natural gas in the US. erstwhile seventh Nizam of Hyderabad. To know about hydro Educational Objective: Vindhyachal Thermal Power Station in the carbon resources. of Madhya Pradesh, with an 7. Key: B installed capacity of 4,760MW, is currently the Explanation: Tehri Hydropower is a multi- biggest thermal power plant in India. It is a coal- purpose rock and earth-fill embankment dam based power plant owned and operated by NTPC.

www.laexias.com Page No. 128 https://elearn.laex.in Geography UPSC Previous Year Questions

Hussain Sagar Thermal Power Station, Gangetic drainage system. The Chambal Telangana was India's first thermal power originates at janapav, south of Mhow town, near station. Indore. Gandhisagar Dam, the Rana Vindhyachal Thermal Power Station, Madhya PratapSagar Dam, the Jawahar Sagar Dam, Pradesh is currently the biggest. Kota Barrage are some of the major projects on Educational Objective: To know about thermal Chambal river. power project. Damodar Valley Project is on river Damodar. Damodar River originates at Chota Nagpur 63 Plateau in Jharkhand.it flows across the Indian 10. Key: D states of Jharkhand and West Bengal.Damodar Explanation: is a rift valley river Bhakra and Nangal Project is on the river Satluj. Educational Objective: To learn about the important river tributaries, geography of rivers and riverine dam projects.

13. Key: B Explanation: Point B in the map is kudankulam, As of March 2018,India has 22 nuclear Tamil Nadu. Kudankulam Nuclear Power Plant reactors in operation in 7 nuclear power plants, is the largest nuclear power station in India, with a total installed capacity of 6,780 MW. situated in Koodankulam in the Tirunelveli Educational Objective: To know about nuclear district of the southern Indian state of Tamil power plant. Nadu. Construction on the plant began on 31 11. Key: B March 2002. It is built in collaboration with Explanation: Kothagudem power station is Russia.s located in Andhra Pradesh.  Raichur power station is a coal based power plant, which is in karnataka.  Mettur power station is located in Salem district of Tamilnadu.  Wanakbori power plant is in Gujarat, which is on the bank of Mahi River. Educational Objective: power station location and economic geography of India Educational Objective: To know the various 12. Key: A nuclear and atomic plants in India and their Explanation: The River Chambal is a tributary locations. of River Yamuna and thus forms part of

www.laexias.com Page No. 129 https://elearn.laex.in Geography UPSC Previous Year Questions

(d) Exotic species invade the fertile soil of rain 7. Forest forest 1. Which of the following is/are unique 4. Consider the following statements: characteristic/ characteristics of equatorial forests? 1) Biodiversity is normally greater in the lower 1) Presence of tall closely set trees with crowns latitudes as compared to the higher forming a continuous canopy latitudes. 2) Coexistence of a large number of species 2) Along the mountain gradients, biodiversity 3) Presence of numerous varieties of epiphytes is normally greater in the lower altitudes as Select the correct answer using the codes compared to the higher altitudes. given below. Which of the statements given above is/are (a) 1 only correct? (b) 2 and 3 only (a) 1 only (c) 1 and 3 only (b) 2 only (d) 1,2 and 3 (c) Both 1 and 2 (d) Neither 1 nor 2 2. What is the difference between the 5. In India, which type of forest among the antelopes Oryx and Chiru? following occupies the largest area? (a) Oryx is adapted to live in hot and arid areas (a) Montane Wet Temperate Forest whereas Chiru is adapted to live in steppes (b) Sub-tropical Dry Evergreen Forest and semi-desert areas of cold high (c) Tropical Moist Deciduous Forest mountains (d) Tropical Wet Evergreen Forest (b) Oryx is poached for its antlers whereas Chiru is poached for its musk 6. From the point of view of evolution of (c) Oryx exists in western India only whereas living organisms, which one of the Chiru exists in north-east India only following is the correct sequence of (d) None of the statements (a), (b) and (c) given evolution? above is correct (a) Otter –Tortoise- Shark (b) Shark –Tortoise- Otter 3. If a tropical rain forest is removed, it (c) Tortoise – Shark- Otter does not regenerate quickly as compared (d) Shark -Otter - Tortoise to a tropical deciduous forest. This is because 7. Assertion: Unlike temperate forests, the (a) The soil of rain forest is deficient in tropical rain forests, if cleared, can yield nutrients productive farmland that can support (b) Propagules of the trees in a min forest have intensive agriculture for several years even poor viability without chemical . (c) The rain forest species are slow growing

www.laexias.com Page No. 130 https://elearn.laex.in Geography UPSC Previous Year Questions

Reason: The primary productivity of the (b) 2-3-4-1 tropical rain forest is very high when (c) 3-2-4-1 compared to that of temperate forests (d) 3-2-1-4 (a) Both A are R are true R is the correct 11. With reference to 'Red Sanders', explanation of A sometimes seen in the news, consider (b) Both A and R are true but R is not a correct the following statements: explanation of A 1) It is a tree species found in a part of South (c) A is true but R is false India (d) A is false but R is true 2) It is one of the most important trees in the 8. Consider the following eco systems: tropical rain forest areas of South India 1) Taiga Which of the statements given above is/are 2) Tropical evergreen correct? 3) Tropical deciduous (a) 1 only 4) Tundra (b) 2 only The correct sequence in decreasing order of (c) Both 1 & 2 the albedo values of these ecosystems is (d) Neither 1 nor 2 (a) 1, 4, 3, 2 12. Which one of the following is not (b) 2, 3 and 4 essentially a species of the Himalayan (c) 4, 1,3, 2 vegetation? (d) 1,2,3 and 4 (a) Juniper 9. Which one among the following covers (b) Mahogany the highest percentage of forest area in (c) Silver fir the world? (d) Spruce (a) Temperate coniferous forest 13. Which one of the following regions of (b) Temperate deciduous forests India has a combination of mangrove (c) Tropical monsoon forests forest, evergreen forest and deciduous (d) Tropical rain forests forest? 10. Consider the following States: (a) North Coastal Andhra Pradesh 1) Chhattisgarh (b) South-West Bengal 2) Madhya Pradesh (c) Southern Saurashtra 3) Maharashtra (d) Andaman and Nicobar Islands 4) Odisha 14. Consider the following States: With reference to the States mentioned 1) Arunachal Pradesh above, in terms of percentage of forest cover to the total area of State, which one of the 2) Himachal Pradesh following is the correct ascending order? 3) Mizoram (a) 2-3-1-4

www.laexias.com Page No. 131 https://elearn.laex.in Geography UPSC Previous Year Questions

In which of the above States do Tropical Wet (d) It has less human interference Evergreen forest occur? 18. The approximate representation of land (a) 1 only use classification in India is (b) 2 & 3 only (a) Net area soon 25 %; forests 33%; other (c) 1 & 3 only areas 42% (d) 1,2 & 3 (b) Net area soon 58 %; forests 17%; other 15. In India, in which one of the following areas 25% types of forests is teak a dominant tree (c) Net area soon 43 %; forests 29%; other species? areas 28% (a) Tropical moist deciduous forest (d) Net area soon 47 %; forests 23%; other (b) Tropical rain forest areas 30% (c) Tropical thorn scrub forest 19. Consider the following statements: (d) Temperate forest with grasslands 1) In India, Red Panda is naturally found in 16. If you travel through the Himalayas, you the Western Himalayas only. are likely to see which of the following 2) In India, Slow Loris lives in the dense plants naturally growing there? forests of the North East. 1) 1. Oak Which of the statements given above is/are 2) 2. Rhododendron correct? 3) 3. Sandalwood (a) 1 only Select the correct answer using the code (b) 2 only given below. (c) Both 1 & 2 (a) 1 & 2 only (d) Neither 1 nor 2 (b) 3 only 20. Amongst the following Indian States (c) 1 & 3 only which one has the minimum total forest (d) 1,2 & 3 cover? 17. The Himalayan Range is very rich in (a) Sikkim species diversity. Which one among the (b) Goa following is the most appropriate reason (c) Haryana for this phenomenon? (d) Kerala (a) It has a high rainfall that supports 21. Open stunted forests with bushes and luxuriant vegetative growth having long roots and sharp thorns or (b) It is a confluence of different spines are commonly found in biogeographical zones (a) Eastern Orissa (c) Exotic and invasive species have not been (b) North-Eastern Tamil Nadu introduced in this region (c) Siwaliks and regions

www.laexias.com Page No. 132 https://elearn.laex.in Geography UPSC Previous Year Questions

(d) Western Andhra Pradesh they are seen in North Bengal, Assam and a few other states." The animal 'X' 22. Match List-I (Mangrove) with List-II (State) referred to in this quotation is and select the correct answer: (a) Lion List-I List-II (b) Elephant 1) Achra Ratnagiri Karnataka (c) Tiger 2) Coondapur Kerala (d) One-horned rhinoceros 3) Pichavaram Andhra Pradesh 4) Vembanad Maharashtra 25. Consider the following statements: 5) Tamil Nadu 1) The forest cover in India constitutes around Codes: 20% of its geographical area. Out of the total (a) A-2; B-1; C-5; D-4 forest cover, dense forest constitutes around (b) A-4; B-5; C-3; D-2 40%. (c) A-2; B-5; C-3; D-4 2) The National Forestry Action Programme (d) A-4; B-1; C-5; D-2 aims at bringing one-third of the area of India under tree forest cover. 23. The minimum land area recommended Which of the statements given above is/are for forest cover to maintain proper correct? ecological balance in India is (a) 1 only (a) 25% (b) 2 only (b) 33% (c) Both 1 and 2 (c) 43% (d) Neither 1 nor 2 (d) 53% 26. Forest areas have been labelled as 1, 2, 3 24. "India has the largest population of the and 4 in the rough outline map given: Asian X. Today, there are just about 20,000 to 25,000 X in their natural habitat spreading across the evergreen forests, dry thorn forests, swamps and grasslands. Their prime habitats are, however, the moist deciduous forests. The X population in India ranges from North-West India where they are found in the forest divisions of Dehradun, Bijnor and Nainital districts of UP to the Among these, those which were threatened in Western Ghats in the states of Karnataka 1997 by a serious epidemic include and Kerala and in Tamil Nadu. In Central (a) Teak forests of 3 and 4 India, their population is distributed in (b) Oak forests of 1 and Sal forests of 2 southern Bihar and Orissa. In the East,

www.laexias.com Page No. 133 https://elearn.laex.in Geography UPSC Previous Year Questions

(c) Sal forests of 3 They do not harm the trees and they do not get (d) Sandalwood forests of 4 their nutrients from them. They only use the

27. Consider the following rivers : trees for physical support. Hence, statement 3 is 1) Barak correct 2) Lohit Educational Objective: To know about the 3) Subansiri vegetation of Equatorial forest. Which of the above flows / flow through Arunachal Pradesh? 2. Key: A (a) 1 only Explanation: Chiru (b) 2 & 3 only Is also called as Tibetan goat, whose underfur is (c) 1 & 3 only used for making the famous Shahtoosh shawls. (d) 1,2 & 3 It is placed under ‘Near Threatened’ by the International Union for Conservation of Nature 7. Key and Explanation and is Endemic to the Tibetan Plateau, the 1. Key: D Tibetan antelope inhabits open alpine and cold Large number of species of trees Explanation: steppe environments between 3,250 and 5,500 occurs in a particular area (trees do not occur in m (10,660 and 18,040 ft) elevation. From homogenous stands or pure stands). That is why, Ladakh, the chiru's distribution extends 1600 countries in equatorial region import timber km (990 mi) eastward across Tibet. despite having dense forests. Hence, statement 2 Oryx: Oryx species prefer near-desert conditions is correct and can survive without water for long periods. The uppermost layer of rainforest is made up of The horns are narrow, and straight mostly, the crowns of the tallest trees with average poaching is leading to their extinction. Hence, height 45 – 60 meters are known as emergent, statement 1 is correct with their crowns exposed to the direct sunlight. Educational Objective: To know about The next layer is known as the canopy with an antelopes – Oryx and Chiru. average height of about 20 to 40 meters.The 3. Key: A crowns of the trees in this layer are tightly Explanation: In the tropical rain forests, packed together forming a nearly unbroken Torrential downpours wash out most of the top cover for the forest floor. Hence, statement 1 is soil nutrients and the fertility of top soil in correct rainforest regions is very poor. It takes decades Some plants, known as epiphytes, grow on trees to replenish the soil of lost nutrients. The (their roots are not in the soil) for sunlight.

www.laexias.com Page No. 134 https://elearn.laex.in Geography UPSC Previous Year Questions

development of plant is also hindered due to Educational Objective: To learn about spread little availability of sunlight. of various forests in India

Where as in tropical deciduous forests rainfall is 6. Key: B comparatively lower and enough light reaches Self Explanatory the ground to permit the growth of grass and 7. Key: D climbers. Explanation: The fertility of top soil in rainforest Educational Objective: To learn about the regions is very poor. Torrential downpours wash tropical rain forests. out most of the top soil nutrients. The soil 4. Key: C deteriorates rapidly with subsequent soil erosion Explanation: Biodiversity means number and and soil impoverishment. It takes decades to variety of plants and animals. As you move from replenish the soil of lost nutrients. Hence, equator towards the poles, biodiversity decreases. Assertion is wrong. Biodiversity is normally greater in the lower In terrestrial ecosystems the order of latitudes as compared to the higher latitudes. productivity in decreasing order is 1. Swamps, Hence, statement 1 is correct marshes, tropical rain forests (most productive), Biodiversity is normally greater in the lower 2.Temperate forest, 3.Northern coniferous forest altitudes as compared to the higher altitudes. (taiga), 4. Savanna, 5.Agricultural land, The higher the altitude, colder is the atmosphere 6.Woodland and shrub land, 7.Temperate and resultant less biodiversity. Hence, statement grassland, 8.Tundra (arctic and alpine), 9.Desert 2 is correct. scrub, 10. Extreme desert (least productive). Educational Objective: To know about Hence, Reason is correct biodiversity and its features Educational Objective: To learn about the 5. Key: C productivity of Tropical rain forests and Explanation: Forest Type in India. % of Total Temperate forests Area 8. Key: C Tropical Moist Deciduous 37 Explanation: Albedo is the portion of solar Tropical Dry Deciduous 28 energy reflected from the surface of the Earth 8 Tropical Wet Evergreen. back into space. It is a reflection coefficient and Sub-Tropical Moist Hill. 6 has a value of less than one. Tropical Semi-Evergreen 4 Rest below 4 % Hence, statement 3 is correct.

www.laexias.com Page No. 135 https://elearn.laex.in Geography UPSC Previous Year Questions

Table-1: Reflectivity Values of Various 9. Key: A Surfaces Explanation: Temperate Coniferous forests Surface Details Albedo cover the highest percentage of forest rate in the Dark and Wet 0.05 world. Temperate coniferous forest is a Soil Light and dry 0.40 terrestrial biome found in temperate regions of the world with warm summers and cool winters Sand 0.15 – 0.45 and adequate rainfall to sustain a forest. Long 0.16 Coniferous forests are of moderate density and Gross Short 0.26 are more uniform. The trees in coniferous forests

Crops 0.18 – 0.25 grow straight and tall. Almost all conifers are evergreen. There is no annual replacement of Tundra 0.18 – 0.25 new leaves as in deciduous trees. Deciduous 0.15 – 0.20 Educational Objective: To know about Forests Coniferous 0.05 – 0.15 distribution of Forest types in the world.

Small zenith 10. Key: C 0.03 – 0.10 angle Explanation: Forest area refers to land area Water Large zenith covered by forests or forest canopy or open 0.10 – 1.0 angle woodland. Among the given state Maharashtra has lowest Old 0.40 Snow forest cover due to it’s urbanization and dry Fresh 0.95 weather conditions which cannot support Sea 0.30 – 0.45 luxuriant vegetation. Ice Glacier 0.20 – 0.40 Chhattisgarh and Odisha once supported diverse vegetation but most of forests have been Thick 0.60 – 0.90 Clouds cleared for mining as forest there represent rich Think 0.30 – 0.50 source of coal and iron. When the solar radiation passes through the Madhya Pradesh has preserved its forests due to atmosphere, a certain amount of it is scattered, large tribal population and thus Madhya reflected and absorbed. The reflected sum of Pradesh has largest forest area under forests in radiation is called the albedo of the earth. India. Educational Objective: To learn about the albedo values of the ecosystem.

www.laexias.com Page No. 136 https://elearn.laex.in Geography UPSC Previous Year Questions

Total State/Union Geographi Very Moderatel Open % change since Forest % Territory cal Area Dense y Dense Forest 2015 Area Andaman & Nicobar 8,249 5,678 684 380 6,742 81.73 -0.11% Islands Andhra Pradesh 162,968 1,957 14,051 12,139 28,147 17.27 +1.31% Arunachal Pradesh 83,743 20,721 30,955 15,288 66,964 79.96 -0.23% Assam 78,438 2,797 10,192 15,116 28,105 35.83 +0.72% Bihar 94,163 332 3,260 3,707 7,299 7.75 +0.05% Chandigarh 114 1 14 6 22 18.91 -0.09% Chhattisgarh 135,192 7,064 32,215 16,268 55,547 41.09 -0.01% Dadra & Nagar Haveli 491 0 80 127 207 42.16 +0.20% Daman & Diu 111 1 6 13 20 18.46 +0.79% Delhi 1,483 7 56 129 192 12.97 +0.25% Goa 3,702 538 576 1,115 2,229 60.21 +0.51% Gujarat 196,244 378 5,200 9,179 14,757 7.52 +0.02% Haryana 44,212 28 452 1,108 1,588 3.59 +0.02% Himachal Pradesh 55,673 3,110 6,705 5,285 15,100 27.12 +0.71% Jammu & Kashmir 222,236 4,075 8,579 10,587 23,241 10.46 +0.11% Jharkhand 79,716 2,598 9,686 11,269 23,553 29.55 +0.04% Karnataka 191791 4,502 20,444 12,604 37,550 19.58 +0.57% Kerala 38,852 1,663 9,407 8,251 20,321 52.30 +2.68% Lakshadweep 30 0 17 10 27 90.33 0.13% Madhya Pradesh 308,252 6,563 34,571 36,280 77,414 25.11 -0.00% Maharashtra 307,713 8,736 20,652 21,294 50,682 16.47 -0.01% Manipur 22,327 908 6,510 9,928 17,346 77.69 +1.18% Meghalaya 22,429 453 9,386 7,307 17,146 76.76 -0.52% Mizoram 21,081 131 5,861 12,194 18,186 86.27 -2.52% Nagaland 16,579 1,279 4,587 6,623 12,489 75.33 -2.71% Odisha 155,707 6,967 21,370 23,008 51,345 32.98 +0.57% Puducherry 490 0 18 36 54 10.95 -0.67% Punjab 50,362 8 806 1,023 1,837 3.65 +0.13% Rajasthan 342,239 78 4,340 12,154 16,572 4.84 +0.14% Sikkim 7,096 1,081 1,575 688 3,344 47.13 -0.13 Tamil Nadu 130,060 3,672 10,979 11,630 26,281 20.21 +0.06% Telangana 112,077 1,596 8,738 10,085 20,419 18.22 +0.50% Tripura 10,486 656 5,246 1,824 7,726 73.68 -1.56% Uttar Pradesh 240,928 2,617 4,069 7,993 14,679 6.09 +0.12% Uttarakhand 53,483 4,969 12,884 6,442 24,295 45.43 +0.04% West Bengal 88,752 2,994 4,147 9,706 16,847 18.98 +0.02% Total 3,287,469 98,158 308,318 301,797 708,273 21.54 +0.21%

www.laexias.com Page No. 137 https://elearn.laex.in Geography UPSC Previous Year Questions

Educational Objective: To know the South west Bengal also supports only deciduous distribution of different forest and their location and mangrove forest and not ever green as it and to achieve sustainable forest cover of 33% in also lacks moisture required for it sustainance. all states Southern saurashtra is one of the driest parts of peninsula which supports only scrub type of 11. Key: A vegetation and few deciduous trees. Explanation: Red sanders also called Rakth Andaman and Nicobar Islands with it’s location chandan is the tropical deciduous tree of south near the equator provides the ideal conditions Indian specillay known for it’s wood and for all the three types of forest thus providing for medicinal importance. increase diversity of species. Distribution: Seshachalam forest, palkonda rage Hence D is the only correct option and various range of eastern ghats. To study different It is used as a herbal medicine for antipyretic, Educational Objective: geographical regions of India and their flora and inflammatory, tonic, haemorrage, dysentery, fauna. aphrodisiac and diaphoretic porposes. Hence only statement 1 is correct as Red 14. Key: C sanders is endemic to deciduous forest of south Explanation: Arunachal Pradesh and Mizoram India being located in the north east of India provide Educational Objective: To known about the ideal condition for the development of moist endemic plants species of different regions in evergreen forest due to ready availability of India. moisture all around the year. Some trees of wet evergreen forests are: 12. Key: B Rhododendron, magnolia, cinnamomum etc Explanation: Himalayan vegetation is mainly Himachal Pradesh even though being located in composed of coniferous species. They Occur in the Himalayas cannot support wet ever green the temperate zone of the Himalayas between forests due to average moisture because of it’s 1500 and 3300 metres.Trees are 30 to 50 m distance from the sea. high. Eg.Pines, juniper, cedars, silver firs, spruce, etc. are most important trees. Some dry evergreen trees of western Himalayas: oaks, Quercus etc Mahogany species is found in Indian Tropical forests Hence option c is correct answer To study different Educational Objective: To know about the Educational Objective: geographical regions of India and their flora and Himalayan vegetation. fauna. 13. Key: D 15. Key: A Explanation: North coastal Andhra Pradesh have only mangrove and deciduous forest and Explanation: Teak is one of the important cannot support evergreen due to not enough species of moist deciduous forest of south India moisture.

www.laexias.com Page No. 138 https://elearn.laex.in Geography UPSC Previous Year Questions

in the state of Andhra Pradesh, Tamilnadu and deciduous forests. It provides diverse habitats Karnataka. for a range of species including endangered ones It has leather like smell when freshly milled and such as Hangul and Musk Deer. The is valued for it’s durability and water resistance. transHimalaya zone includes high altitude cold It is the commonly used wood for furniture and and arid mountain areas, including cold Burma is largest producer of it. deserts.Thus, statement 2 is the most Tropical rain forest do not support teak due to appropriate answer. its heavy rainfall which is not feasible for it’s Educational Objective: To know about the growth. diversity of Himalayan range and reason for it.

Tropical thorn and grasslands cannot support 18. Key: D large trees like teak which can grow upto 70 feet Explanation: Net Sown Area is 47% of the total due to their dry and scrub vegetation. geographic area because of extensive availability Hence option a is the most feasible answer of flat terrain in India. Educational Objective: To study different forest About 23% area of the country is under forest types of India and the species that is supported cover. by them. Barren and un-culturable waste land amounts 16. Key: A to about 8.5%. Explanation: Himalayan region is known for its About 5.5% is under non-agricultural uses like diverse landscape and beauty. Due to this houses, industries etc. diverse it supports diversity of forests from Rest of the area is under tree crops, grooves, tropical to Tundra vegetation as we scale the permanent pastures and grazing lands etc. mountains until permanent snowline appears. Rhododendron is one of the most important tree of evergreen forests of Himalayas and oak is distributed both as evergreen and deciduous forests of Himalayas. Sandal wood tree is completely absent in Himalayas as it represent the tropical deciduous forests of central India. Educational Objective: To learn about the land Educational Objective: To study different forest use classification in India types of India and the species that is supported by them. 19. Key: B Explanation: Red Panda 17. Key: B The red panda is a small reddish-brown arboreal Explanation: Himalayan range covers 6.4 % of mammal. the total geographical area and has alpine and sub-alpine forests, grassy meadows and moist

www.laexias.com Page No. 139 https://elearn.laex.in Geography UPSC Previous Year Questions

It is also the state animal of Sikkim and listed as 21. Key: D Endangered in the IUCN red list of Threatened Explanation: The western Andhra Pradesh Species and under Schedule I of the Indian region lies in the rain shadow region of both Wildlife (Protection) Act, 1972. Western Ghats and Eastern Ghats. As a result, It is found in the forests of India, Nepal, Bhutan the Districts of Anantapur, kurnoolare and the northern mountains of Myanmar and characterized by tropical thorn forests. southern China. Characteristics of Tropical thorn forests It thrives best at 2,200-4,800m, in mixed Annual rainfall is less than 75 cm. deciduous and conifer forests and present in Humidity is less than 50 per cent. Sikkim, western Arunachal Pradesh, Darjeeling Mean temperature is 25°-30°C. district of West Bengal and parts of Meghalaya. The trees are low (6 to 10 metres maximum) and Slow Lorises: Slow lorises are found in South widely scattered. Mostly, open stunted forests and . Their collective range with bushes and having long roots and sharp stretches from Northeast India through thorns or spines are commonly found. Indochina, east to the Sulu Archipelago (the Educational Objective: To know about Tropical small, southern islands of the Philippines), and thorn forests and their distribution. south to the island of Java (including Borneo, Sumatra, and many small nearby islands). 22. Key: D Educational Objective: To learn about the Explanation: habitats of Red panda and slow loris. List-I (Mangroves) List-II (States) A. Achra Ratnagiri Maharashtra 20. Key: C B. Coondapur Karnataka Explanation: According to Indian state of Forest C. Pichavaram Tamil Nadu report, 2019 D. Vembanad Kerala Haryana has Total forest cover is very low at Mangroves— Tolerant to salt water and 3.62% anaerobic condition. Littoral plants and Total Forest cover of Goa is 60.44% subtropical coastlines grow on tidal flats, deltas, Total Forest cover of kerala is 54.42% estuaries, bays, creeks and Barrier Island. It Total forest cover of Sikkim 47.11% produces pneumatophores to overcome Largest forest cover in India: Madhya Pradesh > respiration problem in the anaerobic soil Arunachal Pradesh > Chhattisgarh > Odisha > condition. It can absorb freshwater from saline Maharashtra and brackish water. Mangroves exhibit viviparity Forest cover as percentage of total geographical modes of reproduction. Mangroves secrete salts area: Mizoram (85.41%) > Arunachal Pradesh through their leaves. In India all coastal states (79.63%) > Meghalaya (76.33%) > Manipur has mangroves. (75.46%) > Nagaland (75.31%). Educational Objective: Geography of India and Educational Objective: Forest cover of India. plant vegetation

www.laexias.com Page No. 140 https://elearn.laex.in Geography UPSC Previous Year Questions

Appendix I of the Convention on International 23. Key: B Trade in Endangered Species of Wild Fauna and Explanation: Ecological balance is a term used Flora (CITES) and Schedule I of the Wildlife to describe the equilibrium between living (Protection) Act, 1972. organisms such as human being, plants, and Educational Objective: To know about the animals as well as their environment. distribution of Asian Elephant. At least 33% of the land area of a country should be under the forest. This is the ideal balance for 25. Key: B maintaining ecological balance in the Explanation: The total forest cover of the environment. country is 7,12,249 sq km which is 21.67% of If the balance is not maintained there is high the geographical area of the country. The Tree chance of suffering from natural calamities like cover is 2.89% of the geographical area of the irregular rain, drought, flood, and loss of flora country. and fauna etc., All lands with tree canopy density of 70% and Educational Objective: To know about the above are considered as Very Dense Forest. Out concept of ecological balance of the total forest cover, dense forest constitutes around 3.02%. Hence, statement 1 is wrong. 24. Key: B National Forestry Action Programme (NFAP) is a Explanation: The X referred in the question is comprehensive workplan for sustainable Asian Elephant development of forests in the country .to achieve India is the natural home of the largest the national goal of 33% geographic area of the population of Asian elephants. It is also found in country under the forest and tree cover as Nepal, Bangladesh, Bhutan and Myanmar. enshrined in the National Forest Policy, 1988. It usually resides in shrublands, Hence, statement 2 is correct. artificial/terrestrial forests and grasslands. Educational Objective: To learn about forest cover, percentage of very dense forest and National Forestry Action Programme.

26. Key: C Explanation: In 1997 Sal forest were badly affected by the insects which cause destruction of Sal forest by turning the trees dry. Sal borer attack began in 1995 and by 1997 it turned into a serious epidemic affecting the Sal forests. This majorly occurred in Madhya Pradesh.

It is listed as ‘Endangered’ on the IUCN Red List of threatened species. It is also listed in

www.laexias.com Page No. 141 https://elearn.laex.in Geography UPSC Previous Year Questions

8. Agriculture

1. Consider the following pairs: Region Well-known for the production of 1) Kinnaur A. Areca nut 2) Mewat B. Mango 3) Coromandel C. Soya bean Which of the above pairs is/are correctly matched? (a) 1 and 2 only (b) 3 only (c) 1, 2 and 3 (d) None The map shows the Sal forest distribution in 2. Assertion: There are no tea plantations in India any African country. Educational Objective: To be aware of serious Reason: Tea plants need fertile soil with environmental degradation issues, especially high humus. forests and water bodies in India and across the (a) Both A are R are true R is the correct world. explanation of A 27. Key: B (b) Both A and R are true but R is not a correct Explanation: Barak, Lohit and Subansiri are explanation of A the tributaries of Brahmaputra river. Only Lohit (c) A is true but R is false and Subansiri flows through the state of (d) A is false but R is true Arunachal Pradesh. 3. Though coffee and tea both are cultivated Whereas the Barak River flows through the on hill slopes, there is some difference states of Manipur, Nagaland, Mizoram and between them regarding their cultivation. Assam in India and into the Bay of Bengal via In this context, consider the following Bangladesh. statements: Dibang, Kameng, Lohit, Manas River, Subansiri 1) Coffee plant requires a hot and humid River, Teesta River and Kopili River are the other climate of tropical areas whereas tea can be tributaries of Brahmaputra. cultivated in both tropical and subtropical Educational Objective: To learn about the areas. tributaries of Brahmaputra river and its flow in 2) Coffee is propagated by seeds but tea is Arunachal Pradesh. propagated by stem cuttings only.

www.laexias.com Page No. 142 https://elearn.laex.in Geography UPSC Previous Year Questions

Which of the statements given above is/are Select the correct answer using the code correct? given below (a) 1 only (a) 1 and 2 only (b) 2 only (b) 3 only (c) Both 1 and 2 (c) 1 and 3 only (d) Neither 1 nor 2 (d) 1,2 and 3

4. Consider the following statements: 7. Why does the Government of India 1) Chikmagalur is well-known for sugar promote the use of “Neem- coated Urea” production. in agriculture? 2) Mandya is well- known as a coffee (a) Release of Neem Oil in the soil increases producing region. nitrogen fixation by the soil microorganisms. Which of the statements given above is/are (b) Neem coating slows down the rate of correct? dissolution of urea in the soil. (a) 1 only (c) Nitrous oxide, which is a greenhouse gas, is (b) 2 only not at all released into atmosphere by crop (c) Both 1 and 2 fields. (d) Neither 1 nor 2 (d) It is a combination of a weedicide and a for particular crops. 5. Match List- I with –II and select the correct answer using the code given 8. Which of the following is/ are the below the lists: advantage/ advantages of practising drip List-I (Board) List-II (Headquarters) irrigation? A. Coffee Board 1. Bengaluru 1) Reduction in weed B. Rubber Board 2. Guntur 2) Reduction in soil salinity C. Tea Board 3. Kottayam 3) Reduction in soil erosion D. Tobacco Board 4. Kolkata Select the correct answer using the code Codes: given below (a) A-2; B-4; C-3; D-1 (a) 1 and 2 only (b) A-1; B-3; C-4; D-2 (b) 3 only (c) A-2; B-3; C-4; D-1 (c) 1 and 3 only (d) A-1; B-4; C-3; D-2 (d) None of the above is an advantage of practising drip irrigation 6. Which of the following practices can help in water conservation in agriculture? 9. Consider the following statements: 1) Reduced or zero tillage of the land 1) Maize can be used for the production of 2) Applying gypsum before irrigating the field starch 3) Allowing crop residue to remain in the field

www.laexias.com Page No. 143 https://elearn.laex.in Geography UPSC Previous Year Questions

2) Oil extracted from maize can be feedstock (c) 1,2, and 3 for biodiesel (d) 2, 3 and 4 3) Alcoholic beverages can be produced by 12. Which of the following is the chief using maize characteristic of “mixed farming”? Which of the statements given above is/are (a) Cultivation of both cash crops & food crops correct? (b) Cultivation of two or more crops in the same (a) 1 only field (b) 1 and 2 only (c) Rearing of animals & cultivation of crops (c) 2 and 3 only together (d) 1,2 and 3 (d) None of the above 10. What are the significances of a practical 13. Consider the following crops of India. approach to sugarcane production known 1) Groundnut. as “Sustainable Sugarcane Initiative”? 2) Sesamum 1) Seed cost is very low in this compared to the 3) Pearl Millet conventional method of cultivation Which of the above is/ are predominantly 2) Drip irrigation can be practiced very rainfed crop/crops? effectively in this (a) 1 and 2 only 3) There is no application of (b) 2 and 3 only chemical/inorganic fertilizers at all in this (c) 3 only 4) The scope for intercropping is more in this (d) 1,2 and 3 compared to the conventional method of cultivation. 14. With reference to micro- irrigation, which of the following statements is / are Select the correct answer using the code correct? given below. 1) Fertilizer/Nutrient loss can be reduced (a) 1 and 3 only 2) It is only means of irrigation in dry land (b) 1, 2 and 4 only farming (c) 2,3 and 4 only 3) In some areas of farming, receding of (d) 1,2,3 and 4 ground water table can be checked Select the correct answer using the codes 11. Consider the following crops: given below: 1) Cotton (a) 1 only 2) Groundnut (b) 2 and 3 only 3) Rice (c) 1 and 3 only 4) Wheat (d) 1,2 and 3 Which of these are Kharif crops? (a) 1 and 4 (b) 2 and 3 only

www.laexias.com Page No. 144 https://elearn.laex.in Geography UPSC Previous Year Questions

15. Salinization occurs when the irrigation (d) A is false but R is true water accumulated in the soil evaporates, 19. Consider the following crops: leaving behind salts & minerals. What are the effects of salinization on the 1) Cotton irrigated land? 2) Groundnut (a) It greatly increases the crop production 3) Maize (b) It makes some soils impermeable 4) Mustard (c) It raises the water table Which of the above are Kharif crops? (d) It fills the air spaces in the soil with water (a) 1 and 2

16. In India, during the last decade the total (b) 1, 2 and 3 cultivated land for which one of the (c) 3 and 4 following crops has remained more or less (d) 1.2.3 and 4 stagnant? 20. Assertion: India does not export natural (a) Rice rubber. (b) Oil Seeds Reason: About 97% of India’s demand for (c) Pulses natural rubber is met from domestic (d) Sugarcane production Choose the correct answer 17. Which one of the following is the correct (a) Both A and R are correct and R is correct sequence in the decreasing order of explanation for A production (in million tones) of the given (b) Both A and R are correct and R is not food grains in India? correct explanation for A (a) Wheat- Rice- Pulses- Coarse Cereals (c) A is true but R is false (b) Rice- Wheat- Pulses- Coarse Cereals (d) A is false but R is true (c) Wheat- Rice- Coarse Cereals-Pulses (d) Rice- Wheat- Coarse Cereals- Pulses 21. Assertion: The Eastern Coast of India produces more rice than the Western Coast. 18. Assertion (A): The percentage of net sown Reason: The Eastern Coast receives more area in the total area of Andhra Pradesh is rainfall than the Western Coast. less as compared to that of West Bengal. Choose the correct answer Reason(R): The soil of most of Andhra (a) Both A and R are correct and R is correct Pradesh is laterite. explanation for A Choose the correct answer (b) Both A and R are correct and R is not (a) Both A and R are correct and R is correct correct explanation for A explanation for A (c) A is true but R is false (b) Both A and R are correct and R is not (d) A is false but R is true correct explanation for A (c) A is true but R is false

www.laexias.com Page No. 145 https://elearn.laex.in Geography UPSC Previous Year Questions

22. Consider the following statements. (a) Maharashtra, U.P, Tamil Nadu, Andhra 1) India is the original home of the cotton plant Pradesh 2) India is the first country in the world to (b) U.P, Maharashtra, Tamil Nadu. Andhra develop hybrid cotton variety leading to Pradesh increased production (c) Maharashtra, U.P, Andhra Pradesh, Tamil Which of these statements is/ are correct? Nadu (a) Only 1 (d) U.P, Maharashtra, Andhra Pradesh, Tamil (b) Only 2 Nadu (c) Both 1 and 2 26. Consider the following statements: (d) Neither 1 nor 2 1) Maharashtra has the highest acreage under 23. Consider the following high yielding Jawar in India varieties of crops in India 2) Gujarat is the largest producer of groundnut 1) Arjun in India. 2) Jaya 3) Rajasthan has the largest area of the 3) Padma cultivable waste lands in India 4) Sonalika 4) Andhra Pradesh has the highest per hectare Which of these are Wheat? yield of maize in India (a) 1 and 2 Which of these statements are correct? (b) 2 and 3 (a) 1 and 4 (c) 1 and 4 (b) 2 and 3 (d) 3 and 4 (c) 1 and 3 (d) 2 and 4 24. With reference to Indian agriculture, which one of the following statements is 27. Match List I with List II and select the correct? correct answer using the codes given (a) About 90% of the area under pulses in India below the lists: is rainfed List-I List-II (b) The share of pulses in the gross cropped A. Cotton 1. Rainfall 1000-1500mm: area at the National level has doubled in the Temp.40 _60 C last 2 decades B. Flax 2. Rainfall 1500-2000mm: (c) India accounts for about 15 percent of the Temp, 25_35C total area are under rice in the world C. Sugarbeet 3. Rainfall 600-800mm: (d) Rice occupies about 34 percent of the gross Temp 5_ 18 C cropped area in India. D. Jute 4. Rainfall 500-1000mm:

25. The correct sequence in decreasing Temp 18_22 C order of the four sugarcane producing 5. Rainfall 500-600mm: states in India is. Temp 18 _ 22 C

www.laexias.com Page No. 146 https://elearn.laex.in Geography UPSC Previous Year Questions

Codes: 31. The lower Gangetic plain is characterised (a) A-1; B-3; C-4; D-2 by humid climate with high temperature (b) A-2; B-3; C-5; D-4 throughout the year. Which one among (c) A-4; B-5; C-2; D-1 the following pairs of crops is most (d) A-4; B-3; C-5; D-2 suitable for this region? (a) Paddy and Cotton 28. Which one of the following agricultural (b) Wheat and Jute practices is eco-friendly? (c) Paddy and Jute (a) Organic farming (d) Wheat and cotton (b) Shifting cultivation (c) Cultivation of high yielding varieties 32. Consider the following statements: (d) Growing plants in glass- houses 1) India is the only country in the world producing all the five known commercial 29. Match List I with List II and select the varieties of silk correct answer using the codes given 2) India is the largest producer of sugar in the below the lists: world. List-I List-II Which of the statements given above is/are A. Cotton 1. Madhya Pradesh correct? B. Gram 2. Gujarat (a) 1 only C. Black Pepper 3. West Bengal (b) 2 only D. Pineapple 4. Kerala (c) Both 1 and 2 Codes: (d) Neither 1 nor 2 (a) A-2; B-1; C-4; D-3 33. Among the following States, which one (b) A-2; B-1; C-3; D-4 has the most suitable climatic conditions (c) A-1; B-2; C-4; D-3 for the cultivation of a large variety of (d) A-1; B-2; C-3; D-4 orchids with minimum cost of production, 30. Consider the following crops of India: and can develop an export oriented 1) Cowpea industry in this field? (a) Andhra Pradesh 2) Green gram (b) Arunachal Pradesh 3) Pigeon pea (c) Madhya Pradesh Which of the above 1s/are used as pulse, (d) Uttar Pradesh fodder green manure? (a) 1 &2 only 34. Dalbergia species is associated with (b) 2 only which one of the following? (c) 1 & 3 only (a) Cashew nut (d) 1,2 &3 (b) Coffee (c) Tea (d) Rosewood

www.laexias.com Page No. 147 https://elearn.laex.in Geography UPSC Previous Year Questions

Mango:The main mango producing states in 35. Following are the characteristics of an India are Uttar Pradesh (23.86%), Andhra area in India: Pradesh (22.14%), Karnataka, Bihar etc., not 1) Hot and humid climate Mewat – Haryana. Hence, statement 2 is wrong. 2) Annual rainfall 200 cm Soyabean:Top two producers of soyabean are 3) Hill slopes up to an altitude of 1100 metres Madhya Pradesh and Maharashtra not 4) Annual range of temperature 15°C to 30°C Coromandel (Andhra Pradesh and Tamil Nadu). Which one among the following crops are you Hence, statement 3 is wrong. most likely to find in the area described Educational Objective: To know about the well- above? known agricultural production areas of India (a) Mustard region wise. (b) Cotton (c) Pepper 2. Key: D (d) Virginia tobacco Explanation: There are Tea Plantations in many African countries such as Rwanda, South Africa, 36. What are the benefits of implementing Uganda, Tanzania etc., Hence, Assessment is the 'Integrated Watershed Development false. Programme'? Required Conditions for Tea Growth: 1) Prevention of soil runoff Climate: Tea is a tropical and sub-tropical plant 2) Linking the country's perennial rivers with and grows well in hot and humid climate. seasonal rivers Soil: The most suitable soil for tea cultivation is 3) Rainwater harvesting and recharge of slightly acidic soil (without Calcium) with porous groundwater table sub-soil which permits a free percolation of 4) Regeneration of natural vegetation water with high fertility and humus content. Select the correct answer using the code Hence, Reasonis correct. given below. Temperature: The ideal temperature for its (a) 1 & 2 only growth is 20°-30°C and temperatures above (b) 2, 3 & 4 only 35°C and below 10°C are harmful for the bush. (c) 1, 3 & 4 only Rainfall: It requires 150-300 cm annual rainfall (d) 1,2,3 & 4 which should be well distributed throughout the year. 8. Key and Explanation Educational Objective: To know about tea cultivation conditions and its distribution in the 1. Key: D world Explanation: Arcea nut: Areca nut Production is mostly confined to 3. Key: A Karnataka, Kerala and Assam. Whereas Kinnaur Explanation: Coffee and tea are the most Himachal Pradesh. Hence, statement 1 is wrong. important beverage crops and also commercial

www.laexias.com Page No. 148 https://elearn.laex.in Geography UPSC Previous Year Questions

crops which have huge demand in the world and also has one of the largest sugarcane mills market. In India both crops are grown with in Karnataka. Karnataka accounting for 70% of coffee Hence both the statements are incorrect production and Assam accounting for 52% of Educational Objective: to know about different total tea produced in India. crops cultivated in India and the important Even though both are grown on hilly slopes are places associated with their cultivation. is certain difference in distribution in India. 5. Key: B Coffee is mostly concentrated in the southern Explanation: The Coffee Board of India is an states of Karnataka, TamilNadu and Kerala as it organisation managed by the Ministry of requires ample sunlight and also frequent Commerce and Industry of the government of rainfall with out any stagnation which is mostly India to promote coffee production in India. available in tropical areas. Head Office is in Bangalore. Its tradition duties Tea on other hand is cultivated both in tropical included the promotion of the sale and and sub-tropical areas due to it’s ability to with consumption of coffee in India and abroad, strand more low temperature as compared to conducting coffee research, financial assistance coffee. And aslo it requires heavy rainfall of 150 to establish small coffee growers, safeguarding cm which is available in subtropical climates. working conditions for labourers, and managing Distribution: Karnataka, Assam, West Bengal. the surplus pool of unsold coffee. Hence statement 1 is correct The Tea Board of India is a state agency of Coffee is propagated by seed while tea can be the Government of India established to promote propagated from both seed and stem cuttings. the cultivation, processing, and domestic trade Hence statement 2 is incorrect. as well as export of tea from India. It was Educational Objective: to know about various established by the enactment of the Tea Act in commercial crops and their distribution and 1953 with its headquarters in Kolkata (formerly their contribution to Indian agri exports. Calcutta). The Tea Board India is responsible for 4. Key: D the assignment of certification numbers to Explanation: Chikmagalur one of the famous exports of certain tea merchants. This hill station towns in Karnataka is known as land certification is intended to ensure the teas’ origin, of coffee as it accounts for major production and which in turn would reduce the amount of also coffee was first introduced here in the fraudulent labelling on rare teas such as ones Bababudangiri hills. Being located along the harvested in Darjeeling. westernghats it provides ideal conditions for The Rubber Board is a statutory body coffee production. constituted by the Government of India, under Mandya known as sakkare nadu is one of the the Rubber Act 1947, for the overall towns in southern Karnataka which is famous development of the rubber industry in the for sugarcane cultivation and sugar industry country. Head Office is in Kottayam, Kerala.

www.laexias.com Page No. 149 https://elearn.laex.in Geography UPSC Previous Year Questions

To promote by such measures as it thinks fit the  The Tobbacco board is headquartered in development of the rubber industry. Guntur Andhra with aim to strive for the Without prejudice to the generality of the overall development of tobacco growers and foregoing provision the measures referred to the Indian Tobacco Industry. therein may provide for: Hence option B is right choice 1. Undertaking, assisting or encouraging Educational Objective: to know about different scientific, technological or economic organizations and their importance in promoting research. agri exports of commercial crops. 2. Training students in improved methods of 6. Key: D planting, cultivation, manuring and spraying. Explanation: Zero tillage is a condition in which 3. The supply of technical advice to rubber soil is not tilled before cultivated other crop after growers harvesting. Thus the soil is not disturbed and 4. Improving the marketing of rubber. also the residue prevents erosion and helps in 5. The collection of statistics from owners of water conservation. estates, dealers and manufacturers. Gypsum added to the soil decrease acidity of soil 6. Securing better working conditions and the and also changes the structure of soil by provision and improvement of amenities and breaking it increasing its water retention incentives to workers. capacity and inturn promoting water saving. 7. Carrying out any other duties which may be Crop residue on the soil decrease run-off and vested with the Board as per rules made also prevents evaporation and also provides under this Act. shade help in reducing the water needed. It shall also be the duty of the Board: Hence all the statements are correct 8. To advise the Central Government on all Educational Objective: to study and matters relating to the development of the understand on how to increase irrigation rubber industry, including the import and efficiency in India and save the water as in India export of rubber. Agriculture accounts for more than 80% of water 9. To advise the Central Government with consumption. regard to participation in any international 7. Key: B conference or scheme relating to rubber. Explanation: Urea is one of the most used 10. To submit to the Central Government and fertilizers in India and due to its high subsidy such other authorities as may be prescribed, and cheapness it is used more than needed half yearly reports on its activities and the altering the standard natural nutrient working of this Act, and distribution of soil making the soil unfit for 11. To prepare and furnish such other reports cultivation. relating to the rubber industry as may be To address this issue government introduced required by the Central Government from neem coated urea which reduces the dissolution time to time.

www.laexias.com Page No. 150 https://elearn.laex.in Geography UPSC Previous Year Questions

of urea in the soil and also prevents frequent Maize is also used for production of alcohol due usage of it by farmers and also prevents water it’s rich starch content and also ready and easy pollution due to excessive leaching of urea. fermentation process. Hence B is right choice Ex: Corn whiskey of USA Educational Objective: to understand various Hence all statements are correct answers steps taken by the government to promote Educational Objective: To understand the sustainable agriculture. importance of food grains in agriculture and their diversified uses. 8. Key: C Explanation: Drip irrigation reduces the 10. Key: B appearance of weeds due to its limited water Explanation: Sustainable Sugarcane availability and also localized irrigation which Initiative is a method of sugarcane production helps in growth of only the cultivated crop and which involves using less seeds, less water and limiting the water for weed plants. optimum utilization of fertilizers and land to Drip irrigation has no direct affect on salinity achieve more yields. Driven by farmers, SSI is an reduction it helps in growing the crops in saline alternate to conventional seed, water and space soil by creating localized saline free zones near intensive Sugarcane cultivation. the crop area. The major principles include the following: One of the most important benefits of drip Raising nursery using single budded chips In irrigation is to control soil erosion which is the conventional method, 2-3 budded sugarcane common phenomenon in traditional irrigation. setts are used for planting. However, in SSI, they Due to drop by drop water from drip irrigation is use single budded chips from a healthy mother reduces the leaching of top soil due to its low cane. This would give high percentage of intensity and thus reduce soil erosion. germination depending. Hence only 1 and 3 are correct statements Water management is the key issue in SSI. It Educational Objective: To study about drip emphasizes that sufficient moisture is provided irrigation it’s potential and benefits associated rather than inundating the field with water with it. because flooded condition will actually hinder the growth of the plant. SSI methods use furrow 9. Key: D / alternate furrow / drip irrigation so that only Explanation: Maize is one of the staple food required quantity of water is given. Overall, SSI grains which is mainly used as animal food but saves around 40% of water. it can be used as replacement for wheat flour to Although SSI does not put an end to use of make corn breads and various baked products chemical fertilizers; yet it discourages high which are rich source of starch. application of chemical fertilizers and use of Maize also forms important raw material for and eedicides. It promotes more use of biodiesel production due to it’s high starch organic manures, bio-fertilizers and follow content and ease of conversion to ethanol.

www.laexias.com Page No. 151 https://elearn.laex.in Geography UPSC Previous Year Questions

biocontrol measures. However, it does not ask farming due to its diversity and increased for a sudden switch to organic cultivation. productivity. SI promotes intercropping in sugarcane with Advantages of mixed farming crops like wheat, potato, cowpea, French bean, Farmers can keep their fields under continuous Chickpea, water melon, brinjal etc. In addition production. to effective utilization of land, this practice will It enhances the productivity of the farm land. reduce the weed growth up to 60% and give It increases the per capita profitability. extra income to farmers. Both farmings compliment each other. Thus only 1, 2 and 4 are correct It enhances the productivity of the farmer also. Educational Objective: to learn and Reduce dependency on external inputs and costs. understand different methods of sustainable Hence option C is most rational choice farming. Educational Objective: to study different types

11. Key: C of farming and their benefits and issues. Explanation: On the basis of cultivation crops 13. Key: D can be divide into Explanation: Rainfed crops are those crops Kharif crops: cultivated during the rainy season which depend mainly on rain water for their Ex: Rice, cotton, groundnut, jowar etc production and it provides most of the food Rabi crops: cultivated during post-monsoon consumed by the poor people. seson and harvested in early spring The main rainfed crops are the millets which are Ex: Wheat, mustard important source of various nutrients like finger Zabi crops: short summer seasons crops millet, pearl millet etc Ex: watermelon, cucumber etc In India most of the oil seeds like ground nut, Thus on 1, 2 and 3 are correct mustard and sesamum are also rainfed which Educational Objective: to have knowledge depend mainly on monsoon for their production. about the seasonality of crops in India and This is why India unable to meet its oil demands. necessary conditions for different crops. Hence all crops are rainfed crops to study about different 12. Key: C Educational Objective: crops in India and season of their cultivation Explanation: Cultivation of two or more crops and their source of irrigation. on the same field is called as mixed cropping which is done to better utilize the potential of land also as security in case of price crash of 14. Key: C any one. Explanation: Micro irrigation is defined as the While mixed farming on the other hand involves frequent application of small quantities of water rearing of plants and animals on same land so directly above and below the soil surface; usually as discrete drops, continuous drops or that the mutual benefits of both of them are tiny streams through emitters placed along a harnessed. It is one of the most familiar form of water delivery line.

www.laexias.com Page No. 152 https://elearn.laex.in Geography UPSC Previous Year Questions

Due to low intensity of water in micro irrigation table and also hydraulic capacity of water to the loss of nutrients as well as soil erosion is through the soil. Thus 3 and 4 are incorrect. prevented. Hence statement 1 is correct. Educational Objective: to study about salinity Dryland farming is one in which crops mostly and its role in soil degradation and various other depend on rainwater with some water irrigation in case of stress. But micro irrigation is not the effects. only method as other like sprinkler, drip can be 16. Key: A used. Hence statement 2 is incorrect. Explanation: Pulses are edible seeds of plants Due to slow application of water drop by drop belonging to legume family which are most near the crop itself micro irrigation reduces important source of protein to people. But due to water requirement and is seen as solution to being rainfed crop and skewed MSP policy the address decreasing water table in India. Hence production of pulses has remained more or less statement 3 is correct. constant making people depend on food grains thus ultimately leading to protein deficiency. Educational Objective: To study different Para TE 1973 (Period- TE 2008 (Period- irrigation methods and verify their benefits and met Year I II) to adopt best so that irrigation efficiency can be er Orissa India Orissa India increased. Area 6.6 125.8 6.7 122.6 mha (0.89) (0.15) (-0.34) (-0.13) Producti 5.6 129.0 7.4 196.6 15. Key: B Food on (mt) (2.46) (2.64) (0.69) (1.27) Explanation: Salinity becomes a problem when Yield 847 1024 1089 1603 enough salts accumulate in the root zone to (kg/ha) (1.57) (2.49) (1.02) (1.40) negatively affect plant growth. Excess salts in Area 4.4 (- 39.6 4.5 (- 43.3 mha 0.42) (0.54) 0.05) (0.13) the root zone hinder plant roots from Padd Producti 4.2 52.3 6.0 83.6 withdrawing water from surrounding soil. This y on (mt) (1.50) (2.83) (0.78) (1.26) lowers the amount of water available to the plant, Yield 964 1312 1335 1940 (kg/ha) (1.92) (2.29) (0.82) (1.16) regardless of the amount of water actually in the Area 1.6 22.9 1.8 (- 22.5 (- root zone. For example, when plant growth is mha (4.92) (0.16) 0.69) 0.10) compared in two identical soils with the same Puls Producti 0.8 11.6 0.8 (- 13.3 es on (mt) (6.02) (0.92) 2.14) (0.25) moisture levels, one soil receiving salty water Yield 503 504 441. (- 592 and the other receiving salt-free water, plants (kg/ha) (1.10) (0.76) 1.45) (0.36) are able to use more water from the soil Area 0.7 18.2 0.9 (- 25.3 receiving salt-free water. Although the water is mha (7.10) (1.37) 2.67) (0.04) Oilse Producti 0.5 10.9 0.6 (- 22.2 not held tighter to the soil in saline ed on (mt) (8.09) (3.05) 3.38) (1.54) environments, the presence of salt in the water Yield 683 595 669 (- 873 causes plants to exert more energy extracting (kg/ha) (0.99) (1.68) 0.72) (1.49) water from the soil. The main point is that Note: Figures in Parentheses are Annual excess salinity in soil water can decrease Compound Growth rate (%) in Period-I and plant available water and cause plant stress. Period-II Hence statement 1 is in correct and 2 is : To study the trends and correct. Educational Objective Due to formation of impermeable layer it cultivation of crops in India with time. prevents the infiltration of water reducing water

www.laexias.com Page No. 153 https://elearn.laex.in Geography UPSC Previous Year Questions

17. Key: D Educational Objective: To study the trends and Explanation: Rice is one of the most edible cultivation of crops in India with time. crops which occupies largest area under food 18. Key: C grains in India accounting for more than 140 Explanation: The percentage net sown area of metric tons. Second largest in world after China. Andhra Pradesh is 40% where as net sown area Wheat is second largest produced food grain in of West Bengal is 60%. Hence statement 1 is India accounting for more than 100mt also correct second in world after China. The major soil of the Andhra Pradesh is laterite Coarse cereals occupy third place in production soil but it is not the sole reason for low net sown due to rainfed nature and also displacement of it area. Because net sown area depends on various by rice and wheat by green revolution reducing factors like urbanization, availability of irrigation, area for coarse cereals. weather situation etc. Hence statement 2 is Pulses comes last as area under pulses correct but not reason for statement 1. remained stagnant since last decade due to Hence option B is right choice skewed MSP policy which leads to preference of Educational Objective: To understand the rice and wheat over pulses. diversity of crops cultivated and various factors Hence D is the correct choice. affecting the net sown area in different parts of the country.

19. Key: B Explanation: On the basis of cultivation crops can be divide into Kharif crops: cultivated during the rainy season Ex: Rice, cotton, groundnut, jowar, maize etc Rabi crops: cultivated during post-monsoon seson and harvested in early spring Ex: Wheat, mustard Zabi crops: short summer seasons crops Ex: watermelon, cucumber etc Thus on 1, 2 and 3 are correct Educational Objective: To have knowledge about the seasonality of crops in India and necessary conditions for different crops.

20. Key: D Explanation: The natural rubber production in India during 2019 was 7,30,000 tonnes but consumption was 12 lakh tonnes. Thus because

www.laexias.com Page No. 154 https://elearn.laex.in Geography UPSC Previous Year Questions

of it huge domestic consumption India even Explanation: is the home though produces rubber in sufficient quantity for origin of cotton plant which can be seen from and is 6th largest producer of rubber. But due to the Indus valley civilization. The Greeks called its enormous domestic demand it is net importer Indus as Sindon which means cotton. of natural rubber. India was the first country to develop hybrid Thus statement 1 is correct and 2 is incorrect. cotton way back in 1970s called H-4 variety and also it has the highest area 1/3rd under hybrid variety. Educational Objective: To study about cotton and it’s varieties

23. Key: C Explanation: Jaya and Padma are important varieties of hybrid rice crops cultivated in India which greatly increased the rice production in the country. Arjun and Sonalika are the hybrid varieties of wheat developed by ICAR to increase resistant to Educational Objective: To study about different rust bowl and also to promote early maturity so commercial crops and their contribution to that it could be harvested early. Indian exports. Hence C is correct option 21. Key: C Educational Objective: To study about different Explanation: East coast of India produces more types and hybrids of crops cultivated and their rice than west coast due to presence of various advantages over traditional variety.

deltas on the west coast like sunderbans, 24. Key: A Mahanadi and Godavari deltas which are the Explanation: More than 90% area under pulses rice bowls of the country which provide ideal is under rainfed conditions which has made condition for rice cultivation due to their wetland India net importer of pulses to meet its growing nature. demand from the raising middle class. West coast of India receives more rain than east The share of pulses under cultivation has due to influence of western ghats and also remained stagnant since last decade due to coromandel coast on east coast remains dry skewed nature of MSP increasing attraction during southwest monsoons. towards rice and wheat. Educational Objective: To understand the rice India accounts for more than 25% of rice area in cultivation trends in India and also the area of the world and is second largest producer of rice largest production. after China.Rice occupies more than 25% of 22. Key: C gross cropped area under cultivation.

www.laexias.com Page No. 155 https://elearn.laex.in Geography UPSC Previous Year Questions

Educational Objective: To understand the trends of sugar cultivation in India and various issues associated with sugar Industry in India.

26. Key: B Explanation: Maharashtra accounts for 52% of production and 50% of area under Jowar cultivation in India. Gujarat is the largest producer of groundnut in India followed by Andhrapradesh and India second largest after China.

Educational Objective: To analyse the cropping trends in Indian agriculture and various factors affecting production of different crops.

25. Key: B Explanation: Sugarcane is one of the most important commercial crops in India after cotton

providing important raw material for sugar Rajasthan has the largest area under cultural industry and providing employment for lakhs wasteland due to large area of state under desert. indirectly. Andhra Pradesh and Karnataka are largest Uttarpradesh is the largest producer of producers of maize in India but Andhra has the sugarcane in India followed by Maharasthra highest yield among all states. where as Maharashtra is largest producer of sugar.

Educational Objective: To analyse the cropping trends in Indian agriculture and various factors affecting production of different crops.

www.laexias.com Page No. 156 https://elearn.laex.in Geography UPSC Previous Year Questions

27. Key: D Educational Objective: to study the different Explanation: Successful cultivation of cotton climatic requirements for different crops and requires a long frost-free period, plenty of their distribution. sunshine, and a moderate rainfall, usually from 28. Key: A 60 to 120 cm (24 to 47 in). Soils usually need to Explanation: Organic agriculture can be be fairly heavy, although the level defined as: an integrated farming system that of nutrients does not need to be exceptional. In strives for sustainability, the enhancement of general, these conditions are met within the soil fertility and biological diversity while, with seasonally dry tropics and subtropics in the rare exceptions, prohibiting synthetic pesticides, Northern and Southern hemispheres, but a large antibiotics, synthetic fertilizers, genetically proportion of the cotton grown today is modified organisms, and growth hormones. cultivated in areas with less rainfall that obtain Shifting cultivation is the practice of clearing the water from irrigation. forests and cultivating crops until fertility is The soils most suitable for flax, besides exhausted and moving to other allowing that to the alluvial kind, are deep loamscontaining a regain fertility. It is on of the main reasons for large proportion of organic matter. Flax is often soil erosion in India. found growing just above the waterline Cultivation of high yielding varities requires in cranberry bogs. Heavy clays are unsuitable, large use of pesticides and insecticides for pest as are soils of a gravelly or dry sandy nature. control hence is not eco-friendly. Farming flax requires few fertilizers or pesticides. Growing plants in green houses also requires Within eight weeks of sowing, the plant can large investment in pesticides and insecticides reach 10–15 cm (3.9–5.9 in) in height, reaching hence also not friendly 70–80 cm (28–31 in) within 50 days. Hence organic farming is the most eco-friendly Sugar beet is a native of temperate climatic of all the four. regions with moist and mild winters and Educational Objective: To study different moderate summer temperature and thus, in agricultural practices and adopt the best one for wide contrast to sugarcane it grows in mid- sustainable development. latitude regions. It is very sensitive to annual ranges of rainfall which should necessarily be 29. Key: A around 60 to 65 cm. Explanation: Gujarat is one of the largest cotton Jute crop requires humid climate with producers due to ideal environment condition temperature fluctuating between 24 degree and also presence of black cotton soil. Celsius and 38 degree Celsius. Minimum rainfall Madhya Pradesh due it’s rainfed agriculture is required for jute cultivation is 1000 mm. New most suitable for cultivation of various pulses as grey alluvial soil of good depth receiving silt it can withstrand some water stress. from annual floods is most suitable forjutegrowth.

www.laexias.com Page No. 157 https://elearn.laex.in Geography UPSC Previous Year Questions

Kerala due to it monsoon climate and hilly Even though wheat is grown here it is mostly in terrain is suitable for production of various the western side where temperature is in range spices like pepper, turmeric, cardamom etc of 20-25 degree and rainfall is 80-100 cm. wheat Pineapple is mostly grown in warm and humid being temperate crop cannot with strand cannot climate with temperature between 15-30 degrees. high temperatures of lower gangetic plains. Hence best suitable location is north-east and Cotton is a tropical crop grown mostly grown in West Bengal. Maharashtra, Karnataka and Gujarat. It Hence the correct order is the option A requires 6 to 8 months of frost free season which Educational Objective: To study the different is not available in middle gangetic plains. climatic requirements for different crops and Hence option C is the better choice. their distribution. Educational Objective: To study about different agro-climatic zones and their major crops. 30. Key: D Explanation: Pulses are the edible seeds which 32. Key: A are rich source of protein to the human beings. Explanation: India is known for it uniquess of The various pulses are gram, tur, pegion pea etc being the only country producing all the five And also belonging to the legume family they varities of silk in world. have the ability to absorb nitrogen directly from  Eri the environment and thus help in mainting  Muga nitrogen ratio of soil and prevents excessive use  Tasar of urea. Thus these are used as green manure in  Mulberry different parts of country before sowing the  Oak tasar crops to enrich nitrogen naturally. India is the second largest producer of Hence all being leguminous crops are used both sugarcane and Brazil is the largest producer. as pulses and green manure. Educational Objective: To study about different legumes and their role in improving soil fertility.

31. Key: C Explanation: Lower gangetic region occupying the ganga Yamuna is one of the fertile regions of the country due to ready availability of water for irrigation and temperature of above 30 degree. The best suitable crops are paddy and jute which require temperature of 30-35 degree and rainfall of 100-150cm which is readily available in lower gangetic plains.

www.laexias.com Page No. 158 https://elearn.laex.in Geography UPSC Previous Year Questions

Educational Objective: To study about different 35. Key: C varieties of various crops grown in India and Explanation: The above are the climate place of India in world in their production. requirements of pepper crop. It grows well in 33. Key: B well in drained soil, It needs high rainfall of Explanation: Orchids is beautiful plant species more than 200cm and temperature range of which is divided into two types 15°C to 30°C.It grows in up to a height of 1500 1. Epiphytic Orchids—Plant growing in another meters above sea level. In June – December plant, which is found in tropical evergreen season, high quality pepper crop is cultivated in forest. rainfed areas. 2. Terestial Orchids—Plant growing directly on Educational Objective: To know about the soil, which is found in temperate and alpine climate requirements of pepper crop. vegetation 36. Key: C Arunachal Pradesh state has tropical evergreen Explanation: Integrated Watershed and semi evergreen and alpine vegetation, development Programme (IWDP) is suitable for growing orchids. So Arunachal implemented by Department of Land Resources Pradesh is called Orchid state of India of Ministry of Rural Development. The main Educational Objective: Arunachal Pradesh objective of IWMP is to restore ecological balance location and economic geography of India and by harnessing, conserving and developing plant vegetation degraded natural resources such as soil, 34. Key: D vegetative cover and water. Explanation: Dalbergia species is commonly Benefits of IWDP known Rosewood, Shisham and is a medium to large deciduous tree, native to India, with a slight crown. It is found in Himalayan foothills. It is used as firewood, timber, poles, posts, tool handles, fodder, erosion control and as a windbreak. Oil is extracted from the seed and tannin from the bark. IWDP has no link with the inter-linking of rivers. Current Status: India, with the help of Hence only 1, 3 and 4 are correct. Bangladesh and Nepal, is trying to de-list Educational Objective: To study different ‘Dalbergia sissoo’, from the list of threatened programmes by government to revitalize Indian varieties (Appendix II of CITES) in order to Agriculture and also make farming a protect the livelihood of handicraft remunerative and sustainable activity. manufacturers and farmers in the Sub-continent. Educational Objective: To know about Dalbergia species

www.laexias.com Page No. 159 https://elearn.laex.in Geography UPSC Previous Year Questions

www.laexias.com Page No. 160 https://elearn.laex.in Geography UPSC Previous Year Questions

2) Merger of IISCO with SAIL has been completed 9. Industry Which of the statements given above is/are 1. In India, the steel production industry correct requires the import of (a) 1 only (a) Saltpetre (b) 2 only (b) Rock phosphate (c) Both 1 and 2 (c) coking coal (d) Neither 1 nor 2 (d) All of the above 5. Match List I with List II and select the 2. Which one among the following industries correct answer using the code given is the maximum consumer of water in below the Lists: India? List-I (Aluminium List-II(Location) (a) Engineering Company) (b) Paper and pulp A. BALCO 1. Hirakud (c) Textiles B. HINDALCO 2. Korba (d) Thermal power C. Indian Aluminium 3. Koraput Company 3. Tamil Nadu is a leading producer of mill- D. NALCO 4. Renukoot made cotton yarn in the country. What Codes: could be the reason? (a) A-3; B-1; C-4; D-2 1) Black cotton soil is the predominant type of (b) A-2; B-4; C-1; D-3 soil in the State (c) A-; B-4; C-1; D-2 2) Rich pool of skilled labour is available (d) A-2; B-1; C-4; D-3

Which of the above is/ are the correct 6. Match List I with List II and select the reasons? correct answer using the codes given (a) 1 only below the lists: (b) 2 only List-I List-II (c) Both 1 and 2 (Industries) (Industrial Centres) (d) Neither 1 nor 2 A. Pearl fishing 1. Pune B. Automobiles 2. Tuticorin 4. With reference to the steel industry in C. Ship building 3. Pinjore India in the recent times, consider the D. Engineering goods 4. Marmagao following statements: Codes: 1) Vizag Steel Plant (RINL) has been declared (a) A-2; B-1; C-4; D-3 Mini Ratna. (b) A-2; B-1; C-3; D-4 (c) A-1; B-2; C-4; D-3

www.laexias.com Page No. 161 https://elearn.laex.in Geography UPSC Previous Year Questions

(d) A-1; B-2; C-3; D-4 10. Match List I with List II and select the correct answer using the codes given 7. Which of the following places are known below the lists: for paper manufacturing industry? 1) Yamunanagar List-I (Places) List-II (Industries) A. Jamnagar 1. Aluminium 2) B. Hospet 2. Woolen Textile 3) Punjab C. Korba 3. Fertilizers 4) Ballarpur D. Haldia 4. Cement Choose the correct answer using the codes 5. Iron and Steel given below (a) 1, 2 and 3 Codes: (a) A-4; B-3; C-1; D-2 (b) 1, 2 and 4 (b) A-2; B-5; C-1; D-3 (c) 1,3 and 4 (c) A-4; B-5; C-2; D-1 (d) 2, 3 and 4 (d) A-2; B-1; C-4; D-3 8. With reference to the usefulness of the by-products of sugar industry, which of 11. The rough outline map given shows the following statements is/are correct? centres of cement Industry labelled 1, 2, 1) Bagasse can be used as biomass fuel for 3 and 4. Match these centres with the the generation of energy. following sets of names: 2) Molasses can be used as one of the feedstocks for the production of synthetic chemical fertilizers. 3) Molasses can be used for the production of ethanol. Select the correct answer using the codes (a) Katni given below. (b) Tirunelveli (a) 1 only (c) Sikka (b) 2 & 3 only (d) Churk (c) 1 & 3 only Select the correct answer using the codes (d) 1,2 & 3 given below: 9. Rawa offshore block with great potential Codes: of oil is located on (a) A-3; B-4; C-2; D-1 (a) Krishna - Godavari basin (b) A-2; B-4; C-1; D-3 (b) Cauvery basin (c) A-1; B-2; C-4; D-3 (c) Mahanadi basin (d) A-2; B-3; C-1; D-4 (d) Palar - Pennar basin

www.laexias.com Page No. 162 https://elearn.laex.in Geography UPSC Previous Year Questions

Thermal power plants in India use water for 9. Key and Explanation cooling purposes and the disposal of fly ash, a 1. Key: C byproduct in combustion processes. This Explanation: The coal found in India is mainly excessive water use creates two interlinked of non-coking quality and hence coking coal has problems: Thermal power plants affect water to be imported. 70% of the steel produced security and are, in turn, affected by the non- today uses coal. Coking coal is a vital availability of water. ingredient in the steel making process. And according to the Central Electricity India now ranks as the second-largest producer Authority, thermal power plants are already of crude steel after China. India's crude steel facing difficulties due to non-availability of water, production in 2018 was at 109.3 million tonnes, particularly in coal-bearing states like Odisha, up by 7.7 percent from 101.5 million tonnes in Jharkhand and Chhattisgarh. 2017. This moved up to 111.2 million tonnes in Educational Objective: To know about Thermal 2019 power plants. Process of Steel Manufacturing 3. Key: B Transport of raw material to plant Explanation: Red soil is predominant in Tamil Blast Furnace: Iron ore is melted. Lime stone is Nadu. Hence statement 1 is wrong. Rich pool of fluxing material which is added. Slag is removed. skilled labour is available in Tamil Nadu and Coke is burnt to heat the ore. this labor force will help in producing mill made Pig Iron: Molten matter is poured into moulds cotton yarn in our country. Hence statement 2 is called pigs. correct. Shaping Metal: Rolling, pressing, casting and forging. Steel Making: Pig iron is further purified and oxidises the impurities. Manganese, nickel, chromium added. Educational Objective: To know about coking coal. India is known worldwide for its production of 2. Key: D textiles in general, and cotton in particular, both Explanation: As of August 30, 2019, there were being major industries for the country. Indian 269 thermal power plants in India, according to textile industry is the mother of all industries the Central Electricity Authority. Taken together, and is among the world’s top producers of yarns these plants consume 87.8% of the total and fabrics. amount of water consumed by the industrial Educational Objective: To know about textile sector, according to a study conducted by The industry. Energy and Resources Institute (TERI). 4. Key: B

www.laexias.com Page No. 163 https://elearn.laex.in Geography UPSC Previous Year Questions

Explanation: RashtriyaIspat Nigam Ltd, The Hindustan Aluminum Corporation (abbreviated as RINL), also known as Vizag Steel, Limited was established in 1958 by the Aditya is a Public steel producer based in Birla Group. In 1962 the company began Visakhapatnam, India.RashtriyaIspat Nigam production inRenukoot in Uttar Pradesh Limited (RINL) is the corporate entity of making 20 thousand metric tons per year of Visakhapatnam Steel Plant (VSP), India's first aluminium metal and 40 thousand metric tons shore-based integrated Steel Plant built with per year of alumina. In 1989 the company was state-of-the-art technology. restructured and renamed Hindalco. RINL was wholly owned by the Government of The Indian Aluminium Company Ltd. (INDAL), India. In November 2010, the company was Hirakud: It started production in 1938 as a granted the Navratna status by the Government private company and was converted into a of India. Hence statement 1 is wrong. public company in 1944. It is an integrated The Indian Iron and Steel Company (IISCO), a plant having three units at five different places 100% subsidiary of Steel Authority of India for the pro-duction of alumina and aluminium Limited (SAIL) has been amalgamated with the sheets. parent company with effect from 16 February The National Aluminium Company Ltd. 2006. The Ministry of Company Affairs (MoCA) (NALCO), Koraput: It is the largest aluminium issued the final order on 15 February 2006 plant of the country, located at Koraput. It approving the scheme of IISCO’s amalgamation obtains bauxite from the bauxite mines at with SAIL. Hence statement 2 is correct. Panchpatmali (District Koraput). It has an Miniratna, Navratna and Maharatna Status are installed capacity of 1.6 million tonnes of ingots given to PSUs depending on their success. per year. The Department of Public Enterprises under the Educational Objective: To know about Ministry of Heavy Industries & Public aluminium industry. Enterprises grants these statuses. 6. Key: A Educational Objective: To know about steel Explanation: based PSU.

5. Key: B Explanation: Bharat Aluminium Company Ltd. (BALCO) is an Indian aluminium company. BALCO was incorporated in 1965 as a Public Sector Undertaking (PSU) and it was the Public The Pearl Fishery Coast refers to a coastal area sector until 2001 and has its corporate office in of southern India, extending along the New Delhi. Its main plant and facilities are Coromandel Coast from Tuticorin to Comorin situated in Korba, Chhattisgarh. ruled by Paravars.

www.laexias.com Page No. 164 https://elearn.laex.in Geography UPSC Previous Year Questions

Mormugao is a seaport situated in the  As feed eponymous taluka of the district of South Goa,  Boiler fuel to produce steam and power Goa state, India. It has a deep natural harbour  With tar it is used as leak proof material and is Goa's chief port.  Molasses Educational Objective: To know about harbour.  In fermentation industry  7. Key: B To produced denatured power alcohol and Explanation: Ballarpur Industries Ltd was potable alcohol founded in 1945. The leading paper mill is  Industry and automobile fuel headquartered in Gurgaon, India. It is a leading  Filter cake- used as fertilizer manufacturer of writing and printing paper. It Hence all the statements are correct. has manufacturing units in different locations Educational Objective: To study about sugar including, industry and it’s situation in India and crisis 1. Ashti, Gadchiroli, Maharashtra associated with it. 2. Bhigwan, Pune District, Maharashtra 9. Key: A 3. Sewa, , Orissa Explanation: It was developed in partnership 4. Ballarpur or Ballarshah, Chandrapur with Cairn India, ONGC, Videocon and Ravva Oil District, Maharashtra Singapore Private Limited, under a 25-year 5. Yamuna Nagar, Yamuna Nagar District, production sharing contract (PSC) that expires Haryana in 2019 6. Kamalapuram, Warangal District, Telangana Educational Objective: To learn about the locations of paper manufacturing industries in India.

8. Key: D Explanation: Sugar industry is one of the important agro-based industries in India providing employment to large number of people in India. It is a seasonal industry due to seasonal availability of raw material and has to be located close to the source of raw material To have idea about due to its weight loosing nature. Educational Objective: various oil producing blocks in India and their The various by products of sugar industry and locations it’s uses  Bagasse- solid fibrous material containing 10. Key: D cellouse Explanation: Jamnagar is a city located on the  Used in paper and pulp industry western coast of India in the state

www.laexias.com Page No. 165 https://elearn.laex.in Geography UPSC Previous Year Questions

of Gujarat in Saurashtra region. It is famous for Which of the above pairs is/are correctly woolen textiles. Hence is matched with 2. matched? Option A and C are eliminated. (a) 1 and 2 only Hosapete, also known as Hospet, is a city (b) 3 only in Ballari District in central Karnataka, India. It (c) 1,2 and 3 is located on the Tungabhadra River, 12 km (d) None from Hampi. It is noted for noted for its iron ore 2. Which one of the following National mining and steel plants. Hence option b is Highways passes through Maharashtra, correct. Chattisgarh and Orissa? Korba is famous for aluminium industries and (a) NH4 Haldia for cement Industries. (b) NH5 Educational Objective: To know about the (c) NH6 major industrial cities and their prominent (d) NH7 industry types 3. Consider the following statements: 11. Key: B Among the Indian States Explanation: Katni is a town on the banks of 1) Andhra Pradesh has the longest coastline the Katni River in Madhya Pradesh, India. 2) Gujarat has the highest number of airports Hence point 2 is Katni. Which of the statements given above is/are Tirunelveli/ Tinnevelly, is a major city in the correct? Indian state of Tamil Nadu. Point 4 is (a) 1 only Tirunelveli. Sikka is a census town in (b) 2 only Jamnagar district in the Indian state of Gujarat. (c) Both 1 and 2 Point 1 is Sikka. Churk Ghurma is a town and (d) Neither 1 nor 2 a nagar panchayat in Sonbhadra district in the 4. Which among the following National state of Uttar Pradesh, India. Point 3 is Churk. Highway routes is the longest? Option B matches with the above points (a) Agra-Mumbai Educational Objective: to have knowledge (b) Chennai-Thane about various major industrial cities in India like (c) Kolkata-Hajira cement Industry, steel and iron factory etc. (d) Pune-Machilipatnam 5. What is the correct sequence of the 10. Transport following Indian states in descending 1. Consider the following pairs: order of their length of surface roads per 2 National Highway Cities connected 100 km of their area? 1) NH4 Chennai& Hyderabad 1) Haryana 2) NH6 Mumbai & Kolkata 2) Maharashtra 3) NH 15 Ahmedabad& Jodhpur 3) Punjab

www.laexias.com Page No. 166 https://elearn.laex.in Geography UPSC Previous Year Questions

4) Tamil Nadu (c) 2 and 3 Select the correct answer using the codes (d) 2 and 4 given below: 8. Which one of the following statements is Codes: not true of the Konkan Railway? (a) A-4; B-3; C-2; D-1 (a) The total length is about 760 km (b) A-4; B-3; C-1; D-2 (b) It runs through the states of Karnataka, (c) A-3; B-4; C-1; D-2 Goa, Maharashtra and Kerala (d) A-3; B-4; C-2; D-1 (c) It is the only rail route that cuts across the 6. What is the correct sequence of the Western Ghats following Indian states in descending (d) The Konkan Railway Construction Company order of their length of surface roads per which came into being, raised money 100 km of their area? through Public Issues; 1) Haryana 9. The four railway junction shown by 2) Maharashtra 1,2,3,4 on the rough outline map of 3) Punjab Gujrat are respectively 4) Tamil Nadu Select the correct answer using the codes given below: (a) 4,3,2,1 (b) 4,3,1,2 (c) 3,4,1,2 (d) 3,4,2,1 (a) Palanpur,Mahesana,Ahmedabad & 7. With reference to Indian transport Vadodara systems, consider the following (b) Mahesana, Surendranagar, Rajkot statements: &Junagarh 1) Indian railway system is the largest in the (c) Palanpur, Kanda, Bhuj and Okla world (d) Ahmedabad, Vadodara, Bhavnagar& Broach

2) National Highways cater to 45 percent of 10. Which one of the following sets of states the total road transport demand stands to benefit the most from the 3) Among the states, Kerala has the highest Konkan Railway? density of surface road (a) Goa, Karnataka, Maharashtra, Kerala 4) National Highway No.7 is the longest in the (b) Madhya Pradesh, Maharashtra, Tamil Nadu, country Kerala Which of these statements are correct? (c) Tamil Nadu, Kerala, Goa, Maharashtra (a) 1 and 2 (d) Gujarat, Maharashtra, Goa, Tamil Nadu (b) 1 and 3

www.laexias.com Page No. 167 https://elearn.laex.in Geography UPSC Previous Year Questions

11. The canal-carrying capacity of Farakka is: 14. Which one among the following has the (a) 75,000 Cusecs largest shipyard in India? (b) 70,000 Cusecs (a) Kolkata (c) 40,000 Cusecs (b) Kochi (d) 35,000 Cusecs (c) Mumbai

12. Match the different ports of Union (d) Visakhapatnam Territory of Pondicherry labelled as A, B, 15. Which one of the following ports of India C and D in the given map with the handles the highest tonnage of import respective names and select the correct cargo? answer using the codes given below the (a) Calcutta list of ports. (b) Kandla (c) Mumbai (d) Visakhapatnam

16. Match the locations of ports labelled as A, B, C and D in the given map with the names of those ports and select the correct answer using the codes given List (Ports of Pondicherry) below the names of the ports: 1) 2) Mahe 3) Pondicherry 4) Yanam Codes: (a) A-2; B-1; C-3; D-4 (b) A-1; B-2; C-3; D-4 Names of Ports: (c) A-2; B-1; C-4; D-3 1) Kakinada (d) A-1; B-2; C-4; D-3 2) Karwar 13. In India, the ports are categorized as 3) Managalore major and non major ports. Which one of 4) Tuticorin the following is a non major port? 5) Veraval (a) Kochi(Cochin) Codes: (b) Dahej (a) A-4; B-2; C-3; D-5 (c) Paradip (b) A-5; B-2; C-4; D-1 (d) New Mangalore (c) A-1; B-3; C-4; D-2 (d) A-5; B-3; C-2; D-1

www.laexias.com Page No. 168 https://elearn.laex.in Geography UPSC Previous Year Questions

17. Consider the following statements: Odisha, Jharkhand and West Bengal states in 1) Tides are of great help in navigation and India. It connects Surat with Kolkata. fishing Statement 3 is also wrongly matched as NH 15 2) High tide enables big ships to enter or leave is a major National Highway in the Western and the harbour safely North-Western India. NH 15 connects 3) Tide prevents siltation in the harbours Samakhiali in Gujarat with Pathankot in Punjab. 4) Kandla and Diamond Harbour are tidal Education Objective: Understanding the ports spread of Major National Highways in India. Which of these statements are correct? (a) 1 and 4 (b) 2, 3 and 4 (c) 1,2 and 3 2. Key: C (d) 1, 2, 3 and 4 Explanation: NH 6 passes through Gujarat, 18. Which one of the following posts shown Maharashtra, Chhattisgarh, Odisha, Jharkhand on the rough outline map of India is a and West Bengal. This road is 1,949 km long. riverine port? Education Objective: Understanding the spread of Major National Highways in India.

3. Key: B Explanation: Gujarat has the largest coastline in India (1915km), followed by Andhra Pradesh (1037km). Maharashtra has the highest number of airports in India. Maharashtra has the highest number of ports followed by Gujarat. (a) 1 Educational objective: Airports location and (b) 2 economic geography of India (c) 3 4. Key: C (d) 4 Explanation: Length of Agra-Mumbai NH – 1161 km 10. Key and Explanation Length of Chennai-Thane NH – 1235 km 1. Key: D Length of Kolkata-Hajira NH – 1946 km Explanation: Statement 1 is wrongly matched Length of Pune-Machilipatnam NH – 841 km as NH 4 is a major National Highway in the So, among the given options, the length of Western and Southern India. It connects Kolkata-Hajira NH, also known as NH 6 earlier, Mumbai with Chennai. is the longest route. Statement 2 is also wrong as NH 6 runs Education Objective: Understanding the through Gujarat, Maharashtra, Chhattisgarh, spread of Major National Highways in India.

www.laexias.com Page No. 169 https://elearn.laex.in Geography UPSC Previous Year Questions

Explanation: Konkan Railway was the missing 5. Key: D link between India’s commercial capital Mumbai Explanation: Arrangement of states in and Mangalore. This railway line connects descending order of the surface roads per 100 sq. Maharashtra, Goa and Karnataka State. It’s km of their area is as follows: length is about 760 km. 49% equity is from four Tamil Nadu has 158.78 km, Punjab has 127.78 states governments of Goa, Maharashtra, Kerala km, Maharashtra has 117.61 km and Haryana and Karnataka and 51% of equity of Konkan has 63.70 km. Railway Corporation. So, option D which says Educational Objective: Understanding the that the money was raised through Public issues network of roads in India is wrong. Educational Objective: Understanding about 6. Key: D Konkan Railway and its history. Explanation: Arrangement of states in 9. Key: B descending order of the surface roads per 100 sq. Explanation: The four railway junctions shown km of their area is as follows: by 1, 2, 3 and 4 in the map of Gujarat are Tamil Nadu has 158.78 km, Punjab has 127.78 Mahesana, Surendranagar, Rajkot and km, Maharashtra has 117.61 km and Haryana Junagarh respectively. has 63.70 km. Educational Objective: Learning about Gujarat Educational Objective: Understanding the Railway Map network of roads in India 10. Key: A 7. Key: D Explanation: Konkan Railway was the missing Explanation: Statement 1 is incorrect as link between India’s commercial capital Mumbai Indian Railway System is the 4th largest railway and Mangalore. This railway line connects in the world after USA, Russia and China. Maharashtra, Goa and Karnataka State. Its Statement 2 is correct as National Highway is length is about 760 km. 49% equity is from four nearly 2% (1.96%) of total roads, but it carries states governments of Goa, Maharashtra, Kerala nearly 40 – 45% of road traffic of India. and Karnataka and 51% of equity of Konkan Statement 3 is incorrect Maharashtra has the Railway Corporation. So, option A is correct. highest density of the surface roads, not Kerala. Educational Objective: Understanding about Statement 4 is correct as National Highway No. Konkan Railway and its history. 7 is the longest in the country which joins 11. Key: A Varanasi to Kanyakumari. Explanation: The canal carrying capacity of Educational Objective: Understanding Indian Farakka barrage is 75,000 cusecs. Farakka Railways Network barrage is a 2245 metre long barrage across the 8. Key: D river Ganga.

www.laexias.com Page No. 170 https://elearn.laex.in Geography UPSC Previous Year Questions

Educational Objective: Understanding the Tamil Nadu and ‘D’ marked port is Kakinada of Farakka Barrage. Andhra Pradesh. Educational Objective: Learning about major and minor 12. Key: A ports of India. Explanation: The ports of Union Territory of Pondicherry as given in the figure are as follows: 17. Key: D A – Mahe Explanation: Tides and Fishing B – Karaikal Tides move water, bringing water in and taking C – Podicherry water out. Understanding the cycles and effects D – Yanam of tides on fish helps in better fishing. The full Educational Objective: Understanding the map and new moons normally create better fishing and geography of Pondicherry. conditions because of the spring tides. The reason behind this is that fish are easier to 13. Key: B catch when they are feeding and it’s the tide and Explanation: Dahej is a not a major port of India. Kochi currents that dictate this. When the water (Cochin), Paradip and New Mangalore are considered as begins to move, smaller fishes are at the mercy major ports. of the current and get confused in the turbulent Educational Objective: Learning about major and minor water. Larger fishes have an advantage because ports of India. they are equipped to feed in this turbulent water. 14. Key: B These larger fishes get more easily trapped when Explanation: Kochi shipyard is the largest shipyard in there are tides. India. Tidal Ports: During high tides, water rushes Educational Objective: Learning about the various into harbours (Tidal Bore). This helps ships shipyards of India. enter and exit harbours safely. High tides make 15. Key: B ocean/sea water rush into the mouths of rivers. Explanation: As per the figures, Kandla port This helps ships to enter port towns like New handled the highest traffic volume at 115.40 MT York, London, Rotterdam, and Hamburg. This is during 2018-19, followed by Paradip (109.27 perfectly advantageous for some ports in India MT), JNPT (70.70 MT), Visakhapatnam (65.30 such as Kandla, Mangalore and Kolkata’s MT), Kolkata with Haldia (63.71 MT) and diamond harbour. Mumbai (60.58 MT). Tides and Siltation: Tides take away the Educational Objective: Understanding about terrigenous material brought by the rivers at the the traffic volume of various ports of India. river harbours and thus help in the prevention of siltation. 16. Key: B Tides and Prevention of Rivers from Freezing: Explanation: In the given map ‘A’ marked ports The temperature at which sea water freezes is is Veraval of Gujarat, ‘B’ marked port is Karwar much lower than that of river water. In cities like of Karnataka, ‘C’ marked port is Tuticorin of

www.laexias.com Page No. 171 https://elearn.laex.in Geography UPSC Previous Year Questions

London, due to the high tide, the sea water enters the river and prevents it from freezing Educational Objective: Understanding the various roles of Tides and Tidal Ports of India.

18. Key: D Explanation: Hoogly in West Bengal is a major riverine port. Educational Objective: Knowledge about various ports of India and riverine ports as well.

www.laexias.com Page No. 172 https://elearn.laex.in Geography UPSC Previous Year Questions

III. Demography (a) 1 only (b) 2 only 1. World Demography (c) Both 1 and 2

1. As per the UN-Habitat's Global Report on (d) Neither 1 nor 2 Human Settlements 2009, which one 5. For India, China, the UK and the USA, among the following regions has shown which one of the following is the correct the fastest growth rate of urbanization in sequence of the median age of their the last three decades? (a) Asia populations? (b) Europe (a) China

(d) Its high total population 7. Sustainable development is described as 3. Which one among the following South the development that meets the needs of Asian countries has the highest the present without 1SE compromising population density? the ability of future generations to meet (a) India their own needs. In this perspective, (b) Nepal inherently the concept of sustainable (c) Pakistan development is intertwined with which of (d) Sri Lanka the following concepts? (a) Social justice and empowerment 4. Consider the following statements: (b) Inclusive Growth 1) Infant mortality rate takes into account the (c) Globalization death of infants within a month after birth. (d) Carrying capacity 2) Infant mortality rate is the number of infant deaths in a particular year per 100 8. Consider the following statements: live births during that year. 1) The first telegraph line in India was laid Which of the above statements is/are correct? between Kolkata (formerly Calcutta) and Diamond Harbour.

www.laexias.com Page No. 173 https://elearn.laex.in Geography UPSC Previous Year Questions

2) The first Export Processing Zone in India today 55% of the world’s population lives in was set up in Kandla. urban areas, a proportion that is expected to Which of the statements given above is/are increase to 68% by 2050. correct? (a) 1 only Projections show that urbanization, the gradual (b) 2 only shift in residence of the human population from (c) Both 1 and 2 rural to urban areas, combined with the overall (d) Neither 1 nor 2 growth of the world’s population could add another 2.5 billion people to urban areas by 9. Consider the following pairs: 2050, with close to 90% of this increase taking Programme/Project Ministry place in Asia and Africa. 1) Drought-Prone Ministry of Educational Objective: To learn about the UN- Agriculture Habitat's Global Report on Human Settlements, Area Programme 2009. 2) Desert Development Ministry of Environment and 2. Key: B Forests Programme Explanation: The demographic dividend is a rise 3) National Watershed Ministry of Rural in the rateof economic growth due to a rising Developmentfor share ofworking age people in a population. This RainfedAreas usuallyoccurs late in the demographic transition Development project whenthe fertility rate falls and the youth Which of the above pairs is/are correctly dependencyrate declines. matched? To reap the benefits of demographic dividend, (a) 1 and 2 only skills have to be developed because a relatively (b) 3 only largerportion of population fall under the (c) 1, 2 and 3 category of (d) None Productive labour force when there is a demographicdividend. 1. Key and Explanation Educational Objective: A decline in fertility and mortality rates boosts working population 1. Key: A productivity, which leads to a demographic Explanation: According to UN-Habitat's Global dividend. Report on Human Settlements, 2009 Asiaian 3. Key: A continent is registering fastest growth rate of Explanation: Overall population density of India urbanization in the last three decades is 324. Nepalis 102, Pakistan is 146, Sri Lanka According to a new -Department is 280 person persquare kilometres. of Economic and Social Affairs (2018 report), Many of the world’s small island or isolated states have large populations for their size.

www.laexias.com Page No. 174 https://elearn.laex.in Geography UPSC Previous Year Questions

Macao, Monaco, Singapore, and Gibraltar are the five most densely populated. Of the larger countries, Bangladesh is the most densely-populated with 1,252 people per square kilometre; this is almost three times as dense as its neighbour, India. Educational Objective: To know about population density. 4. Key: D Explanation: Infant mortality rate indicates the number of infant death under one years of age per 1000 live birthunderone year of age. Neo-Natal Mortality Rate (<29 days) and Post Neo-Natal Mortality Rate (1 month to 11 months). Every 6th death in the country pertains to an infant. Educational Objective: Worldwide Median age IMR in India has registered a 3 points decline to population is Japan > Italy > Germany > France 50 from 53 in 2008. Maximum IMR in Madhya Pradesh (67) and 6. Key: D minimum IMR in Kerala Explanation: Population ages 65 and above (% (12) of total) in India was reported at 6.18 % in 2018, according to the World Bank collection of development indicators, compiled from officially recognized sources. India - Population ages 65 and above (% of total) - actual values, historical data, forecasts and projections were sourced from the World Bank on March of 2020.

Educational Objective: To know about mortality rate.

5. Key: B Educational Objective: To know about Explanation: For India, China, UK and USA, the correct sequence ofthe Median age of their population ages. population isIndia < China < USA < UK 7. Key: D Explanation: In 1987, the Brundtland Commission published its report, Our Common

www.laexias.com Page No. 175 https://elearn.laex.in Geography UPSC Previous Year Questions

Future, in an effort to link the issues of water and human resources ultimately leading economic development and environmental to drought proofing of the affected areas. stability. In doing so, this report provided the The programme also aims to promote overall oft-cited definition of sustainable development economic development and improving the socio- as “development that meets the needs of the economic conditions of the resource poor and present without compromising the ability of disadvantaged sections inhabiting the future generations to meet their own needs”. programme areas. This concept of sustainable development aims to Desert Development Programme (DDP): The maintain economic advancement and progress main aim of the programme is to minimise the while protecting the long-term value of the adverse effect of drought and control environment; it “provides a framework for the through rejuvenation of the integration of environment policies and natural resource base of the identified desert development strategies” areas. The programme strives to achieve Educational Objective: To learn about the ecological balance in the long run. sustainable development and its main objectives. Ministry of Agriculture: National Watershed

8. Key: C Development Project for Rainfed Areas: Rainfed areas constitute about 57% of the total Explanation: Telegraph services in India date 140.30 million hectares cultivated in the country. back to 1850, when the first experimental Rainfed agriculture is characterised by low levels telegraph line was established between Calcutta of productivity and low input usage. and Diamond Harbour. The British East India Company started using The main objective of the scheme are: the telegraph a year later, and by 1854. when 1. Conservation, development and sustainable the system opened to the public, telegraph lines management of natural resources. had been laid across the country. 2. Enhancement of agricultural production The Kandla Free Trade Zone - India's first Export and productivity in a sustainable manner. Processing Zone - was set up in 1965. Educational Objective: To learn about the Subsequently, six more EPZs were set up. projects and its respective ministries. Thus both the statements are correct. 2. Language Educational Objective: To learn about the telegraph services and export processing zones. 1. The language spoken by the largest number of people in the world is 9. Key: D (a) Explanation:Ministry of Rural Development- (b) English Drought Prone Areas Programme (DPAP): The (c) Mandarin basic objective of the programme is to minimise (d) Spanish the adverse effects of drought on production of crops and livestock and productivity of land,

www.laexias.com Page No. 176 https://elearn.laex.in Geography UPSC Previous Year Questions

2. Which one of the following languages 2. Key and Explanation belongs to the Austric group? (a) Marathi 1. Key: C (b) Ladakhi Explanation: Mandarin is spoken in people’s (c) Khasi Republic of China, Taiwan, Singapore, Malayasia, (d) Tamil some part of UnitedStates. This language covers 885 million people.English is spoken by 500 3. Consider the following international million people. languages: 1) Arabic Educational Objective: Five most languages 2) French spoken in the world 3) Spanish Mandarin > Hindi > Spanish > English > Arabic The correct sequence of the languages given 2. Key: C above in the decreasing order of the number Explanation: Austric group (Nishada) spoken by of their speakers is. (a) 3-1-2 1.38% of totalIndian population, this group (b) 1-3-2 includes mainly triballanguages and of (c) 3-2-1 the central tribal belt, Khasiand Jaintia hills of (d) 1-2-3 Meghalaya and Nicobar island. 4. Which one among the following languages has largest number of speakers in the world? (a) Bengali (b) French (c) Japanese (d) Portuguese 5. Berber speaking community, frequently in the news, lives in (a) Afghanistan (b) Algeria (c) Argentina (d) Australia Indian languages have evolved from different 6. In which one of the following countries, stocks and are closely associated with the is Tamil a major language? different ethnic groups of India. Broadly the (a) Myanmar Indian languages can be put into six groups: (b) Indonesia 1) Indo-Aryan, 2) Dravidian, 3) Sino-Tibetan, (c) Mauritius 4) Negroid, 5) Austric and 6) Others. (d) Singapore

www.laexias.com Page No. 177 https://elearn.laex.in Geography UPSC Previous Year Questions

Educational Objective: To know about tribal of latitude north of the equator, and is situated languages. off the southern tip of the Malay Peninsula. As a result of historical migration and settlement 3 . Key: A patterns, Indian came to the Explanation: Total speaker of Spanish in world island from various parts of South Asia speaking is 329 million, Arabic speakers are 220 million a variety of South Asian languages, mostly Tamil. and French speakers are 200 million. So, the According to the population census in 2010, correct sequences in decreasing order of the about 9.2 percent of the Singaporean population number of speakers are Spanish, Arabic and were Indians, It was also found out that French. approximately 76.7 percent of these Indians Educational Objective: To know about spoke Tamil as their native language. Language and population distribution. Educational Objective: To learn about the 4. Key : A usage of Indian language in other countries. Explanation: According to 2005 edition of the National encyclopaedia, the largest numbers of 3. Population speakers in the world are, Portuguese (213 1. With reference to the role of UN-Habitat millikon speakers); Bengali (215 million in the United Nations programme working speakers); Japanese; (127 million speakers); towards a better urban future, which of French (130 million speakers). the statements is/are correct? Educational Objective: To know about 1) UN-Habitat has been mandated by the language distribution across world. United Nations General Assembly to 5. Key : B promote socially and environmentally Explanation: Berber speaking community lives sustainable towns and cities to provide in Algeria, Morocco, Libiya, Tunisia, Mali and adequate shelter for all. Egypt. 2) Its partners are either governments or local The Berber languages, also known as Berber or urban authorities only. the Amazigh languages, are a branch of 3) UN-Habitat contributes to the overall the Afroasiatic language family. They comprise objective of the United Nations system to a group of closely related languages spoken by reduce poverty and to promote access to the Berbers, who are indigenous to . safe drinking water and basic sanitation. Educational Objective: To know about berber Select the correct answer using the code community. given below:

6. Key: D (a) 1,2 and 3 Explanation: Singapore is a sovereign city-state (b) 1 and 3 only and island country located in maritime (c) 2 and 3 only Southeast Asia. Singapore lies about one degree (d) 1 only

www.laexias.com Page No. 178 https://elearn.laex.in Geography UPSC Previous Year Questions

2. Amongst the following States, which one rates among the 28 states of India (Delhi & has the highest percentage of rural Pondicherry are not included) population to its total population (on the 2) According to the Census 2001, Rajasthan basis of the Census, 2001)? has literacy rate above the national average (a) Himachal Pradesh literacy rate. (b) Bihar Which of the statements given above is/are (c) Orissa correct? (d) Uttar Pradesh (a) 1 only 3. Which of the following are among the (b) 2 only million- plus cities in India on the basis (c) Both 1 and 2 of data of the Census, 2001? (d) Neither 1 nor 2 1) Ludhiana 2) Kochi 7. According to the Census-2001, which one 3) Surat of the following Indian States has the 4) Nagpur maximum population in India after Uttar Codes: Pradesh? (a) 1, 2 and 3 only (a) West Bengal (b) 2, 3 and 4 only (b) Maharashtra (c) 1 and 4 only (c) Bihar (d) 1, 2, 3 and 4 (d) Tamil Nadu

4. Among the following, which one has the 8. Which one of the following is the correct minimum population on the basis of data statement on the basis of Census-2001? of Census of India, 2001? (a) Bihar has the highest percentage of the (a) Chandigarh Scheduled of its population (b) Mizoram (b) The Decadal growth of population of India (c) Puducherry (1991-2001) has been below 20% (d) Sikkim (c) Mizoram is the Indian state with the least 5. Which one among the following States of population India has the lowest density of population? (d) Pondicherry has the highest sex ratio among (a) Himachal Pradesh the Union Territories (b) Meghalaya 9. Consider the following statements: (c) Arunachal Pradesh 1) Area wise, Chhattisgarh is the larger than (d) Sikkim West Bengal 6. Consider the following statements: 2) According to the population 2001 Census, 1) According to the Census 2001, Kerala has population of West Bengal is larger than that the smallest gap in male & female literacy of Chhattisgarh

www.laexias.com Page No. 179 https://elearn.laex.in Geography UPSC Previous Year Questions

Which of the statements given above is/are D. 1981-2001 4. High growth with correct? definite signs of (a) 1 only slowdown (b) 2 only Codes: (c) Both 1 and 2 (a) A-3; B-; C-4; D-2 (d) Neither 1 nor 2 (b) A-1; B-3; C-2; D-4 (c) A-3; B-1; C-2; D-4 10. Consider the following statements: As (d) A-1;B-3;l C-4; D-2 per 2001 Census 1) The two states with the lowest sex ratio are 13. Which one among the following States Haryana & Punjab has the highest female literacy rate as 2) The two states with the lowest population per the Census 2001: per sq. km of area are Meghalaya & Mizoram (a) Chhattisgarh 3) Kerala has both the highest literacy rate and (b) Madhya Pradesh sex ratio. (c) Orissa Which of the statements given is/are correct? (d) Rajasthan (a) 3 only 14. Which amongst the following States has (b) 2 and 3 the highest population density as per (c) 1 and 2 Census-2001: (d) 1 and 3 (a) Kerala 11. Which among the following countries (b) Madhya Pradesh has the largest population? (c) Uttar Pradesh (a) Indonesia (d) West Bengal (b) Japan 15. Consider the following countries (c) Pakistan 1) Brazil (d) Sudan 2) Indonesia 12. India’s population growth during the 3) Japan 20th century can be classified into four distinct phases. 4) Russia Match List-I(Period) with List-II(Phase) and select What is the descending order of the size of the correct answer using the codes given below the following countries population wise? the list: (a) 1, 2 ,4 , 3 List-I (Period) List-II(Phase) (b) 2, 3, 1 , 4 A. 1901-1921 1. Steady growth (c) 2, 1, 4 ,3 B. 1921-1951 2. Rapid high growth (d) 1, 2, 3, 4 C. 1951-1981 3. Stagnant growth 16. Consider the following countries of South Asia.

www.laexias.com Page No. 180 https://elearn.laex.in Geography UPSC Previous Year Questions

1) Bangladesh (d) Uttar Pradesh 2) India 20. Consider the following statements about 3) Pakistan the megacities of India. 4) Srilanka 1) Population of each mega city is more than The descending order of literacy status of 5 million these countries. 2) All the megacities are important sea ports. (a) 4, 2, 1, 3 3) Mega cities are either National / State (b) 2, 4, 3 ,1 Capitals. (c) 4, 2, 3, 1 Which of these statements are correct? (d) 2, 4, 1, 3 (a) 1, 2 and 3 17. The high density of population in Nile (b) 1 and 2 Valley and Island of Java is primarily due (c) 2 and 3 to (d) 1 and 3 (a) Intensive agriculture 21. Which one of the following statements is (b) Industrialisation true according to 1991 Census data? (c) Urbanisation (a) UP has the highest density of population in (d) Topographic constraints India 18. Consider the Decadal Census data given (b) Himachal Pradesh has the highest female to below: male sex ratio in India Decadal population (in millions) (c) West Bengal has the highest growth rate of Year Population population in India 1961 10.7 (d) Bihar has the lowest literacy rate in India 1971 14.3 1981 16.2 The next two questions are based on the 1991 18.9 following table. Study the same and attempt The above data refer to which one of the the two items population by religion groups? Table (a) Sikhs Indicators of Development for some (b) Jains Asian Countries (c) Christians Infant Adult (d) Buddhist Life Mortality Literac Expectancy 19. The largest number of Buddhists is found in Rate(Per y Countries at (a) Bihar 1000 live Rate(Pe birth(years births) rcent (b) Karnataka 1995) (c) Maharashtra 1996 1995)

www.laexias.com Page No. 181 https://elearn.laex.in Geography UPSC Previous Year Questions

Minorities. India 62.4 72 52 1) Buddhists China 69.2 38 82 2) Christians Indonesia 64.0 47 84 3) Jains Malaysia 71.4 11 84 4) Muslims Thailand 69.5 31 94 5) Sikhs 71.7 6 98 Codes: Philippine 67.4 32 95 (a) A-5; B-1; C-3; D-2 s (b) A-4; B-3; C-2; D-1 (c) A-5; B-3; C-1; D-2 22. Which of the following statement is false? (d) A-4; B-2; C-1; D-3 (a) All countries other than India have over 80% literacy rate 25. The population growth rate in Kerala is (b) Malaysia & Korea have life expectancy the lowest among major Indian states. higher than all other countries Which one of the following is the most (c) Higher the adult literacy, lower is the infant widely accepted reason for this? mortality (a) Kerala had made the highest investment in (d) The life expectancy at birth in India is family planning almost the same at that of Indonesia (b) Kerala has the highest literacy rate in India (c) Kerala has invested heavily in promoting 23. The best performance with regard to literacy & public health & placed high human development among the countries priority on social policies is (d) The population pyramid in Kerala has (a) China relatively fewer women in the reproductive (b) Malaysia age group (c) Korea (d) Philippines 26. Among which of the following sets of social/religious groups is the extent of 24. Match the areas shown as A, B, C and D poverty highest as per Government on the given map showing with the statistics for the 90s. largest religious minorities. Select the (a) Muslims in Kerala, Gujarat & Andhra correct answer using the codes given Pradesh below the list: (b) Tribals in Bihar, MP & Maharashtra (c) SCs in Punjab, Western UP, North Rajasthan & Tamil Nadu (d) Christians in Gujarat, Maharashtra & Assam

www.laexias.com Page No. 182 https://elearn.laex.in Geography UPSC Previous Year Questions

27. As per 1991 Census, which one of the (d) 2055 following groups of Union Territories had 31. Consider the following statements. the highest literacy rate? 1) India is the second country in the world to (a) Chandigarh & Dadra Nagar Haveli adopt a National Family Planning Program. (b) Delhi & Andaman Nicobar Islands 2) The National Population Policy of India 2000 (c) Andaman Nicobar Islands & Pondicherry seeks to achieve replacement level of fertility (d) Pondicherry & Delhi by 2010 with a population of 111 crores. 28. Southeast Asia has captivated the 3) Kerala is the first state in India to achieve attention of the global community over replacement level of fertility. space and time as a geostrategically Which of the statements given above is/are significant region. Which among the correct? following is most convincing explanation (a) 1 only for this global perspective (b) 1 and 2 (a) It was the hot theatre during the Second (c) 2 and 3 World War (d) 1, 2 and 3 (b) Its location between the Asian powers of China and India 3. Key and Explanation (c) It was the arena of superpower a. 1. Key: B confrontation during the Cold War period Explanation: Statement 1 is correct. UN-Habitat (d) Its location between the Pacific and Indian is a Unites ’s programme working towards oceans and its preeminent maritime a better urban future. Its mission is to promote character socially and environmentally sustainable human settlements development and the achievement of 29. Which one amongst the following has adequate shelter for all. It is the first statement the largest livestock population in the on their about us page. world? Statement 2 is wrong, as UN-Habitat also (a) Brazil partners with private organisations and civil (b) China society. Also, the statement contains ‘extreme’ word ONLY. (c) India Statement 3 is correct.Cities are facing (d) USA unprecedented demographic, environmental, 30. As per India’s National Population Policy, economic, social and spatial challenges. There 2000, by which one of the following years has been a phenomenal shift towards urbanization. In many places around the world, is it our long-term objective to achieve the effects can already be felt: lack of proper population stabilization? housing and growth of slums, inadequate and (a) 2025 out-dated infrastructure – be it roads, public (b) 2035 transport, water, sanitation, or electricity – (c) 2045 escalating poverty and unemployment, safety

www.laexias.com Page No. 183 https://elearn.laex.in Geography UPSC Previous Year Questions

and crime problems, pollution and health issues, as well as poorly managed natural or man-made disasters and other catastrophes due to the effects of climate change. Mindsets, policies, and approaches towards urbanization need to change in order for the growth of cities and urban areas to be turned into opportunities that will leave nobody behind. UN-Habitat, the United Nations programme for human settlements, is at the helm of that change, assuming a natural leadership and catalytic role in urban matters. Educational Objective: Understanding about UN-Habitat and its functions. According to the 2011 census, there were 46 2. Key: A million-plus cities in India, with Mumbai, Delhi Explanation: On the basis of 2001 census, 9.8% and Kolkata having populations over 10 million. of population Himachal Pradesh, 10.5% of Educational Objective: To know about 2011 population of Bihar, 15% of population of Orissa facts.

& 20.8% of population of Uttar Pradesh live in 4. Key: D rural areas. Explanation: Population of Sikkim is 540,851. According to 2001 Census Out of 1027 million Population of Chandigarh is 901,000, Population (or 102.7 crore) population of India, as per of Pondicherry is 974000 and population of Census 2001, 742 million live in rural areas and Mizoram is 889,000. 285 million in urban areas comprising of 72.2% Educational Objective: To know about and 27.8% of the population respectively. The Population statistics. highest percentage of urban population is in 5. Key: C Delhi (93.0%) and the lowest in Himachal Explanation: As per Census, 2001Population Pradesh (9.8%). density of Arunachal Pradesh is 13, Himachal Educational Objective: To know about census Pradesh is 109, Meghalaya is 103, and Sikkim is related facts. 76. 3. Key: D Educational Objective: To know about Explanation: According to the Census of 2001 Population statistics. of India, the population of Ludhiana is 6. Key: D 1,398,467, Surat is 2,433,835 and Nagpur is Explanation: Mizoram is the state where male 2,052,066. literacy is 90.7% and female literacy is 86.7%. There were total of 35 million- plus cities in So, Mizoram is the state of smallest gap in male India on the basis of data of the Census, 2001 and female literacy, where Kerala has 94.2% in male and 87.7% in female.

www.laexias.com Page No. 184 https://elearn.laex.in Geography UPSC Previous Year Questions

In statement-2 literacy of Rajasthan is 60.4%, State with where national literacy is 65.38%. Highest West Bengal 903 Population Educational Objective: To know about Literacy Density rate. State with Arunachal Lowest 13 7. Key: B Population Pradesh Density Explanation: According to 2001 census, UT with Highest Maharashtra had the maximum population in Population Delhi 9.340 India after Uttar Pradesh. The total population Density in Maharashtra is 193,977,000. UT with Lowest Andaman & Population 43 Nicobar Islands The combined population of UP and Density District with Maharashtra (the second most populous state) North East Highest 29,468 at 312 million , is substantially greater than that Population (Delhi) of USA, the third most populous country of the Density world District with Lahul & Spiti Lowest (Himachal 2 Educational Objective: To know about Population Population statistics. Density Pradesh)

8. Key: D 10. Key: D Explanation: Pondicherry, a Union Territory, Explanation: As per census 2001, child sex has the sex ratio of 1001. Chandigarh has 777, ratio in Haryana was 861, in Punjab 876. Hence, Delhi has 861, Daman and Diu has 710, Dadra and Nagar Haveli has 812. Haryana and Punjab were the two states with Improvement in the sex ratio during 1991 to lowest sex ratios. 2001, which has marginally improved to 933 females per thousand males from 927 per The population per sq. km. of area in Arunachal thousand in 1991, an increase of six points. Pradesh was 13, in Meghalaya was 103 and in Educational Objective: To know about sex Mizoram was 42. Hence, the two states with the ratio. lowest population per sq. km of area are 9. Key: C Arunachal Pradesh and Mizoram. Explanation: Area of Chhattisgarh is Kerala has both the highest literacy rate and sex 1,36,03489 km and Area of West Bengal is ratio. Kerala has 94.2% in male literacy rate and 88,572 sq km but the population density in Chhattigarh is 154, whereas the population 87.7% in female literacy rate. density in West Bengal is 904. The sex ratio of Kerala is-1058 females per 1000 To know about Educational Objective: males according to 2001 census Population density. Educational Objective: To know about Population Density development aspects. Persons/ 11. Key: A Sq.Km Explanation: Indonesia has the largest India 325 population in the given countries.

www.laexias.com Page No. 185 https://elearn.laex.in Geography UPSC Previous Year Questions

Educational Objective: To know about 15. Key: C countries with highest population. Explanation: The descending order of the size of the following countries population wiseare 12. Key: C Indonesia(21,38,29,469) > Brazil(17,63,19,621) Explanation: > Russia(14,67,09,971) > Japan(12,67,29,223) Educational Objective: The top ten countries with the highest population

16. Key: C Explanation: According to UNESCO; 2015, the Educational Objective: To know about stages of literacy rate of given countries are – demographic transition. Paskistan - 56.4% 13. Key: A India - 72.1% Explanation: As per the census of 2001, Bangladesh - 72.8% Chhattisgarh has the highest female literacy Srilanka - 92.6% of 51.85, Orissa has 50.51%, Madhya The figures represented are almost entirely Pradesh - 50.29 and Rajasthan has 43.85%. collected by the UNESCO Institute for Educational Objective: To know about Literacy Statistics (UIS) on behalf of UNESCO with 2015 estimates based on people aged 15 or over who 14. Key: D can read and write. Where data is taken from a Explanation: Population density in West Bengal different source, notes are provided is 904. The global literacy rate for all people aged 15 In 2001, density of Uttar Pradesh was 690 and above is 86.3%. The global literacy rate for persons per sq km, while nation average all males is 90.0% and the rate for all females is in 2001 was 324 per sq km. 82.7%. The rate varies throughout the world That of Madhya Pradesh was 196 persons per sq with developed nations having a rate of 99.2% km. (2013);Oceania having 71.3%; South and West Kerala was 859 person per sq km its land is Asia having 70.2% (2015) and sub-Saharan three times as densely settled as the rest of India. Africa at 64.0% (2015). Educational Objective: To know about density Educational Objective: To know about literacy of population. rate.

17. Key: A Explanation: Due to availability of fertile alluvial soil intensive agricultural practices was possible in the Nile River and islands of Java. So, in these regions high density of population is found.

www.laexias.com Page No. 186 https://elearn.laex.in Geography UPSC Previous Year Questions

Many of the world’s small island or isolated community in India. Most Buddhist Marathi states have large populations for their size. people belong to the former Mahar community Macao, Monaco, Singapore, Hong Kong and Educational Objective: To know about Gibraltar are the five most densely populated. Population distribution across states. Educational Objective: To know about 20. Key: D population and economic activity. Explanation: Megacities of India have more 18. Key: C than 5 million populations. Explanation: Decadal population growth rate of But not all the major megacities are located on religious groups: Hindus: 16.8%; Muslim: 24.6%; the sea coast or are sea ports. Delhi, which is Christian: 15.5%; Sikh: 8.4%; Buddhist: 6.1% the national capital, is not a sea port. and Jain: 5.4%. To qualify as a megacity under the UN definition, Among the given options Christians Decadal an urban area must have a population of 10 population growth slightly matches the data million people. The UN takes into account given in the above table. urban sprawl and measures populations beyond Census-2011 official city limits. On these criteria, India Out of total population: Hindu (79.8%) currently has five megacities. >Muslim(14.2%) > Christian(2.3%) > Sikh (1.7%) 1. New Delhi: The capital city has a population > Buddhist (0.7%) > Jain(0.4%) >Parsis (0.06%) of 26.5 million people > Animist & others (0.72%) 2. Mumbai: India’s financial hub has a Educational Objective: To know about population of 21.4 million people population distribution. 3. Kolkata: An important trading hub, with 15 million people living 19. Key: C 4. : The ‘’ of India; 10.5 Explanation: Maharashtra has the highest Bengaluru number of Buddhists in India, with 5.81% of the million people call it home total population. 5. Chennai: Home of the Indian motor industry, The Buddhist percentage has decreased from with 10.2 million people 0.74% in 1961 to 0.70% in 2011. Between 2001 Educational Objective: To know about and 2011, Buddhist population declined in Uttar Mega cities. Pradesh, Karnataka, and Punjab. 21. Key: D According to the 2011 Census of India there are Explanation: According to 1991 census data, 8.4 million Buddhists in India. Maharashtra has Bihar had the lowest rate of literacy at 47.53%. the highest number of Buddhists in India, with As of 2011 Census India's literacy rate is at 5.81% of the total population. Almost 90 per 74.04%. Kerala has achieved a literacy rate of cent of Navayana or Neo-Buddhists live in the 93.91%. Bihar is the least literate state in India, state. Marathi Buddhists, who live in with a literacy of 63.82%. Maharashtra, are the largest Buddhist

www.laexias.com Page No. 187 https://elearn.laex.in Geography UPSC Previous Year Questions

India’s density of population is 382 per square the state is all set to achieve “Zero population kilometre. Bihar is the most thickly populated growth” in the coming years state (1106 persons/sq km.) followed by west While the national rate of growth of population bengal-1028 and Kerala 860. in the last 10 years is 17.6 per cent, the growth Educational Objective: To Know about the rate of state population during the period is 4.9 Literacy Rate. per cent, the lowest rate among Indian states, according to the latest Economic Review by the 22. Key: D state planning Board. Explanation: Self explanatory from the above India's literacy rate is at 74.04%. Kerala has table achieved a literacy rate of 93.91%. 23. Key: C Kerala ranks highest in India with respect to Explanation: Human development in a country social development indices such as elimination depends on life expectancy of the individual and of poverty, primary education and healthcare. standard of living of the population. Among This resulted from significant efforts begun in Asian countries Korea is the best performer in 1911 by the erstwhile Princely states of human development index. Its infant mortality Cochin and Travancore to boost healthcare and rate is 6 and literacy rate is 98%. education among the people Educational Objective: To know about human Educational Objective: To know about human development. development 24. Key: C 26. Key: B Explanation: ‘A’ marked area in the given map Explanation: According to the recent data is Punjab and Haryana, where Sikhs as the made available by Central Statistical largest religious minorities. Organisation, the incidence of poverty is highest ‘B’ marked area denoted part of Rajasthan and in tribes of Bihar, Orissa and Madhya Pradesh. Gujarat, where Jains are the largest religious India has halved its poverty rate since the 1990s minority. and achieved a seven plus growth rate over the ‘C’ marked area denoted Sanchi area of Madhya last 15 years, said the World Bank. Pradesh, where Buddhists are the religious Educational Objective: To know about Poverty minority. ‘D’ refers the part of Andhra Pradesh where 27. Key: B Christians are religious minority. Explanation: As per 1991 census, Pondicherry and Delhi had the literacy rates of 75.3 and 74.7 Educational Objective: To know about minority respectively. population distribution. As of 2011 Census, literacy rate of Union 25. Key: C territories of India Lakshadweep (92.28%) > Daman &Diu (87.07%) Explanation: With Kerala registering the lowest > Puducherry (86.55%) > Chandigarh (86.43%) > growth rate of population in the last one decade, NCT of Delhi (86.34%) > Andaman &

www.laexias.com Page No. 188 https://elearn.laex.in Geography UPSC Previous Year Questions

Nicobar(86.27%) > Dadra & Nagar Haveli raised in an agricultural setting to produce labor (77.65%). and commodities such as meat, eggs, milk, fur, Educational Objective: To know about Literacy leather, and wool. Rate. Educational Objective: To know about Livestocks of India. 28. Key: D Explanation: Southeast Asia occupies a 30. Key: C strategic position for traffic linking the Pacific Explanation: As per India’s National Population and the Indian Oceans, such as the Straits of Policy - 2000, by 2045 India will achieve Malacca, the . Hence statement population stability whichmeans that the size of d is the correct answer. population will not go up. The countries in Southeast Asia are making In 1952, India was the first country in the world efforts to achieve political stability and steady to launch a national programme, emphasizing economic growth, and lately have realized overall family planning to the extent necessary for economic development to varying degrees. Such reducing birth rates "to stabilize the population economic development has deepened at a level consistent with the requirement of interdependence within and outside the region. national economy". Educational Objective: To know about south Educational Objective: To know about national east Asian region. population policy.

29. Key: C 31. Key: C Explanation: India is- Explanation: India was the first country in the ● World’s highest livestock owner at about world to launch a national programme. Family 535.78 million. Planning programme was started in 1952 by ● First in the total buffalo population in the Government of India. world – 109.85 million buffaloes. Kerala is the first state inIndia to achieve ● Second in the population of goats – 148.88 replacement level of fertility. million goats The National Population Policy, 2000 (NPP 2000) ● Second largest poultry market in the world. affirms the commitment of government towards ● The second-largest producer of fish and also voluntary and informed choice and consent of the second-largest aquaculture nation in the citizens while availing of reproductive health world. care services, and continuation of the target free ● Third in the population of sheep (74.26 approach in administering family planning million). services. The NPP 2000 provides a policy ● Fifth in the population of ducks and chicken framework for advancing goals and prioritizing (851.81 million). strategies during the next decade, to meet the reproductive and child health needs of the ● Tenth in camel population in the world – 2.5 people of India, and to achieve net replacement lakhs levels (TFR) by 2010. Livestock is defined as the farm animals raised In 1952, India was the first country in the world especially in a farm for the use of humankind. to launch a national programme, emphasizing Livestock is commonly domesticatedanimals family planning to the extent necessary for

www.laexias.com Page No. 189 https://elearn.laex.in Geography UPSC Previous Year Questions

reducing birth rates "to stabilize the population 3) They are kept in the category of Scheduled at a level consistent with the requirement of Tribes national economy". Which of the statements given above is/are Educational Objective: To know about national correct? family planning program. (a) 1 only (b) 2 and 3 only 4. Tribes (c) 3 only 1. Match List-I (Ethnic Community) with (d) 1,2 and 3

List-II (Country) and select the correct 4. Consider the following pairs: answer: Tribe State List-I List-II 1) Limboo (Limbu) Sikkim A. Apatani 1. China 2) Karbi Himachal Pradesh B. Dayak 2. India 3) Dongaria Kondh Odisha C. Dinka 3. Indonesia 4) Bonda Tamil Nadu D. Uighur 4. Sudan Which of the above pairs are correctly Codes: matched? (a) A-2; B-3; C-4; D-1 (a) 1 and 3 only (b) A-3; B-2; C-4; D-1 (b) 2 and 4 only (c) A-2; B-3; C-1; D-4 (c) 1,3 and 4 only (d) A-3; B-2; C-1; D-4 (d) 1,2,3 and 4

2. Every year, a month-long ecologically 5. In which one of the following places is important campaign/ festival is held the Shompen tribe found? during which certain communities/tribes (a) Nilgiri Hills plant saplings of fruit- bearing trees. (b) Nicobar Islands Which of the following are such (c) Spiti Valley communities/ tribes? (d) Lakshadweep Islands (a) and Lepcha 6. Which one of the following statements is (b) Gond and Korku not correct? (c) Irula and Toda (a) There is no definition of the Scheduled Tribe (d) Sahariya and Agariya in the Constitution of India 3. With reference to “Changpa” community (b) North-East India accounts for a little over of India, consider the following half of the country’s tribal population statements: (c) The people known as Todas live in the 1) They live mainly in the State of Uttarakhand Nilgiri area 2) They rear the Pashmina goats that yield a (d) Lotha is a language spoken in Nagaland fine wool

www.laexias.com Page No. 190 https://elearn.laex.in Geography UPSC Previous Year Questions

7. In which one of the following Union of percent of scheduled tribe population Territories, do the people of the Onge to their total population is: tribe live? (a) Andaman and Nicobar Island (b) Dadra and Nagar Haveli (c) Daman and Diu (d) Lakshadweep

8. Which one of the following pairs of primitive tribes and place of their inhabitation is NOT correctly matched? (a) 1,3,2,4 (a) Buksa : Paun-Garhwal (b) 3,2,1,4 (b) Kol : Jabalpur (c) 3,1,4,2 (c) Munda : Chhotanagpur (d) 1,3,4,2 (d) Korba : Kodagu

9. A person of mixed European and Indian 4. Key and Explanation blood in Latin America is called a (a) Mulatto 1. Key: A (b) Mestizo Explanation: The Apatani, or Tanw, are a tribal (c) Meiji group of people living in the Ziro valley in the (d) Mau Mau Lower Subansiri district of Arunachal Pradesh 10. The term “Aryan” denotes in India. (a) An ethnic group Dayak tribe’s people are the indigenous nomadic (b) A nomadic people farmers of Sarawak and Brunei, Indonesia. They (c) A speech group are fierce head hunters (d) A superior race Dinka tribe is the pastoralist tribe in South Sudan. They are currently fighting with the 11. Which one of the following pairs of Sudan People’s Liberation Army (SPLA) on behalf states & tribes is not correctly matched? of South Sudan President Salva Kiir Mayardit, a (a) Assam : Miri Dinka himself. (b) Nagaland : Konyak Uighurs are a Turkic minority ethnic group of (c) Arunachal Pradesh: Apatani Central Asia. They are recognized as native to (d) Madhya Pradesh : Lambada the Xinjiang Province in Northwest China. 12. Among the Indian states shown labelled Educational Objective: To know about 1,2,3& 4 in the rough outline map given, important tribes all over the world. the correct sequence of descending order 2. Key: B

www.laexias.com Page No. 191 https://elearn.laex.in Geography UPSC Previous Year Questions

Explanation: The festival starts with the green Changpa are a semi-nomadic community found march celebrating the beginning of the rains and in Ladakh region. They are kept in the category new sowing season during which planting of of Scheduled Tribes. sapling of fruit bearing trees is done on mass Educational Objective: To know about scale. Changpa tribes. This question is based upon recent trends in 4. Key: A Gond and Korku tribes in Madhya Pradesh of Explanation: Limbu tribe is from Sikkim. planting fruit tree saplings conciding with Hari Tamang and the Limbu communities of Sikkim jiroti festival. have been placed in the ST category. To combat the twin problems of malnutrition Dongaria Kondh tribe is from Odisha. and environmental degradation, of Karbi is from Assam and Harda and Betul districts of Madhya Pradesh Bonda is in Odisha. have decided to launch ‘Operation Guerrilla Assam: Chakma, Chutiya, Dimasa, Hajong, Green’ — a movement to plant large numbers of Garos, Karbi, Khasis, Gangte. fruit bearing trees on vacant land, wherever it is Odisha: Gadaba, Ghara, Kharia, Khond, Matya, available. Oraons, Rajuar, Santhals. Educational Objective: To create awareness, Sikkim: Bhutia, Khas, Lepchas, Limbu, the adivasis and their festivals. Tamang. 3. Key: B Educational Objective: To know about tribes of Explanation: The Changpa are a semi-nomadic India. Tibetan ethnic group found mainly in Zanskar 5. Key: B region of Jammu and Kashmir. They rear the Explanation: Tribes of Andaman and Nicobar Pashmina goats that yield fine wool. They are can be split into two broad tribal groups mainly kept in the category of Scheduled Tribes. based on their place of origin. Changpas, a hardly nomadic community, The Andaman Islands are home to four ‘Negrito’ followed the silk route trail over the high passes tribes namely the Great Andamanese, Onge, and the meandering rivers of the Changthang Jarawa and Sentinelese. plains of the Tibetan Plateau to settle in the high Nicobar Islands are home to two ‘Mongoloid’ altitude lake regions of Ladakh in the valleys of tribes – the Shompen and Nicobarese. , and the Tsomoriri somewhere The Shompen and other aboriginal tribes of the in the Eighth Century. They transmigrated with archipelago are protected under The Andaman the hope of finding better pastures for their and Nicobar (Protection of Aboriginal Tribes) livestock, to trade salt and butter and to find a Regulation, 1956. market for their pashmina wool. Educational Objective: To know about Andaman and Nicobar.

www.laexias.com Page No. 192 https://elearn.laex.in Geography UPSC Previous Year Questions

6. Key: B 8. Key: D Explanation: The term 'Scheduled Tribes' first Explanation: Korba tribe in found in appeared in the Constitution of India. Article Chhotanagpur of Chhattisgarh and Jharkhand. 366 (25) defined scheduled tribes as "such tribes Korba is a tribal majority district situated in the or tribal communities or parts of or groups north-central part of Chhattisgarh whereas within such tribes or tribal communities as are Kodagu in a district in Karnataka. deemed under Article 342 to be ScheduledTribes Bhoksa, also known as Buksa, are indigenous for the purposes of this constitution". peoples living mainly in the Indian states of Bulk of the tribal population is found in 5 states, Uttarakhand and Uttar Pradesh. i.e., Madhaya Pradesh, Maharashtra, Orissa, The Kol are an ancient tribal community, one of Gujarat and Bihar. the original inhabitants of northern and central As per 2011 census India. They are located throughout the states of ST (%): Lakshadweep > Mizoram > Nagaland > Uttar Pradesh, Bihar, Jharkhand, Madhya Meghalaya > Dadra & Nagar Haveli. Pradesh, Maharashtra and Tripura. ST (Absolute): Madhya Pradesh > Maharashtra > Munda people are an ethnic group of India. They Odisha > Jharkhand > Gujarat > Rajasthan speak the as their native ST list is ‘state wise’. Punjab, Haryana, language, which belongs to the Munda subgroup Chandigarh, Delhi & Puducherry have no of . The Munda are notified Scheduled Tribes. found in the northern areas of east India Educational Objective: To know about concentrated in the states of Jharkhand, Odisha 'Scheduled Tribes' distributions. and West Bengal. Chhattisgarh: Agariya, Bhaina, Bhattra, Biar, 7. Key: A Khond, Mawasi, Nagasia. Explanation: Tribes of Andaman and Nicobar Jharkhand: Birhors, , Gonds, Kharia, can be split into two broad tribal groups mainly Mundas, Santhals, Savar. based on their place of origin. Karnataka: Adiyan, Barda, Gond, Bhil, Iruliga, The Andaman Islands are home to four ‘Negrito’ Koraga, Patelia, Yerava. tribes namely the Great Andamanese, Onge, Uttarakhand: Bhotias, Buksa, Jannsari, Khas, Jarawa and Sentinelese. Raji, Tharu. Nicobar Islands are home to two ‘Mongoloid’ Uttar Pradesh: Bhotia, Buksa, Jaunsari, Kol, tribes – the Shompen and Nicobarese. Raji, Tharu. The Onge and other aboriginal tribes of the Educational Objective: To know about tribes of archipelago are protected under The Andaman India. and Nicobar (Protection of Aboriginal Tribes) Regulation, 1956. 9. Key: B Educational Objective: To know about Explanation: Mestizo originally meant a person Andaman and Nicobar. of combined European and American Indian

www.laexias.com Page No. 193 https://elearn.laex.in Geography UPSC Previous Year Questions

descent, regardless of where the person was Explanation: Lambada tribes are found in born. Rajasthan and not in Madhya Pradesh. Mulatto: a person of mixed white and black Assam: Chakma, Chutiya, Dimasa, Hajong, ancestry, especially a person with one white and Garos, Khasis, Gangte. one black parent. Arunachal Pradesh: Apatanis, Abor, Dafla, Meiji: the period when Japan was ruled by the Galong, Momba, Sherdukpen, Singpho. emperor Meiji Tenno, marked by the Madhya Pradesh: Baigas, Bhils, Bharia, Birhors, modernization and westernization of the country. Gonds, Katkari, kharia, Khond, Kol, Murias. Mestizo = European + American Indian descent. Nagaland: Angami, Garo, Kachari, Kuki, Mikir, Educational Objective: To know about tribes of Nagas, Sema. world. Rajasthan: Bhils, Damaria, , Meenas(Minas), Patelia, Sahariya. 10. Key: C Educational Objective: To know about tribes of Explanation: Aryan is in fact a linguistic term India. indicating a speech group of Indo-European origin, and is not an ethnic term. 12. Key: B Drawing on misinterpreted references in Explanation: As per 2011 census the Rig Veda by Western scholars in the 19th ST (%): Lakshadweep > Mizoram > Nagaland > century, the term "Aryan" was adopted as Meghalaya > Dadra & Nagar Haveli. a racial category through the works of Arthur ST (Absolute): Madhya Pradesh > Maharashtra > de Gobineau, whose ideology of race was based Odisha > Jharkhand > Gujarat > Rajasthan on an idea of blonde northern European ST list is ‘state wise’. Punjab, Haryana, "Aryans" who had migrated across the world and Chandigarh, Delhi & Puducherry have no founded all major civilizations, before being notified Scheduled Tribes. diluted through racial mixing with local Constitution of India has recognized tribal populations. communities in India under ‘Schedule 5’ of the Aryan, name originally given to a people who constitution. Hence the tribes recognized by the were said to speak an archaic Indo-European Constitution are known as ‘Scheduled language and who were thought to have settled Tribes’. There are around 705 distinct tribes in in prehistoric times in ancient Iran and the India. northern Indian subcontinent. Educational Objective: To know about Educational Objective: To know about Indo- Scheduled Tribes. European Language.

11. Key: D

www.laexias.com Page No. 194 https://elearn.laex.in Geography UPSC Previous Year Questions

www.laexias.com Page No. 195 https://elearn.laex.in